FNP Certification Exam Questions

अब Quizwiz के साथ अपने होमवर्क और परीक्षाओं को एस करें!

When counseling a patient about lifestyle modifications to prevent migraines, the NP recommends: Using tinted glasses to minimize glare and bright lights Substituting artificial sweeteners for cane sugar Avoiding regular exercise for at least one week after each migraine episode Moving to a high-altitude environment

Ans: Using tinted glasses to minimize glare and bright lights

When evaluating a patient at 12-weeks' gestation, the nurse practitioner anticipates which physical finding? Fetal heart tones undetectable by Doppler Clear nipple discharge Negative Chadwick's sign Uterus fundus palpable directly above symphysis pubis

Ans: Uterus fundus palpable directly above symphysis pubis

Which of the following findings is most consistent with a diagnosis of acute bacterial rhinosinusitis? Fever >100.7ᵒF (38.2ᵒC) Upper respiratory tract infection symptoms persisting for ≥10 days Concomitant presence of facial pressure Copious amounts of nasal discharge

Ans: Upper respiratory tract infection symptoms persisting for ≥10 days

A 22-year-old man presents with new-onset pain and swelling in the feet and ankles as well as conjunctivitis, oral lesions, and dysuria. The most important test to obtain is: Rheumatoid factor. Sedimentation rate. Urethral cultures. Antinuclear antibody.

Ans: Urethral cultures.

When counseling patients to promote hepatitis C prevention, the nurse practitioner realizes the most important measure from the list below is: Safe sex practices. Using single-use injection drug paraphernalia. Strict handwashing. Avoidance of raw seafood ingestion.

Ans: Using single-use injection drug paraphernalia.

Which of the following is most consistent with the diagnosis of herpes zoster? Papules in a dermatomal distribution Low-grade fever and malaise prior to eruption Presence of pustules and honey-crusted lesions A 1-2-day prodrome of pain prior to lesions erupting

ans: A 1-2-day prodrome of pain prior to lesions erupting

First-line therapy to add for a patient with Stage 3 heart failure and preserved left ventricular ejection fraction is: A statin. An alpha blocker. Digitalis. A diuretic.

ans: A diuretic.

Of the following imaging studies, place in rank order from greatest (1) to least (3) amount of ionizing radiation burden to the patient. A. Abdominal ultrasound __ B. Abdominal CT __ C. Abdominal flat plate __ A-1 B-2 C-3 A-3 B-2 C-1 A-3 B-1 C-2 A-2 B-3 C-1

ans: A-3 B-1 C-2

Which of the following medications can be considered for patients at risk for heart failure but without structural heart disease (Stage A)? Beta-blockers Thiazide diuretics ARB or ACEI Digitalis

ans: ARB or ACEI

While reviewing the records of a 3-year-old patient who is new to your practice, you learn that she has a Still's murmur. As a result, you anticipate which of the following physical findings? An audible systolic murmur with a buzzing quality Height and weight in the bottom 25% of the growth chart A hyperdynamic point of maximal impulse Joint laxity and hyperextension

ans: An audible systolic murmur with a buzzing quality

A mentally competent patient with advanced cancer is explained 2 treatment options. He decides to take the less invasive technique, despite counseling him that his choice has a lower chance for success. Respecting the patient's choice demonstrates the ethical principle of: Justice Veracity Paternalism Autonomy

Ans : Autonomy

The most likely diagnosis is: (Please reference the previous question). Hemangioma. Port wine lesion. Melanoma. Erythema toxicum neonatorum.

Ans: Hemangioma.

Potential adverse effects from medications with significant anticholinergic effect include all of the following except: Hypothermia Dry mouth, skin Blurred vision Urinary retention

Ans: Hypothermia

Sulfonylureas are characterized by their impact on: Insulin release. Insulin receptor site activity. Hepatic glucose production. Renal glucose excretion.

Ans: Insulin release.

Which of the following agents can be considered for migraine controller therapy in a 54-year-old with a history of hypertension? Sumatriptan Topiramate NSAIDs Ergotamine

Ans: Topiramate

Ms. Kane is a 25-year-old woman who presents with finger-shaped ecchymotic areas on her right shoulder that are an incidental finding during a physical examination. She denies abuse or assault. The NP's most appropriate response is: "Your bruises look as if they were caused by someone grabbing you." "Was this an accident?" "I notice the bruises are in the shape of a hand." "How did you fall?"

Ans: "I notice the bruises are in the shape of a hand."

Osteopenia is defined as a spine, hip, or forearm bone mass density as determined by DXA devices that is ____ standard deviation (SD) below that of a young adult norm. <1 1 1-2.5 ≥2.5

Ans: -1 - -2.5

Raymond is an 18 year-old high school basketball player. He is in for a sports clearance physical examination and denies activity intolerance, chest pain, and syncope. Today's Assessment Reveals the Following: 74" (188 cm), 220 lb (99.8 kg), Tanner Stage 5. Both heart sounds intact, a grade 2/6 harsh systolic murmur with radiation to the neck, loudest along USB, softer towards axilla, somewhat softer with position change from supine to standing, no delay in carotid upstroke, other pulses intact. Raymond's findings are most likely the result of: Aortic stenosis. Aortic regurgitation. Mitral valve prolapse. Physiologic murmur.

Ans: Aortic stenosis.

For an individual to demonstrate the capability to make an informed healthcare decision, all of the following must be present except: The patient has the ability to understand the nature of the clinical condition. The patient can interpret the information presented. The patient is able give a reason for the choice if asked. The patient does not need to be able to communicate what the care preference is.

Ans: The patient does not need to be able to communicate what the care preference is.

A 6-year-old boy returns to the clinic with his mother 2 months following an episode of acute otitis media that was treated with a regimen of amoxicillin (Amoxil®). Examination reveals persistent effusion and mild bulging of the tympanic membrane, but there is no observed redness. The patient reports little-to-no pain in the affected ear and he is without fever. The most appropriate management option for this patient is: Watchful waiting. A course of an oral decongestant. 1 week therapy with amoxicillin-clavulanate (Augmentin®). Cefpodoxime (Vantin®) at a therapeutic dose for 5 days.

Ans: Watchful waiting.

A 37-year-old female presents with flank pain, fever, and vomiting. Physical examination is significant for costovertebral angle tenderness. A urinalysis is likely to reveal: Urobilinogen. Specific gravity <1.010. White blood cell (WBC) casts. Ketones.

Ans: White blood cell (WBC) casts.

In a population of 1 million people, 10,000 are diagnosed with peripheral arterial disease (PAD) and 500 new cases are diagnosed each year. The prevalence of PAD in this population is: 1%. 10%. 500. 1/2000 persons.

ans: 1% 10,000/1,000,000= 0.001 x 100% = 1%

Match the viral hepatitis type with the most common method of acquisition. An option can only be used once. A. Hepatitis A B. Hepatitis B C. Hepatitis C 1. Fecal-contaminated food or water __ 2. Injection Drug Use __ 3. Sexual Contact __ 1-C 2-B 3-A 1-B 2-A 3-C 1-A 2-B 3-C 1-A 2-C 3-B

ans: 1-A 2-C 3-B

The typical international normalized ratio (INR) goal during warfarin therapy for a 65-year-old woman with atrial fibrillation is: 1.0‒2.0. 1.5‒2.5. 2.0‒3.0. 2.5‒3.5.

ans: 2.0‒3.0.

Mrs. Garcia is a 78-year-old woman who presents with a chief complaint of progressively increasing aches, limited to her hands and fingers, particularly after doing yard work. Otherwise, she denies musculoskeletal problems. Physical examination reveals bilateral Heberden's and Bouchard's nodes. 1. Identify the location of the Heberden's nodes. _____ 2. Identify the location of the Bouchard's nodes. _____ 1-A 2-B

ans: Bouchar's node is proximal interphalengeal joint of the hands in RA Heberden's node is distal interphalengeal joint of the hands and fingers

Which of the following drug classes will yield the greatest decrease in blood pressure for a middle-aged African American male? Calcium channel blocker Beta-blocker ACE inhibitor Angiotensin receptor blocker

ans: Calcium channel blocker

All of the following are examples of environmental control for a patient with allergic rhinitis except: Keeping pets out of the bedroom. Laundering sheets in hot water. Encasing pillows in plastic covers. Keeping bedroom windows open in the early morning.

ans: Keeping bedroom windows open in the early morning.

Treatment for a 45-year-old woman with pyelonephritis who is otherwise healthy and has no sulfa allergy is: Nitrofurantoin (Macrobid®) Amoxicillin (Amoxil®) Levofloxacin (Levaquin®) Fosfomycin (Monurol®)

ans: Levofloxacin (Levaquin)

A 32-year-old woman is diagnosed with a urinary tract infection. She reports never having a UTI previously. Her records indicate that she is allergic to sulfa medications. You recommend treatment with: Amoxicillin (Amoxil®) Nitrofurantoin (Macrobid®) Moxifloxacin (Avelox®) Ceftriaxone (Rocephin®)

ans: Nitrofurantoin (Macrobid)

You see a 46-year-old woman with hypothyroidism who is currently being treated with levothyroxine at 100 mcg/d. She insists that she takes her medicine every morning. A laboratory test reveals that her TSH is 0.3 mIU/L (NL=0.4-4.0 mIU/L). You recommend: Maintaining the levothyroxine dose and reevaluate in 2 months Increasing the levothyroxine dose and reevaluate in 2 months Decreasing the levothyroxine dose and reevaluate in 2 months Discontinuing levothyroxine therapy and reevaluate in 6 months

ans: Decreasing the levothyroxine dose and reevaluate in 2 months

A person with a latent tuberculosis infection: (True/False) 1. Does not exhibit any signs or symptoms of active infection. __ 2. Can spread the disease to others. __ 3. Will have a positive tuberculin skin test. __ 4. Will have >50% lifetime risk of developing active tuberculosis. __ 1-False 2-True 3-True 4-False 1-True 2-True 3-True 4-True 1-False 2-True 3-False 4-False 1-True 2-False 3-True 4-False

1-True 2-False 3-True 4-False

According to current nationally recognized recommendations (True/False): 1. Screening for lung cancer with a low-dose CT scan is indicated for a 62-year-old man in good health who is a current smoker and has a 40 pack-year smoking history. __ 2. PSA testing is indicated for a 68-year-old man with end-stage heart failure. __ 3. PV test and Pap test are recommended for a 37-year-old woman who had negative cervical cytology results 5 years ago. __ A 24-year-old woman who completed the HPV vaccination regimen 2 years ago is not recommended to undergo cervical cancer screening until she reaches 30 years of age. __ 1-True 2-False 3-False 4-True 1-True 2-False 3-True 4-True 1-True 2-False 3-True 4-False 1-False 2-False 3-False 4-True

1-True 2-False 3-True 4-False

Suicide: True or False? 1. Males represent nearly 80% of all completed suicides. __ 2. When compared with male suicide attempts, female attempts at suicide are approximately 2-3 times more common. __ 3. The highest rate of completed suicide is found in teenage males. __ 4. Inquiring about suicidal ideation could precipitate the act. __ 1-True 2-True 3-False 4-False 1-True 2-Flase3-False 4-True 1-False 2-True 3-False 4-False 1-False 2-False 3-False 4-False

1-True 2-True 3-False 4-False

In a community undergoing a disaster, the use of select vaccines will be prioritized. Match the type of disaster with the priority vaccine: Immunization against tetanus Immunization against influenza Immunization against Hepatitis A 1. Uninjured adults and children evacuated to a crowded group setting __ 2. Adults with multiple deep lacerations from flying debris __ 3. A community exposed to unsafe water supply after a hurricane __ 1. Immunization against tetanus 2. Immunization against influenza 3. Immunization against Hepatitis A 1. Immunization against influenza 2. Immunization against tetanus 3. Immunization against Hepatitis A 1. Immunization against tetanus 2. Immunization against Hepatitis A 3. Immunization against influenza 1. Immunization against Hepatitis A 2. Immunization against tetanus 3. Immunization against influenza

1. Immunization against influenza 2. Immunization against tetanus 3. Immunization against Hepatitis A

Ana is a 22-year-old well woman who presents with a four-day history of an intense itch with skin lesions on hands and arms as well as her right cheek. She has used an over-the-counter hydrocortisone cream on the affected area with little effect, and denies any other symptoms or previous history of similar rash. Ana denies recent travel and exposure to new creams, soaps, or medications. She works as a landscaper. When considering a diagnosis of phytodermatitis due to exposure to urushiol (poison ivy, poison oak, poison sumac), you anticipate finding three of the following. Fever and generalized malaise Vesicles Crusts Fissures Most of the lesions in a linear pattern

Ans: Vesicles Crusts Fissures

Match the best next test to order in an adult with anemia based on hemogram results. 1. Ferritin __ 2. Vitamin B12 and folate __ 3. Hemoglobin electrophoresis __ MVC ↓ MCHC ↓ RWD ↑ MVC ↓ MCHC ↓ RWD NL MCV ↑ MCHC NL RWD ↑ 1-A 2-B 3-C 1-C 2-B 3-A 1-A 2-C 3-B 1-B 2-A 3-C

Ans: 1-A 2-C 3-B

The function of which of the following cranial nerves is being tested with the following patient requests? A. CN I B. CN XI C. CN III D. CN VII E. CN XII 1. Puff out your cheeks. __ 2. Do you recognize this scent? __ 3. Without moving your head, follow my finger with your eyes. __ 4. Shrug your shoulders. __ 5. Stick out your tongue. __ 1-D 2-A 3-C 4-B 5-E 1-B 2-A 3-D 4-C 5-E 1-A 2-E 3-C 4-D 5-B 1-C 2-D 3-B 4-A 5-E

Ans: 1-D 2-A 3-C 4-B 5-E

Which of the following is most likely to be found on the funduscopic exam in the person with angle-closure glaucoma? A deeply-cupped optic disc Arteriovenous nicking Papilledema Hemorrhagic lesions

Ans: A deeply-cupped optic disc

A 54-year-old man of Asian ancestry presents with the following hemogram: -Hg=11.6 g/dL (12-14 g/dL) (116 g/L [120-140 g/dL]) -Hct=37% (42-48%) (0.37 proportion [0.42-0.48 proportion]) -MCHC=24 g/dL (31-37 g/dL) (260 g/L [310-370 g/L]) -MCV=64 fL (80-96 fL) -RDW=14% (11%-15%) (0.14 proportion [0.11-0.15 proportion]) -RBC=6.6 million/mm3 (4.7-6.1 million/mm3) These findings are most consistent with: Folate deficiency. Acute blood loss. Alpha thalassemia minor. Iron deficiency anemia.

Ans: Alpha thalassemia minor.

A 67-year-old man with well-controlled hypertension and dyslipidemia who received one dose of PPSV23 (Pneumovax®) 1 year ago asks about a "new pneumonia shot" he heard about in a TV promotion. You advise that: He is adequately immunized against pneumococcal disease. That there is another pneumococcal vaccine available but its use is not advised in older adults due to its adverse effect profile. He is eligible for a dose of PCV13 (Prevnar®) today to be optimally protected against pneumococcal disease. In 4 more years, he should receive a second dose of PPSV23.

Ans: He is eligible for a dose of PCV13 (Prevnar®) today to be optimally protected against pneumococcal disease.

A 68-year-old woman presents with a 6-month history of increasingly severe peripheral numbness and oral irritation. Hemogram results are as follows. -Hb=6.2 g/dL (62 g/L) -Hct=20% (0.2 proportion) -RBC=2.1 million/mm3 -MCV=132 fL -MCHC=32.4 g/dL (324 g/L) -RDW=19% (0.19 proportion) Physical exam reveals pale conjunctiva, a grade 2/6 systolic ejection murmur over the precordium without radiation, and a smooth, red tongue. In the above-mentioned patient has not been present on previous examination. As a result, you consider that this is likely a _________ murmur and will resolve with anemia treatment. Pathologic Hemic Venous hum

Ans: Hemic

Risk factors for Edgar's condition include: Based on the below scenario: Edgar is a 75-year-old man with a 60 pack-year history of cigarette smoking and COPD who presents with a chief complaint of a "sore" on the base of his tongue. This lesion has been present for a number of months, remaining relatively stable in size and is not painful. Physical examination reveals a painless ulcerated lesion with indurated margin and is accompanied by a firm, nontender submandibular node. His current medications include inhaled corticosteroids with a long-acting beta2-agonist. Recent high-risk sexual contact. Current use of an inhaled corticosteroid. Long-term HPV-16 infection. Chronic irritation from poorly-fitting dentures.

Ans: Long-term HPV-16 infection.

Primary care of Sarah, a 27-year-old woman with beta thalassemia minor, should include: Prescribing a low-dose iron supplement to counteract microcytosis. An evaluation of hemoglobin electrophoresis every 5 years. Advising taking a multivitamin with high-dose folate supplementation daily. Offering genetic counseling prior to pregnancy.

Ans: Offering genetic counseling prior to pregnancy.

Immunization against herpes zoster is recommended by the CDC for all adults: ≥50 years. ≥55 years. ≥60 years. ≥65 years.

Ans: ≥60 years.

Rank the following from highest (1) to lowest (4) level of research design. __Case reports __Meta-analysis __Randomized controlled trials __Expert opinion 1-Case reports 2-Meta-analysis 3-Randomized controlled trials 4-Expert opinion 2-Case reports 1-Meta-analysis 3-Randomized controlled trials 4-Expert opinion 3-Case reports 1-Meta-analysis 2-Randomized controlled trials 4-Expert opinion 4-Case reports 3-Meta-analysis 2-Randomized controlled trials 1-Expert opinion

Ans: 3-Case reports 1-Meta-analysis 2-Randomized controlled trials 4-Expert opinion

Match each treatment option with the relevant condition. A. Permethrin lotion B. Medium-potency topical corticosteroid C. Imiquimod cream D. Topical ketoconazole E. Topical metronidazole 1. Psoriasis Vulgaris __ 2. Scabies __ 3. Verruca Vulgaris __ 4. Tinea Pedis __ 5. Rosacea __ 1-C 2-B 3-A 4-D 5-E 1-D 2-C 3-A 4-B 5-E 1-A 2-B 3-C 4-D 5-E 1-B 2-A 3-C 4-D 5-E

Ans: 1-B 2-A 3-C 4-D 5-E Medium-dose topical corticosteroid - psoriasis vulgaris permethrin lotion - scabies Imiquimod cream - verruca vulgaris topical ketoconazole - tinea pedis topical metronidazole - rosacea

Identify the following types of skin lesions with its corresponding description: A. Macule B. Papule C. Plaque D. Cyst E. Wheal F. Purpura 1. Single, uniformly brown-colored, slightly raised, irregular-shaped with defined borders, 6 mm in diameter. Patient states "That mole on my shoulder hasn't changed in years." __ 2. Single, flat, non-palpable area of discoloration, irregularly-shaped, and 0.5 cm at the widest diameter. Patient states "I've had that spot on my lower lip for years." __ 3. Single, firm, smooth, raised, dome-shaped, fluid-filled, flesh-colored encapsulated lesion of 1.5 cm in diameter on back of neck. Patient states "A smelly, liquid leaks out of it sometimes." __ 4. Raised, irregular-shaped with defined borders, different color than surrounding skin, patches of >2 cm in diameter, located over the knees. Patient states "These patches have been here for years. When I pick at one, it bleeds just a drop or two." __ 5. Flat, non-blanchable, confluent, purple-colored irregular-shaped lesions on skin ranging 2?20 mm in size. Patient states "These purple splotches appeared on my arms and legs following a round of chemotherapy." __ 6. Clustered, smooth, slightly-raised, circumscribed, pruritic skin-colored lesions of various sizes up to 2 cm, surrounded by area of erythema. Patient states "This itchy rash appeared all over my body a few days after starting my antibiotic." __ 1-C 2-B 3-A 4-E 5-F 6-D 1-B 2-F 3-A 4-D 5-C 6-D 1-B 2-A 3-D 4-C 5-F 6-E 1-A 2-E 3-D 4-B 5-F 6-C

Ans: 1-B 2-A 3-D 4-C 5-F 6-E

Match each vision alteration with the most likely etiology. A. Macular degeneration B. Untreated open-angle glaucoma C. Proliferative diabetic retinopathy 1. Peripheral vision loss __ 2. Floating spots in visual field Macular degeneration Untreated open-angle glaucoma __ 3. Central vision loss __ 1-B 2-C 3-A 1-A 2-B 3-C 1-C 2-B 3-A 1-B 2-A 3-C

Ans: 1-B 2-C 3-A

Match each description with the most appropriate diagnosis. A. Psoriasis B. Viral exanthem C. Basal cell carcinoma 1. A 64-year-old ex-truck driver who reports a persistent open sore on his left forearm that repeatedly bleeds and then crusts over. __ 2. A 36-year-old man who presents with a raised patch of skin on his right elbow covered with silvery-white scaly skin. He reports the area is "very itchy." __ A 14-month-old unimmunized girl without recent medication use who presents with a diffuse rash and low-grade fever. __ Match each description with the most appropriate diagnosis. A. Psoriasis B. Viral exanthem C. Basal cell carcinoma 1-C 2-A 3-B

Ans: 1-C 2-A 3-B

Match the following select ophthalmologic tests with the appropriate indication. A. Amsler grid test B. Snellen chart C. Slit-lamp examination D. Tonometry 1. Evaluation of anterior eye structures, including cornea, conjunctiva, sclera, and iris __ 2. General visual acuity screen __ 3. Early detection of macular degeneration __ 4. Measurement of intraocular pressure, glaucoma screening test __ 1-B 2-C 3-D 4-A 1-C 2-B 3-A 4-D 1-A 2-C 3-B 4-D 1-D 2-C 3-B 4-A

Ans: 1-C 2-B 3-A 4-D

dentify each condition with the most likely location. A. Pityriasis rosea B. Psoriasis vulgaris C. Actinic keratosis D. Scabies E. Eczema 1. Antecubital fossa __ 2. Anterior surface of knees __ 3. Sun-exposed areas __ 4. Over waistband area __ 5. Usually preceded by herald patch on the trunk __ 1-A 2-B 3-D 4-E 5-C 1-E 2-B 3-C 4-D 5-A 1-C 2-B 3-D 4-E 5-A 1-D 2-A 3-B 4-E 5-C

Ans: 1-E 2-B 3-C 4-D 5-A 1) eczema 2) psoriasis vulgaris 3) actinic keratosis 4) scabies 5) pityriasis rosea

Match each dermatologic term with the description. A. Annular B. Scattered C. Confluent or coalescent D. Clustered E. Linear 1. In streaks such as the typical phytodermatitis caused by exposure to plant oil (urushiol) contained in poison ivy, poison oak, poison sumac. __ 2. Occurring in a group without pattern, such as the lesions seen in an outbreak of herpes simplex type 1 (HSV-1, "cold sore"). __ 3. Generalized over body without a specific pattern or distribution, as seen in a viral exanthem such as rubella or roseola. __ 4. Multiple lesions blending together, such as the lesions seen in psoriasis vulgaris. __ 5. In a ring, often seen in the characteristic "Bull's Eye" lesion seen in Lyme disease. __ 1-A 2-D 3-E 4-C 5-B 1-B 2-C 3-E 4-D 5-A 1-E 2-D 3-B 4-C 5-A 1-C 2-A 3-B 4-D 5-E

Ans: 1-E 2-D 3-B 4-C 5-A

Untreated hyperthyroidism or hypothyroidism or both? Hypo Hyper Hypo/Hyper 1. Dry Skin __ 2. Fine tremor __ 3. Hypoactive deep tendon reflexes (DTR) __ 4. Mood or mentation change __ 5. Menorrhagia __ 6. Exophath1- Hyper 3-Hyper 4-Hypo/Hyper 5-Hypo 6-Hypo/Hyperalmos __ 1-Hyper 2-Hyper 3-Hyper 4-Hypo/Hyper 5-Hypo 6-Hypo/Hyper 1-Hypo/Hyper 2-Hypo 3-Hyper 4-Hypo/Hyper 5-Hypo 6-Hypo/Hyper 1-Hypo 2-Hyper 3-Hypo 4-Hypo/Hyper 5-Hypo 6-Hyper 1-Hyper 2-Hypo 3-Hypo/Hyper 4-Hypo 5-Hypo 6-Hypo/Hyper 1-Hypo 2-Hyper 3-Hyper 4-Hypo/Hyper 5-Hypo 6-Hypo/Hyper

Ans: 1-Hypo 2-Hyper 3-Hypo 4-Hypo/Hyper 5-Hypo 6-Hyper

According to current nationally-recognized recommendations, are the following cancer screenings indicated? Yes or No 1. Annual digital rectal exam as colorectal cancer screening in a 63-year-old man __ 2. An initial liquid-based Pap test with HPV cotesting in a 19-year-old woman who is one year post-coitarche __ 3. Endometrial biopsy in a 52-year-old woman who is two years post-LMP and who denies vaginal bleeding __ 4. Annual prostate specific antigen testing in an 81-year-old man who has hypertension and benign prostatic hypertrophy __ 5. Lung cancer screening with low-dose CT (LDCT) for a 60-year-old who is generally in good health who has a 35 pack-year cigarette smoking history who quit smoking 5 years ago __ 1-Yes 2-Yes 3-No 4-Yes 5-No 1-No 2-No 3-No 4-Yes 5-No 1-No 2-Yes 3-No 4-Yes 5-No 1-No 2-No 3-No 4-No 5-Yes

Ans: 1-No 2-No 3-No 4-No 5-Yes

In a 46-year-old woman with hypertension and dyslipidemia, choose the correct prevention: Primary prevention Secondary prevention Tertiary prevention Counseling about reducing risk for sexually transmitted infection __ Skin survey for precancerous lesions __ 1-Primary prevention 2-Secondary prevention 1-Secondary prevention 2-Primary prevention 1-Primary prevention 2-Tertiary prevention 1-Tertiary prevention 2-Secondary prevention

Ans: 1-Primary prevention 2-Secondary prevention

In a 76-year-old man with chronic obstructive pulmonary disease (COPD), choose the correct prevention: Primary prevention Secondary prevention Tertiary prevention 1. Ensuring adequate illumination at home __ 2. Screening for physical, emotional, or financial abuse __ 1-Primary prevention 2-Secondary prevention 1-Primary prevention 2-Tertiary prevention 1-Tertiary prevention 2-Secondary prevention 1-Secondary prevention 2-Tertiary prevention

Ans: 1-Primary prevention 2-Secondary prevention

In a 66-year-old woman with type 2 diabetes mellitus (T2DM), choose the correct prevention: Primary prevention Secondary prevention Tertiary prevention 1. Administering influenza vaccine __ 2. Adjusting therapy to enhance glycemic control __ 1-Tertiary prevention 2-Primary prevention 1-Secondary prevention 2-Primary prevention 1-Primary prevention 2-Secondary prevention 1-Primary prevention 2-Tertiary prevention

Ans: 1-Primary prevention 2-Tertiary prevention

Match each example with the appropriate communication skill. Broad opening/open-ended question Clarification Active listening Restating 1. "You said that you got angry when your boyfriend came home late." __ 2. Use of eye contact and non-verbal cues. __ 3. "What's on your mind?" __ 4. "I don't think I see the connection. Can you explain that to me again?" __ 1-Broad opening/open-ended question 2-Clarification 3-Active listening 4-Restating 1-Broad opening/open-ended question 2-Restating 3-Active listening 4-Clarification 1-Restating 2-Clarification 3-Active listening 4-Broad opening/open-ended question 1-Restating 2-Active listening 3-Broad opening/open-ended question 4-Clarification

Ans: 1-Restating 2-Active listening 3-Broad opening/open-ended question 4-Clarification

In a 25-year-old well woman with a strong family history of T2DM, choose the correct prevention: Primary prevention Secondary prevention Tertiary prevention 1. Checking fasting lipid profile __ 2. Teaching the benefits of participating in a consistent program of moderate-intensity physical activity __ 1-Primary prevention 2-Secondary prevention 1-Tertiary prevention 2-Secondary prevention 1-Secondary prevention 2-Primary prevention 1-Primary prevention 2-Tertiary prevention

Ans: 1-Secondary prevention 2-Primary prevention

Match each definition with the appropriate term: Prevalence Incidence False positive False negative Specificity Sensitivity 1. Refers to the ability of a test to correctly identify those with a condition. __ 2. Refers to the ability of a test to correctly identify those without a condition. __ 3. Proportion or number of individuals with a condition at a given time. __ 4. Number of new cases occurring within a period of time. __ 1-Prevalence 2-False negative 3-Sensitivity 4-Incidence 1-Sensitivity 2-Specificity 3-Prevalence 4-Incidence 1-False positive 2-False negative 3-Specificity 4-Incidence

Ans: 1-Sensitivity 2-Specificity 3-Prevalence 4-Incidence

A person with a latent tuberculosis infection (True/False): 1. Does not exhibit any signs or symptoms of active infection. __ 2. Can spread the disease to others. __ 3. Will have a positive tuberculin skin test. __ 4. Will have >50% lifetime risk of developing active tuberculosis. __ 1-False 2-True 3-False 4-False 1-True 2-True 3-False 4-False 1-True 2-False 3-True 4-False 1-False 2-False 3-False 4-False

Ans: 1-True 2-False 3-True 4-False

The use of all of the following nutritional supplements is potentially associated with increased bleeding risk and should be discontinued at least 7‒10 days prior to elective surgical procedure. (True/False) 1. Ginseng __ 2. Gingko __ 3. Fish oil __ 4. Vitamin D __ 1-True 2-False 3-True 4-True 1-True 2-True 3-True 4-False 1-False 2-False 3-True 4-True 1-False 2-False 3-True 4-False

Ans: 1-True 2-True 3-True 4-False

Identify which of the following patients should undergo testing for tuberculosis (Yes/No). 1. A 22-year-old male with HIV. __ 2. A healthy 18-year-old woman who lives with her father who was recently diagnosed with active tuberculosis infection. __ 3. 45-year-old man who was recently released from prison after 2 years of incarceration. __ 4. 32-year-old woman who recently emigrated from Southeast Asia. __ 1-No 2-Yes 3-Yes 4-Yes 1-Yes 2-Yes 3-Yes 4-Yes 1-Yes 2-No 3-No 4-Yes 1-No 2-No 3-No 4-No

Ans: 1-Yes 2-Yes 3-Yes 4-Yes

Identify which of the following patients should undergo testing for tuberculosis. (Yes/No) 1. A 22-year-old male with HIV __ 2. A healthy 18-year-old woman who lives with her father who was recently diagnosed with active tuberculosis infection __ 3. A 45-year-old man who was recently released from prison after 2 years of incarceration __ 4. A 32-year-old woman who recently immigrated from Southeast Asia __ 1-Yes 2-No 3-Yes 4-No 1-Yes 2-No 3-No 4-No 1-No 2-No 3-Yes 4-No 1-Yes 2-Yes 3-Yes 4-Yes

Ans: 1-Yes 2-Yes 3-Yes 4-Yes

True or False? 1. Immune globulin is a concentrated solution of antibodies derived from pooled donated blood. __ 2. Tdap (tetanus, diphtheria, acellular pertussis [Adacel®, Boostrix®]) vaccine should replace a single dose of Td (tetanus, diphtheria) vaccine for most adults who have not previously received a dose of Tdap in order to provide additional protection against pertussis. __ 3. Previously unvaccinated adults age 19 through 59 years with diabetes mellitus type 1 or type 2 should be vaccinated against hepatitis B as soon as possible after the diabetes diagnosis is made. __ 1. True 2. True 3. False 1. True 2. True 3. True 1. False 2. True 3. False 1. True 2. False 3. False

Ans: 1. True 2. True 3. True

You see a 47-year-old man in urgent care who has hypertension, dyslipidemia, and depression. He states, "I am on a big list of medications but I am not sure of all the names." He also has a history of penicillin allergy with a hive-form reaction. He has not taken a systemic antimicrobial in more than a year and denies recent hospitalization. When developing a treatment plan for acute bacterial rhinosinusitis, you consider prescribing the following course of an oral antimicrobial A 5-day course of clarithromycin. A 7-day course of doxycycline. A 10-day course of amoxicillin- clavulanate. A 7-day course of moxifloxacin.

Ans: A 7-day course of doxycycline.

Macular degeneration would most likely be found in which of the following patients? A 72-year-old woman who smoked cigarettes up until 10 years ago A 54-year-old man with a family history of glaucoma A 64-year-old man with well-controlled hypertension A 32-year-old woman recently diagnosed with type 2 diabetes mellitus

Ans: A 72-year-old woman who smoked cigarettes up until 10 years ago

A 30-year-old woman of Asian ancestry presents for a routine health visit. She is in good health and reports she is concerned about her personal breast and ovarian cancer risk and asks about BRCA gene mutation testing. You advise the following: BRCA gene mutation testing should be offered to all women who request this evaluation. A standardized screening questionnaire for BRCA gene mutation such as FHS-7 should be administered today to determine if BRCA gene mutation testing is warranted. She is not a candidate for this test due to her ethnicity. A referral to genetic counseling for guidance on BRCA gene mutation testing should be ordered.

Ans: A standardized screening questionnaire for BRCA gene mutation such as FHS-7 should be administered today to determine if BRCA gene mutation testing is warranted.

When evaluating Ana, you note that approximately 20% total body surface area is affected. You consider treatment with: Based on the Evaluation Below: Ana is a 22-year-old well woman who presents with a four-day history of an intense itch with skin lesions on hands and arms as well as her right cheek. She has used an over-the-counter hydrocortisone cream on the affected area with little effect, and denies any other symptoms or previous history of similar rash. Ana denies recent travel and exposure to new creams, soaps, or medications. She works as a landscaper. Topical application of a medium-potency corticosteroid cream An oral antihistamine A systemic corticosteroid A topical antihistamine

Ans: A systemic corticosteroid

When considering influenza vaccination for an adult who claims to experience a hive-form reaction when exposed to eggs without a history of angioedema or respiratory distress, which of the following is most appropriate? Administer any vaccine appropriate for his age and health status Only offer the recombinant influenza vaccine (RIV3) Referral for vaccination at an allergy practice. Do not administer vaccine as it is contraindicated for this patient

Ans: Administer any vaccine appropriate for his age and health status

Hank is a 58-year-old man who presents with a chief complaint of bilateral itchy eyes occurring intermittently throughout the year. Exam reveals 20/30 vision OD (right eye), OS (left eye), OU (both eyes) with corrective lenses bilateral hyperemic bulbar and palpebral conjunctiva, and a small amount of rope-like pale yellow discharge. These findings are most consistent with: Bacterial conjunctivitis. Blepharoconjunctivitis. Allergic conjunctivitis. Dry-eye syndrome.

Ans: Allergic conjunctivitis.

nitial treatment for a 23-year-old man with ABRS with no recent antimicrobial therapy (>3 months) and no drug allergy is: Ampicillin. Amoxicillin-clavulanate (Augmentin®). Ciprofloxacin (Cipro®). Levofloxacin (Levaquin®).

Ans: Amoxicillin-clavulanate (Augmentin®).

Therapy to help avoid nasal symptoms in a 32-year-old man with allergic rhinitis should include: An oral first-generation antihistamine. An intranasal corticosteroid. An oral decongestant. An intranasal anticholinergic.

Ans: An intranasal corticosteroid.

A 37-year-old woman with ulcerative colitis with poor symptom control without rectal bleeding presents with the following hemogram: -Hg=9.9 g/dL (12-14 g/dL) (99 g/L [120-140 g/dL]) -MCHC=33 g/dL (31-37 g/dL) (330 g/L [310-370 g/L]) -MCV=86 fL (80-96 fL) -RDW=12% (11%-15%) (0.12 proportion [0.11-0.15 proportion]) These findings are most consistent with: Iron deficiency. Beta thalassemia minor. Anemia of chronic disease. Pernicious anemia.

Ans: Anemia of chronic disease.

A 65-year-old woman with rheumatoid arthritis who is on optimized therapy and continues to have significant symptoms presents with the following hemogram. -Hb=10.1 g/dL (12-14 g/dL) 101 g/L (120-140 g/L) -Hct=32% (36-42%) 0.32 proportion (0.36-0.42 proportion) -RBC=3.2 million/mm3 (4.2-5.4 million/mm3) - MCV=82 fL (80-96 fL) -RDW=12.8% (11-15%) 0.128 proportion (0.11-0.15 proportion) -Reticulocytes=0.7% (1-2%) 0.007 proportion (0.01-0.02 proportion) These findings are most consistent with: Iron deficiency anemia. Folate deficiency anemia. Anemia of chronic disease. Alpha thalassemia minor.

Ans: Anemia of chronic disease.

A 58-year-old woman presents for an initial examination in order to become a patient in your primary care practice. She is a nonsmoker, drinks 1 to 2, 5 oz (0.15 L) glasses of wine per week, and works as an administrative assistant in a law office. She is without complaint and reports that she is generally in good health. Physical exam reveals BMI=34 kg/m2 and BP=144/98 mm Hg bilaterally. The rest of her examination is unremarkable. Prescribe a low-dose thiazide diuretic. Arrange for additional blood pressure measurements within the next four weeks. Order a serum creatinine, urea nitrogen and urinalysis. Advise restricting sodium intake and limiting alcohol intake to no more than 1 glass of wine per week.

Ans: Arrange for additional blood pressure measurements within the next four weeks.

Mrs. Lange is a 79-year-old woman with a >20-year history of well-controlled hypertension and dyslipidemia, currently taking an ACE inhibitor, low-dose thiazide diuretic, and a statin. She presents today with a chief complaint of a 6-month history of progressive symptoms, including fatigue, difficulty initiating and maintaining sleep, increased difficulty with raising her arms above her head, and a sensation of "my heart not beating right, sometimes I feel like it's going to hop right out of my chest." She denies shortness of breath, chest pain, cough, or difficulty breathing when supine and admits to "losing weight without even trying." Cardiac examination reveals an irregularly irregular cardiac rhythm, without S3, S4, or murmur and no neck vein distention.The result of Mrs. Lange's electrocardiogram is consistent with: Sinus tachycardia Multifocal atrial tachycardia First-degree heart block Atrial fibrillation

Ans: Atrial Fibrillation

You see Maria, a a 32-year-old well woman of Mediterranean ancestry. Hemogram results are as follows: -Hb=10.6 g/dL (12-14 g/dL) 106 g/L (120-140 g/L) -Hct=32% (36-42%) 0.32 proportion (0.36-0.42 proportion) -RBC=5.2 million/mm3 (3.2-4.3 million/mm3) -MCV=71 fL (80-96 fL) -MCHC=25.2 g/dL (31-37 g/dL) 252 g/L [310-370 g/L] -RDW=12% (<15%) 0.12 proportion (<0.15 proportion) These findings are most consistent with: Iron deficiency anemia. Cooley's anemia. Beta thalassemia minor. Acute blood loss.

Ans: Beta thalassemia minor.

The NP demonstrates fulfillment of the advanced practice nursing leadership role by participating in which of the following activities? Teaching a 56-year-old man with newly-diagnosed type 2 diabetes mellitus about the importance of self-glucose monitoring Volunteering to teach a class on contraceptive methods to a group of teen mothers Discussing barriers to achieving blood pressure control with a 65-year-old woman with hypertension who "does not want to take any medicine" Collaborating with the regional public health department on an initiative to combat obesity through a community-based exercise program

Ans: Collaborating with the regional public health department on an initiative to combat obesity through a community-based exercise program

A 19-year-old woman returns to receive her third dose of the HPV vaccine series. Her first two doses were with the quadrivalent vaccine, but the practice currently only stocks the 9-valent HPV vaccine. The NP considers the best course of action is to: Complete her vaccination series with the 9-valent vaccine. Defer completing the vaccination series until the quadrivalent vaccine is available in the practice. Refer her to a practice where the quadrivalent vaccine is currently available. Restart the series with the 9-valent vaccine and schedule her next two doses within 6 months.

Ans: Complete her vaccination series with the 9-valent vaccine.

Sandra's hearing loss is best described as: Based on the below scenario: Sandra is a 45-year-old well woman diagnosed with a left-sided unilateral acute otitis media 10 days ago and treated with an antimicrobial. She is seen today with a report of resolution of ear pain, but with persistent sensation of ear fullness and diminished ability to discriminate speech in the affected ear. Sensorineural. Conductive. Auditory processing. Mixed origin.

Ans: Conductive.

A first-line treatment for uncomplicated skin infection due to S. aureus in a patient with no identifiable risk factors for MRSA is: Ciprofloxacin (Cipro®) Dicloxacillin Linezolid (Zyvox®) Daptomycin (Cubicin®)

Ans: Dicloxacillin

You are asked to speak to a group of healthcare executives on the NP role. Which of the following provides the strongest support for NP practice? Number of academic credits earned during the NP program Evidence of NP practice outcomes The quantity of clinical hours and type of clinical rotations in the NP program The professional experience of the person prior to entering the NP program

Ans: Evidence of NP practice outcomes

Of the following, which should be performed first in assessing a 48-year-old woman who has a 25-year history of moderate persistent asthma and who now presents with an acute asthma flare? Oxygen saturation Arterial blood gas FEV1 (forced expiratory volume at 1 second) Chest x-ray

Ans: FEV (force expiratory volume)

Across North America, brown recluse spider bites are the most common reason for new-onset ulcerating skin lesion. True False

Ans: False

A 78-year-old woman presents with fatigue, spoon-shaped nails and the following laboratory results. -Hb=9 g/dL (90 g/L) -Hct=28.1% (0.281 proportion) -RBC=2.4 million/mm3 -MCV=70 fL -MCHC=24.2 g/dL (242 g/L) -RDW=19% (0.19 proportion) A critical causative diagnosis to consider as origin of her anemia is: Gastrointestinal blood loss. Micronutrient malabsorption. Chronic ileitis. Folic acid deficiency.

Ans: Gastrointestinal blood loss.

Which of the following influenza vaccines is most appropriate for a 67-year-old man with no reported allergic reactions? Quadrivalent live attenuated influenza vaccine (LAIV) High-dose trivalent IIV Intradermal quadrivalent IIV Cell cultured-based quadrivalent IIV

Ans: High-dose trivalent IIV

In a 51-year-old woman at average risk of breast cancer, choose the prevention: Primary prevention Secondary prevention Tertiary prevention 1. Counseling about the hazard of tobacco use. __ 2. Obtaining breast cancer screening via mammography. __ 1-Secondary prevention 2-Primary prevention 1-Tertiary prevention 2-Primary prevention 1-Primary prevention 2-Secondary prevention 1-Tertiary prevention 2-Secondary prevention

Ans: In a 51-year-old woman at average risk of breast cancer, choose the prevention:

Which of the following is the most appropriate next step in Matthew's care? Based on the below scenario: Matthew is a 29-year-old man who presents with a 6-hour history of sudden onset of inability to raise his eyebrow or smile on the right side. He also reports decreased lacrimation in the right eye and difficulty closing the right eyelid. The rest of his health history and physical examination is otherwise unremarkable. Emergent referral for neuroimaging Initiating a course of oral corticosteroids Prescribing a short course of high-dose antiviral therapy Referral to a neurology specialist within the next 24‒48 hours Feedback

Ans: Initiating a course of oral corticosteroids

A 67-year-old man taking NSAIDs daily for back pain and reporting fatigue presents with the following hemogram: -Hg=8.4 g/dL (12-14 g/dL) (84 g/L [120-140 g/dL]) -MCHC=26 g/dL (31-37 g/dL) (260 g/L [310-370 g/L]) -MCV=69 fL (80-96 fL) -RDW=19% (11%-15%) (0.19 proportion [0.11-0.15 proportion]) These findings are most consistent with: Iron deficiency anemia. Beta thalassemia minor. Vitamin B12 deficiency. Drug-induced macrocytosis.

Ans: Iron deficiency anemia.

Katie asks if she needs "any tests to see what causes my headaches." You respond that she should: Based on the Scenario Below: Katie is a 33-year-old woman who reports a 10-year history of unilateral, pulsing headache that lasts about 6-10 hours, occurring 3?4 times per month. The headache is typically preceded by a gradual onset of paresthesia affecting the ipsilateral face and arm, which lasts about 20 minutes. She reports severe photophobia and phonophobia as well as left-sided cephalgia during the 6-10 h headache duration. She states the headaches appear randomly. As a result of her headaches, Katie typically needs to either call in sick or leave work early at least once a month due to headache. Katie has used OTC medications with partial relief of pain but continued photo-and phonophobia. She is currently headache-free and neurological exam is within normal limits. Have head CT conducted Keep a headache diary for the next month Be promptly referred to a neurologist Have head MRI conducted

Ans: Keep a headache diary for the next month

You see a 20-year-old college wrestler with purulent cellulitis. He reports that a couple of his teammates were recently diagnosed with MRSA skin infections. Initial treatment options would include all of the following except: Trimethoprim-sulfamethoxazole (Bactrim®) Clindamycin (Cleocin®) Doxycycline (Doryx®) Levofloxacin (Levaquin®)

Ans: Levofloxacin (Levaquin®)

Risk factors for ABRS caused by drug-resistant S. pneumoniae (DRSP) include all of the following except: Recent prior antimicrobial use. Exposure to a child in large-group daycare. Living in a rural setting. Hospitalization within the past 5 days.

Ans: Living in a rural setting.

A 51-year-old woman of European ancestry who has had no primary care for more than 10 years presents to your practice. She has a 40 pack-year cigarette smoking history, currently smoking 1 PPD and reports drinking about 3-4, 12 oz (0.35 L) beers per month. Her history is otherwise unremarkable as is her physical examination. You order the following screening test: Hemoglobin electrophoresis. Mammography. Chest x-ray. Fasting serum triglycerides.

Ans: Mammography.

Katie is a 33-year-old woman who reports a 10-year history of unilateral, pulsing headache that lasts about 6-10 hours, occurring 3-4 times per month. The headache is typically preceded by a gradual onset of paresthesia affecting the ipsilateral face and arm, which lasts about 20 minutes. She reports severe photophobia and phonophobia as well as left-sided cephalgia during the 6-10 h headache duration. She states the headaches appear randomly. As a result of her headaches, Katie typically needs to either call in sick or leave work early at least once a month due to headache. Katie has used OTC medications with partial relief of pain but continued photo- and phonophobia. She is currently headache-free and neurological exam is within normal limits. Katie's presentation is consistent with: Migraine with aura Tension-type headache Cluster headache Intracranial lesion

Ans: Migraine with aura

You are setting up a healthcare clinic for migrant farm workers. Which of the following sites is most desirable? Community hall of a local church Local hospital ambulatory care center Mobile van Space in a shopping center

Ans: Mobile van

A first-generation antihistamine can help alleviate all of the following symptoms of allergic rhinitis except: Sneezing. Nasal congestion Rhinorrhea. Itchy, watery eyes.

Ans: Nasal congestion

The most appropriate treatment option for Hank is the use of: Based on the below scenario: Hank is a 58-year-old man who presents with a chief complaint of bilateral itchy eyes occurring intermittently throughout the year. Exam reveals 20/30 vision OD (right eye), OS (left eye), OU (both eyes) with corrective lenses bilateral hyperemic bulbar and palpebral conjunctiva, and a small amount of rope-like pale yellow discharge. Ocular antimicrobial. Lubricating eye solution Ocular antihistamine. Systemic decongestant.

Ans: Ocular antihistamine.

While counseling a 32-year-old man in good health about influenza vaccination, he reports that he occasionally experiences hives when consuming eggs (e.g., scrambled eggs), but has no reaction when consuming foods that contain eggs (e.g., cake). The NP considers the best course of action is to: Note the allergy in his records so that immunization will not be offered in the future. Offer vaccination with trivalent recombinant influenza vaccine (RIV3). Offer vaccination with live attenuated influenza vaccine (LAIV). Refer him to a specialist for allergy testing prior to immunization with high-dose trivalent inactivated influenza vaccine (IIV3).

Ans: Offer vaccination with trivalent recombinant influenza vaccine (RIV3).

Appropriate pharmacologic management options for allergic rhinitis in a 29-year-old woman who is a home daycare provider includes all of the following except: Short-term use of a decongestant nasal spray. Oral chlorpheniramine. Flunisolide nasal spray. Oral loratadine.

Ans: Oral chlorpheniramine.

A 60-year-old otherwise well man presents with a dermatologic condition. Which of the following conditions requires biopsy to confirm the diagnosis? Scaling flesh-colored lesions in a cluster, ranging in size from 3?10 mm on the dorsal aspect of the hand, present for a number of months, without patient complaint Well-demarcated round-to-oval erythematous coin-shaped plaques approximately 10 mm in diameter on the anterior aspects of the lower legs described as being intermittently itchy that has been present for a number of months. Painless ulcerated lesion approximately 1.5 cm over the sternum that has been present for a number of weeks. Oval plaque that is approximately 5 cm in diameter with a central wrinkled salmon-colored area and a dark red peripheral zone on the anterior trunk that has been present for 5 days without patient complaint.

Ans: Painless ulcerated lesion approximately 1.5 cm over the sternum that has been present for a number of weeks.

During a clinical encounter with a 78-year-old man who has presbycusis, the NP considers that communication will be enhanced by all of the following except: Maintaining eye contact with the patient. Ensuring the patient can see the NP's face clearly. Playing soft music in the background. Providing adequate illumination in the exam room.

Ans: Playing soft music in the background.

According to the Stages of Change Theory, he is most likely in which of the following stages? Precontemplation Contemplation Preparation Minimization

Ans: Precontemplation

One of the most common causes of asymptomatic hypercalcemia in an otherwise well adult is: Excessive use of calcium supplements. Primary hyperparathyroidism. Renal insufficiency. Intestinal malabsorption.

Ans: Primary hyperparathyroidism.

A 28-year-old woman who works in food service presents with a chief complaint of an on-the-job injury caused when her right forearm was accidentally exposed to steam. Approximately 2% body surface area is involved. You assess the injury as a partial thickness (second degree) burn and describe the skin lesion as appearing: Reddened easily blanched with gentle pressure Red, moist with peeling borders and scattered bulla Thickened, hypopigmented tissue Vesicular with hyperpigmentation

Ans: Red, moist with peeling borders and scattered bulla

Examination of a 56-year-old woman identifies a palpable thyroid mass of relatively fixed position. TSH level is within normal limits. Ultrasound reveals a solid mass of approximately 5 cm in size. Which of the following is the most appropriate next course of action? Watch and wait with rescan in 6 months Initiate levothyroxine therapy Refer for fine-needle aspiration biopsy Refer for radioiodine ablation

Ans: Refer for fine-needle aspiration biopsy

The next step in Edgar's care should include: Based on the below scenario: Edgar is a 75-year-old man with a 60 pack-year history of cigarette smoking and COPD who presents with a chief complaint of a "sore" on the base of his tongue. This lesion has been present for a number of months, remaining relatively stable in size and is not painful. Physical examination reveals a painless ulcerated lesion with indurated margin and is accompanied by a firm, nontender submandibular node. His current medications include inhaled corticosteroids with a long-acting beta2-agonist. An oral antifungal. Referral for lesion biopsy. Use of topical anesthetic paste. Serologic testing to confirm the diagnosis.

Ans: Referral for lesion biopsy.

Choose the two most important tests to help support Mrs. Lange's diagnosis. Based on the Physical Examination Below: Mrs. Lange is a 79-year-old woman with a >20-year history of well-controlled hypertension and dyslipidemia, currently taking an ACE inhibitor, low-dose thiazide diuretic, and a statin. She presents today with a chief complaint of a 6-month history of progressive symptoms, including fatigue, difficulty initiating and maintaining sleep, increased difficulty with raising her arms above her head, and a sensation of "my heart not beating right, sometimes I feel like it's going to hop right out of my chest." She denies shortness of breath, chest pain, cough, or difficulty breathing when supine and admits to "losing weight without even trying." Cardiac examination reveals an irregularly irregular cardiac rhythm, without S3, S4, or murmur and no neck vein distention. Serum creatine kinase Serum electrolytes Serum thyroid stimulating hormone B-type natriuretic peptide Free thyroxine (FT4)

Ans: Serum thyroid stimulating hormone Free thyroxine (FT4)

You see a 55-year-old woman who presents for a health maintenance visit. She has longstanding myopia and newer-onset presbyopia, both corrected with eyeglasses. She is normotensive and without ocular complaint. You anticipate the following on today's eye exam: Retinal arteries wider than veins. Equal, sluggish pupillary response. Sharp disc margins. Lid ectropion.

Ans: Sharp disc margins.

Edgar is a 75-year-old man with a 60 pack-year history of cigarette smoking and COPD who presents with a chief complaint of a "sore" on the base of his tongue. This lesion has been present for a number of months, remaining relatively stable in size and is not painful. Physical examination reveals a painless ulcerated lesion with indurated margin and is accompanied by a firm, nontender submandibular node. His current medications include inhaled corticosteroids with a long-acting beta2-agonist. This clinical scenario is most consistent with: Syphilitic chancre. Aphthous stomatitis. Squamous cell carcinoma. Oral candidiasis.

Ans: Squamous cell carcinoma.

Which of the following represents the optimal advice to a patient who is taking oral iron therapy to maximize the medication's effectiveness? Take your medication with an antacid. Take your medication on an empty stomach. Take your medication after the largest meal of the day. Take your medication with a large glass of milk.

Ans: Take your medication on an empty stomach.

Mr. Jacobs is a 65-year-old man with COPD and a 60 pack-year history who is currently smoking 1.5 packs of cigarettes per day. He is reading a pamphlet in your office about smoking cessation.You ask him if he has any questions and he states, "I don't plan to quit smoking at my age. I am too old to quit now. Why bother?" The NP's most appropriate response is: Since you have COPD, you really should quit smoking. You know your lungs will get more damaged if you continue to smoke. Tell me what you mean by, "I am too old to quit." I can provide medication that will help you to quit smoking.

Ans: Tell me what you mean by, "I am too old to quit."

For Sandra, which of the following represents the best advice for this point? Based on the below scenario: Sandra is a 45-year-old well woman diagnosed with a left-sided unilateral acute otitis media 10 days ago and treated with an antimicrobial. She is seen today with a report of resolution of ear pain, but with persistent sensation of ear fullness and diminished ability to discriminate speech in the affected ear. She should have a second course of antimicrobial therapy. A short course of an oral corticosteroid should be prescribed. The sensation of ear fullness is an anticipated finding. She should be seen by an otolaryngology specialist.

Ans: The sensation of ear fullness is an anticipated finding.

The remainder of her physical examination reveals flat affect, fine tremor, 3-4+ DTR response, mild proximal muscle weakness, symmetric thyroid enlargement without tenderness or mass and a 10-lb (4.5-kg) weight loss since her last visit 8 months ago. The remainder of Mrs. Lange's examination is at her baseline. Which of the following is the most likely diagnosis? Based on the Physical Examination Below: Mrs. Lange is a 79-year-old woman with a >20-year history of well-controlled hypertension and dyslipidemia, currently taking an ACE inhibitor, low-dose thiazide diuretic, and a statin. She presents today with a chief complaint of a 6-month history of progressive symptoms, including fatigue, difficulty initiating and maintaining sleep, increased difficulty with raising her arms above her head, and a sensation of "my heart not beating right, sometimes I feel like it's going to hop right out of my chest." She denies shortness of breath, chest pain, cough, or difficulty breathing when supine and admits to "losing weight without even trying." Cardiac examination reveals an irregularly irregular cardiac rhythm, without S3, S4, or murmur and no neck vein distention. Thyrotoxicosis Statin-induced myopathy Heart failure Hypothyroidism

Ans: Thyrotoxicosis

When evaluating a patient for a potential thyroid disorder, which of the following is the least informative test to be used during evaluation? Free T4 Total T4 Free T3 TSH

Ans: Total T4

rue or false? (T/F) The U.S. FDA advises that the adverse effects associated with fluoroquinolones generally outweigh the benefits for patients with acute sinusitis, acute bronchitis, and uncomplicated urinary tract infections who have other treatment options. True False

Ans: True

A 65-year-old Native American man presents for a "physical." He feels well, denies tobacco or alcohol use, and has not seen a healthcare provider in more than 10 years. The patient states, "I am a really healthy person. I would not come in except my wife and daughter told me I should have a checkup." As part of today's visit, he should be screened for: Pancreatic cancer. Hemolytic anemia. Hepatic sclerosis. Visual defect.

Ans: Visual defect.

The most likely cause of this anemia is: Based on the below scenario: A 68-year-old woman presents with a 6-month history of increasingly severe peripheral numbness and oral irritation. Hemogram results are as follows. -Hb=6.2 g/dL (62 g/L) -Hct=20% (0.2 proportion) -RBC=2.1 million/mm3 -MCV=132 fL -MCHC=32.4 g/dL (324 g/L) -RDW=19% (0.19 proportion) Physical exam reveals pale conjunctiva, a grade 2/6 systolic ejection murmur over the precordium without radiation, and a smooth, red tongue. Vitamin B12 deficiency. Iron deficiency. Hemolysis. Chronic disease.

Ans: Vitamin B12 deficiency.

You advise a 28-year-old woman who follows a vegan diet to supplement with: Vitamin A. Iron. Vitamin B12. Folic acid.

Ans: Vitamin B12.

Sandra is a 45-year-old well woman diagnosed with a left-sided unilateral acute otitis media 10 days ago and treated with an antimicrobial. She is seen today with a report of resolution of ear pain, but with persistent sensation of ear fullness and diminished ability to discriminate speech in the affected ear. Anticipated findings on today's physical examination include: Erythema of the ear canal. Weber test lateralizing to the affected ear Discomfort on tragus pull. Anterior cervical lymphadenopathy on the affected side. Feedback

Ans: Weber test lateralizing to the affected ear

You examine a 62-year-old woman with a 20-year history of hypertension. She reports intermittent use of antihypertensive medications, stating, "I feel better without the medications." Today she presents for a "check-up."On physical examination, you note the following: PMI with a downward and lateral shift as well as a Gr II/VI holosystolic murmur with radiation to the axilla. This patient's health history is most likely to include a report of: Syncopal episodes. Episodes of chest pain at rest. Dyspnea with exertion. Vertigo.

Ans: dyspnea on exertion

You see a 38-year-old woman with hypothyroidism who is currently taking levothyroxine 75 mcg/d with excellent adherence, stating, "I take the medicine every morning on an empty stomach with a big glass of water." She is feeling well. Results of today's laboratory testing includes a TSH=4.5 mIU/mL. The next best step in her care is to: Continue on the same levothyroxine dose and obtain a repeat TSH in 1 year Decrease the levothyroxine dose by 25 mcg/d and repeat a TSH in 1 month Increase the levothyroxine dose by 25 mcg/d and repeat a TSH in 2 months Provide counseling to take the medication with breakfast

Ans: ncrease the levothyroxine dose by 25 mcg/d and repeat a TSH in 2 months

Mrs. Garcia's most likely diagnosis is: Systemic lupus erythematosus. Rheumatoid arthritis. Osteoarthritis. Reactive arthritis.

Ans: Osteoarthritis.

Primary care of a person with alpha thalassemia minor should include: Routine iron supplementation. Offering genetic counseling prior to pregnancy. Periodic evaluation of hemoglobin electrophoresis. Folate supplementation.

Ans: Offering genetic counseling prior to pregnancy.

A 66-year-old man presents for a "check-up." He has no complaints and admits he's been smoking 2 packs a day for 40 years. Which additional finding would be consistent with a diagnosis of chronic obstructive pulmonary disease? Markedly increased chest AP diameter Pleuritic chest pain FEV1:FVC ratio <0.70 post SABA use FEV1 improving by 50% with use of SAMA

Ans: FEV1:FVC ratio <0.70 post SABA use

Medicaid benefits are paid to the enrollee, then the enrollee is responsible for paying the healthcare bill. True False

Ans: False

The Federal Drug Enforcement Administration (DEA) provides registration numbers for prescription of controlled substances to which of the following groups? NPs with federal prescription authority Physicians, veterinarians, dentists, and NPs who practice in oncology or palliative care NPs with state-determined controlled substance prescriptive authority Nationally-certified APRNs with state licensure

Ans: NPs with state-determined controlled substance prescriptive authority

Joseph is a 28-year-old man who presents to your practice with a 2 h history of a superficial laceration on his forearm that is approximately 5 cm (2") that happened when he was replacing a windowpane and was cut by the glass. Physical examination is consistent with history.His record notes a Tdap vaccine given approximately 2 years ago and documentation of a primary tetanus series. When considering his needs for tetanus immunization today, the NP appreciates that today Joseph should receive: A Td (tetanus, diphtheria) booster. A Tdap (tetanus, diphtheria, acellular pertussis) booster. A dose of TIG (tetanus immune globulin). No tetanus vaccine.

Ans: No tetanus vaccine.

One of the first-line therapy options for moderate acne is: Topical tretinoin. Topical corticosteroids. Oral isotretinoin. Systemic antibiotics.

Ans: Topical tretinoin.

considerations in caring for a 68-year-old man with a BMI=38 kg/m2 who is otherwise well and presents with genital candidiasis includes which of two most helpful measures? Advice on the use of antibacterial soap to the region. Obtain an in-office blood glucose. Prescribing topical miconazole. Order a medium-potency topical corticosteroid to the affected region to help with symptom control.

Ans: Obtain an in-office blood glucose. Prescribing topical miconazole.

Risk factors for pancreatic cancer include all of the following except: Hypertension History of chronic pancreatitis Tobacco use Diabetes mellitus

Ans: Hypertension

The difference between a macule and a patch is: Location. Disease state. Size. Configuration.

Ans: Size.

Dental care for Medicaid enrollees under the age of 21 years is a basic requirement of the plan. True False

Ans: True

A 48-year-old man is recently diagnosed with type 2 diabetes mellitus. All of the following vaccinations are indicated except: Pneumococcal. Hepatitis B. Herpes zoster (shingles). Influenza.

Herpes zoster (shingles).

For the patient described above, the most appropriate treatment would be: Based on the below scenario: You see a 24-year-old male with small flesh-colored verruca-form lesions on the shaft and tip of the penis. He reports some itching and discomfort in the genital area, but no voiding symptoms. IM ceftriaxone (Rocephin®) Topical imiquimod (Aldara®) Oral doxycycline (Doryx®) Topical acyclovir (Zovirax®)

ans: topical imiquimod cream

The process of absorption, distribution, metabolism, and elimination of a drug is known as: Pharmacokinetics. Therapeutic transformation. Pharmacodynamics. Drug interactions study.

ans: Pharmacokinetics.

Thyroid stimulating hormone (TSH) levels are suppressed in all of the following except: Graves' disease. Autonomous toxic nodule. Acute viral thyroiditis. Subacute hypothyroidism.

ans: Subacute hypothyroidism.

Which of the following is consistent with the diagnosis of all stages of chronic obstructive pulmonary disease? FEV1: FVC ratio <0.70 post-bronchodilator Dyspnea on exertion Hypoxemia Orthopnea

Ans: FEV1: FVC ratio <0.70 post-bronchodilator

A 47-year-old man of Northern European ancestry presents for evaluation of erythematous, papular lesions on his forehead and chin. Which of the following differential diagnoses is most likely? Acne rosacea Tinea facialis Hidradenitis suppurativa Atopic dermatitis

Ans: Acne rosacea

The reduction in free androgens noted in the woman taking combined oral contraceptives can yield an improvement in: Menstrual irregularity. Acne vulgaris. Breast tenderness. Rheumatoid arthritis symptoms.

Ans: Acne vulgaris

Which of the following is a typical physical finding in a patient with carpal tunnel syndrome? Decreased vibratory sense of distal phalanges Hyperactive radial reflex Reproduction of symptoms with forced flexion of the wrists Positive Allen's test

Ans: Reproduction of symptoms with forced flexion of the wrists

n a woman taking a combined oral contraceptive, the reduction in free androgens can yield an improvement in: Cycle control. Acne vulgaris. Rheumatoid arthritis. Breast tenderness.

Ans: Acne vulgaris.

A father brings in his 20-month-old daughter who presents with a diffuse rosy-pink macular rash and fever of 102.3ᵒF (39.1ᵒC). He reports that he first noticed the rash this morning but that she has had a fever for the past 3-4 days. Today, the child is without fever. . The most likely diagnosis is: Roseola Rubeola. Scarlet fever. Rubella.

Ans: Roseola - study childhood rash and fevers condition. Scarlet fever: sand-paper like rash with exudative pharyngitis fever, headache, tender localized anterior cervical lymphadenopathy. Rash usually erupts on day 2 of pharyngitis, and often peels a few days later. Treatment: caused by Strep, oral Amoxicillin as 1st line, Penicillin Po or IM, oral Macrolide (Azithromycin, Clarithromycin, Erythromycin) only if pts has PCN allergy. Roseola (Agent Herpes virus) HHV 6: dicrete pink-rose macular rash or maculopapillar rash lasting hours to 3-7 days often with high fevers. 90% cases found in children < 2 yrs. Treatment: supportive therapy Rubella (virus): mild symptoms sore throat, malaise, nasal discharge, diffuse maculopapillar rash lasting about 3 days; posterior cervical and postauricular lymphadenopathy begining 5-10 days. Prior to rash eruption, 10% most common in women c/o myalgia. Incubation period: 10-14 days, most infectious 1 -2 wk prior to onset of rash rash, generally self-limiting, greatest risks to unborn child especially during 1st trimester (infant can expose to congenital rubella syndrome). It is a reportable disease to HHS department, laboratory confirmation by presence of serum Rubella IgM. MMR virus- usually acute on presentation with fever, nasal discharge, cough, generalized lymphadenopathy, conjuctivitis (copius clear discharge), Koplik spots appears 2 days prior with blue rings held within red spots on oral mucosa, pharyngitis w/o exudative, maculopapular rash onset 3-4 days after onset of symptoms, may coalesce to generalized erythema. Complications: CNS and respiratory tract are common, permanent neurological impairment or death possible, is a reportable disease to HHS department, Vaccine-preventable disease, laboratory confirmatory by presence of serum rubeola IgM. Infectious Mononucleosis: caused by Epstein-Barr virus- Maculopapular rash or petechiae rare, fever, purple white pharyngeal exudate, malaise, marked diffuse lymphadenopathy, hepatic and splenic tenderness, splenic enlargement. Dx: Mono spot, lymphocytosis, atypical lymphocytes presence. Incubation period: 20-50 days. >90% will develop rash if given Amoxicillin or Ampixicillin during the illness. Potential for resp. distress due to enlarged tonsils and lymphoid tissues impinged on the airway, corticosteroids maybe helpful, splenomegaly often noted between day 6-21 after onset of illness. Avoid contact sports > 1 mo due to risk of splenic rupture.

A 54-year-old man with COPD is experiencing an exacerbation characterized by increased cough and shortness of breath for the past 48 hours, as well as mild sore throat, clear nasal discharge and body aches. He is currently being managed with a long-acting beta2-agonist and long-acting muscarinic antagonist . An appropriate treatment for this patient would include: Oral amoxicillin/clavulanate. Oral levofloxacin. Oral corticosteroid. Theophylline.

Ans: Oral corticosteroid.

A 3-year-old girl presents with gastroenteritis and who has vomited 4 times in the past 12 hours. The child has signs of mild-to-moderate dehydration. The most appropriate treatment for this child is: Parenteral dexamethasone (Decadron®). Oral metoclopramide (Reglan®). Oral disintegrating tablets of ondansetron (Zofran®). Rectally-administered trimethobenzamide (Tigan®).

Ans: Oral disintegrating tablets of ondansetron (Zofran®).

Likely causative organisms in community-acquired pneumonia include: S. pneumoniae and select respiratory viruses H. influenzae and S. aureus M. catarrhalis and atypical pathogens K. pneumoniae and Legionella species

Ans: S. pneumoniae and select respiratory viruses

A 45-year-old man complains of bilateral, intermittent itchy eyes, often accompanied by rope-like discharge and without vision change or eye pain. The nurse practitioner expects that the history will include a patient report of: Trauma to the eye. Sexually transmitted infection. Seasonal allergies Decreased visual acuity.

Ans: Seasonal allergies

A new 27-year-old female patient presents for a health maintenance examination as required by her employer. She tells you that she has had a heart murmur "all of her life" and has been told that it is an "innocent" murmur. Anticipated findings include: A harsh late systolic snap. A soft systolic murmur that disappears when in a standing position. A diastolic murmur best heard at the base of the heart. A high-pitched murmur that is synchronous with the carotid pulse.

A soft systolic murmur that disappears when in a standing position.

You are evaluating a 19-year-old man with acne vulgaris and determine he has approximately 20 closed comedones. The most cost-effective first-line treatment for this patient is: Oral doxycycline. Benzoyl peroxide lotion with salicylic acid facial wash. Tretinoin gel. Oral isotretinoin.

Ans: Benzoyl peroxide lotion with salicylic acid facial wash.

All of the following are risk factors for child mistreatment except: Child 4-5 years of age. Child with special needs that can increase caregiver burden. Parental history of child maltreatment. Residing in a community with significant baseline violence.

Ans: Child 4-5 years of age.

CPT stands for: Clinical procedures and techniques Current procedural terminology Coding for practice tasks Codes for price tracking

Ans: Current procedural terminology

You see Kevin, a 25-year-old man with a BMI=38 kg/m2 and hyperpigmented plaques with a velvet-like appearance at the nape of the neck and axillary region. He states his skin has had this appearance since he was approximately age 13 and it has not changed significantly over time. He denies itch or pain in these areas. This most likely represents: Acanthosis nigricans. Lichens planus. Actinic keratosis. Erythema migrans.

Ans: Acanthosis nigricans.

In the USA, which of the following is the most common cause of adolescent death? Suicide Homicide Accidental injury Malignancy

Ans: Accidental Injury

James is a 15-year-old who arrives for a well-teen visit with his mother.Prior to the beginning of the visit, his mother pulls you aside and states, "I want him checked for all drugs, but he said he is not using anything and does not want to be tested." Which of the following is your most appropriate response? "What drugs do you think James is taking?" "I cannot force James to take a drug test." "Let's discuss your concerns with James." "Since you are concerned, I can order the test without James' consent."

Ans: "Let's discuss your concerns with James."

Which of the following represents the best choice of abortive migraine therapy for a 55-year-old woman with hypertension that is currently not adequately controlled due to poor medication adherence? Verapamil. Ergotamine. Acetaminophen. Almotriptan.

Ans: Acetaminophen.

You see a 55-year-old woman with type 2 diabetes mellitus, hypertension, and dyslipidemia. Evaluation today reveals a BMI=36 kg/m2. She states, "I just do not know where to start in trying to lose some weight." Which of the following is the most appropriate response to this statement? "How much weight do you want to lose?" "How do you feel about your weight?" "What barriers do you see to losing weight?" "Your blood sugar control will likely improve if you lose some weight."

Ans: "What barriers do you see to losing weight?"

You see Michelle, a 38-year-old woman with moderate persistent asthma who is using medium-dose inhaled fluticasone daily and albuterol via MDI as needed. In a typical month, she uses albuterol 2 puffs "about 2 times, when I feel my chest getting a little tight, and it works right away." In an average month, she has no episodes of nocturnal awakening with cough or wheeze, typically has excellent activity tolerance. Which of the following is the most important additional clinical parameter to obtain today in evaluating Michelle's baseline asthma control? SaO2 Peak expiratory flow Auscultation of breath sounds Resting respiratory rate

Ans: Peak expiratory flow

Which of the following is true concerning acute bacterial prostatitis? Gram-positive organisms are the most common cause of infection. Length of antibiotic therapy is usually 1 week. Perineal pain with defecation is a common complaint. Cephalosporins are first-line therapy.

Ans: Perineal pain with defecation is a common complaint.

You see a 24-year-old mother who presents with her healthy 18-month-old son. He is being seen for a well child visit. An interactive child who is quite playful, his examination is within normal limits. When you ask if the mom has any concerns, she states, "I do not like it when people stare at my baby. He is going to get sick." You respond: "What type of illness do you think your son will get when people look at him?" "You are concerned he will get sick if people look at him?" "Please make sure you come back for a sick visit with your baby whenever you have concerns about your son." "Your baby is just delightful. I can imagine that people often look at him."

Ans: "You are concerned he will get sick if people look at him?"

The most important aspect of skin care for a person with eczema is: Use of occlusive dressings. Frequent bathing with antibacterial soap. Aggressive towel drying after bathing. Frequent application of lubricants.

Ans: Frequent application of lubricants.

Mr. Spaulding's CURB-65 score. __ Based on the scenario below: Mr. Spaulding is a 70-year-old man with a 50 pack-year cigarette smoking history, chronic obstructive pulmonary disease, and hypertension, who presents with a 24-hour history of increasing dyspnea and productive cough with white-yellow sputum. He is alert, oriented, and answers questions with ease. Physical examination reveals the following: Alert, breathing slightly labored at rest, BP=130/78 mm Hg, T=99.8°F (37.7°C), HR=96, RR=22, dullness to percussion over the left base with increased tactile fremitus and tubular breath sounds as well as crackles in the right base. Cardiac examination reveals no S3, no S4, no murmur, with nondistended neck veins. 0 1 2 3 4

Ans: 1

An organization has donated 1000 pediatric vaccines to a community clinic. Ensuring that the distribution of the vaccines is done in a fair and unbiased manner is an example of: Utilitarianism Justice Beneficence

Ans: Justice

Which of the following represents a valid ICD 10 code? 370.21 99202 K25.1 AZ5.32

Ans: K25.1

In a healthy 3½-year-old, what percentage of speech should be intelligible by people who are not in daily contact with the child? About 25% About 50% About 75% Nearly 100%

Ans: Nearly 100%

Match the correct car safety device for each of the following children. An answer may be used more than once. A. Rear-facing car seat B. Forward-facing seat with harness C. Belt-positioning booster seat D. Seat belt 1. An 18-month-old born at 38-weeks' gestation. __ 2. A 9-year-old who is 4 feet 10 inches tall. __ 3. A 1-year-old who is at 50th percentile in height and weight. __ 4. A 7-year-old who is at 75th percentile in height and weight. __ 1-C 2-B 3-D 4-A 1-A 2-D 3-A 4-C 1-B 2-B 3-A 4-D 1-A 2-C 3-C 4-B

Ans: 1-A 2-D 3-A 4-C

A 45-year-old woman calls up your practice and speaks to the triage nurse, stating, "I am going to sue you all. That NP gave me amoxicillin and now I have a have yeast infection." Does this meet the "damages" standard? Yes No

Ans: No

Joe is a 54-year-old man with COPD who presents with an 18-hour history of sudden-onset monoarticular pain consistent with acute gouty arthritis. He denies trauma to the area and has taken acetaminophen 1 g × 2 doses with little effect. The most likely presentation of this condition is: Swelling at the third distal interphalangeal joint. Redness at the first metatarsophalangeal. Firm, white 4 mm nodular auricular lesion. Effusion in the right knee.

Ans: Redness at the first metatarsophalangeal.

True or False? 1. In a febrile child, the degree of temperature reduction in response to antipyretic therapy is not predictive of presence or absence of bacteremia. __ 2. Response to antipyretic medication does not change the likelihood of a febrile child having a serious bacterial infection and should not be used for clinical decision-making. __ 3. The absence of tachypnea is the most useful clinical finding for ruling out pneumonia in children. __ 1-True 2-True 3-False 1-False 2-True 3-False 1-True 2-True 3-True 1-False 2-False 3-False

Ans: 1-True 2-True 3-True

While counseling the mother of a healthy full-term newborn who is being formula-fed, she asks how much and how often will the baby eat. The most appropriate response is: 1.5 to 3 ounces (45-90 mL) every 2-3 hours. 4 to 5 ounces (120-150 mL) every 2-3 hours. About 6 ounces (180 mL) every 3 hours. About 2 ounces (60 mL) every 6-8 hours.

Ans: 1.5 to 3 ounces (45-90 mL) every 2-3 hours.

At which age is a young child at greatest risk of developing iron deficiency anemia? 6 months 18 months 36 months 4 years.

Ans: 18 months

PPIs such as lansoprazole, omeprazole and esomeprazole are all inhibitors of CYP450: 3A4 2C19 1A2 2D6

Ans: 2C19

A first-time father brings in his 1-month-old infant for evaluation. He is concerned that she "throws up all of the time." The child has an appropriate weight gain since the last office visit at age 1 week and appears well today. The nurse practitioner knows that the most common cause of frequent spitting and vomiting in a young infant is: GI tract immaturity allowing reflux. Overfeeding. Pyloric stenosis. Allergy or intolerance to a component of infant formula.

ANs: GI tract immaturity allowing reflux.

A clinical trial is studying whether a vaccine prevents disease. Among those who receive the vaccine, 10% get the disease compared to 40% who received placebo. In this case, the ARR is: 10% 30% 75% 300%

Ans: 30% (40%-10%= 30%) ARR is absolute relative risk reduction or risk difference. If there are 40% who received placebo and only 10% receives the vaccine, then risk difference is remaining 30% of those who do not receive the vaccine or placebo)

In a healthy person, approximately what percentage of the body's total daily physiologic insulin secretion is released basally? 10% 25% 50% 75%

Ans: 50 %

The duration of antimicrobial treatment for active tuberculosis infection is typically: 10 days. 4-6 weeks. 6-9 months. 1-2 years.

Ans: 6-9 months.

In a middle-aged adult, which of the following results would not indicate a category at increased risk for diabetes mellitus? A fasting glucose of 109 mg/dL (6.05 mmol/L) A 1-hour post-prandial glucose of 98 mg/dL (5.44 mmol/L) A hemoglobin A1C=5.9% A 2-hour post-prandial glucose of 152 mg/dL (8.44 mmol/L)

Ans: A 1-hour post-prandial glucose of 98 mg/dL (5.44 mmol/L)

Which of the following is most consistent with the diagnosis of depression? Recurrent diarrhea and cramping Difficulty initiating sleep Diminished cognitive ability Consistent early morning wakening

Ans : Consistent early morning wakening

In which of the following scenarios is parental consent for care required? An 18-year-old female who is seeking a pregnancy termination A 16-year-old female requesting a prescription for oral contraceptives A 15-year-old male requesting testing for sexually transmitted infection A 17-year-old male who requests treatment for contact dermatitis

Ans: A 17-year-old male who requests treatment for contact dermatitis

Mr. Spaulding is a 70-year-old man with a 50 pack-year cigarette smoking history, chronic obstructive pulmonary disease, and hypertension, who presents with a 24-hour history of increasing dyspnea and productive cough with white-yellow sputum. He is alert, oriented, and answers questions with ease. Physical examination reveals the following: Alert, breathing slightly labored at rest, BP=130/78 mm Hg, T=99.8°F (37.7°C), HR=96, RR=22, dullness to percussion over the left base with increased tactile fremitus and tubular breath sounds as well as crackles in the right base. Cardiac examination reveals no S3, no S4, no murmur, with nondistended neck veins. His physical examination findings are suggestive of: A left lower lobe consolidation Diffuse hyperinflation Heart failure Compromised pulmonary vascular perfusion

Ans: A left lower lobe consolidation

A 36-year-old man presents with a 12-h history of anorexia, nausea, and right lower quadrant abdominal pain. A white blood cell count with differential demonstrates: -TWBC=16,500 cells/mm3 -Neutrophils=66% -Bands=8% -Lymphocytes=22% Expected physical examination findings include: Murphy's sign. A palpable left lower quadrant mass. Periumbilical ecchymosis. A positive obturator sign.

Ans: A positive obturator sign (appendicitis)

Maria is a G1P0AB0 woman with is 28-weeks pregnant. Her pregnancy to date has been uneventful and she has no chronic health problems. Maria has not received any immunizations in the past year. 1. What should she receive today? Choose all that apply. A. A Tdap B. Injected influenza immunization C. Live attenuated influenza vaccine (LAIV) D.Pneumococcal vaccine.

Ans: A. A Tdap B. Injected influenza immunization

An appropriate diagnostic test to perform next for Ms. Hopkins is: Based on the below scenario: Ms. Hopkins is a 78-year-old woman with a history of hypertension, dyslipidemia, and stable angina pectoris who presents for follow-up care. She is a former smoker who quit 25 years ago with a 40 pack-year history. She denies chest pain or shortness of breath. Her current medications include a beta-adrenergic antagonist, sustained-release nitrate, angiotensin-converting enzyme inhibitor, statin, low-dose aspirin, and a thiazide diuretic. She complains of feeling a "catch or cramp" in her lower posterior legs bilaterally when she walks for extended periods. This is promptly relieved by rest. Venography of lower legs. Ankle-brachial index (ABI). ECG. Serum B-type natriuretic peptide (BNP).

Ans: Ankle-brachial index (ABI).

Unless no other contraceptive form is acceptable, depot medroxyprogesterone acetate (DMPA; Depo-Provera®) use should be limited to ≤2 years due to the potential for: Endometrial hyperplasia. Acne. Atrophy at the injection site. Bone mineral density loss.

Ans: Bone mineral density loss.

Non-contraceptive benefits of combined oral contraceptive use include a decrease in all of the following except: Ovarian cancer with protracted use. Iron deficiency anemia. C. trachomatis cervicitis. Pelvic inflammatory disease.

Ans: C. trachomatis cervicitis.

Mrs. Rivera is an 82 year-old woman with infrequent contact with her HCP. She presents for sick visit with the following chief complaint, "I get really dizzy when I walk up a flight of stairs." She denies chest pain, agrees that she feels "a bit winded" when physically active; this symptom resolves quickly with cessation of trigger activity. Physical examination reveals the following: BP=110/90, gr 2/6 harsh systolic murmur with radiation to the neck, loudest along USB, softer towards axilla, both heart sounds preserved, no S3 or S4, no neck vein distention, no carotid bruit, delayed carotid upstroke. Her clinical presentation is most consistent with the presence of: Mitral stenosis. Carotid artery dissection. Systolic heart disease. Calcific aortic stenosis.

Ans: Calcific aortic stenosis. MR-ASS or MS-ARD Mitral regurgitation -Aortic stenosis = Systolic murmurs (PMI at 5th ICS) Mitral stenosis- Aortic stenosis = Diastolic murmurs (2nd USB radiating to right side of the neck)

Mrs. Jansen is a 34-year-old healthy female patient who is nursing her first child. She has just been diagnosed with mastitis and wants to know what to do about breastfeeding. The nurse practitioner advises her to: Continue to nurse only on the unaffected side. Pump the breasts and discard the milk. Continue to nurse the baby as the mother tolerates. Discontinue breastfeeding.

Ans: Continue to nurse the baby as the mother tolerates.

breastfeeding mother brings in her 5-month-old son experiencing an episode of gastroenteritis. She reports that he has had 3 loose stools in the past 24 hours and vomited twice. Evaluation reveals that blood pressure, pulse, and skin turgor are all within normal limits. The NP recommends:" Temporarily switching to soy-based formula. Initiating therapy with oral rehydration solution. Supplementing with 50% diluted fruit drink or sports drink. Continuing breastfeeding.

Ans: Continuing breastfeeding.

Major indications that a child could have attention-deficit/hyperactivity disorder (ADHD) include all of the following except: Does not readily respond to his/her name on a regular basis. Symptoms are present before the age of 7 years. Exhibits problematic behavior in 2 different settings. Struggling in school academically.

Ans: Does not readily respond to his/her name on a regular basis.

A new family nurse practitioner is beginning a practice in weight management. While reviewing the literature on techniques to promote weight loss, he finds a research report of 30 adult women who recorded their daily caloric intake on a new iPhone® application while on a weight loss plan. The women lost an average of 10% starting weight during the study. In the discussion section of the paper, the author suggested that based on these data, adult women who used this iPhone® application while on a weight loss plan would lose 10% of starting weight. The new FNP questions whether or not the iPhone® application is the cause of the weight loss. This suggests questionable: Reliability. Power. Confidence level. External validity.

Ans: External validity. In this situation, 30 adult women used Iphone appl. to log in their daily caloric intake has helped these women to lose 10% starting weight. The Iphone appl. is considered an external inference which can be generalised to other situations or outside of the studied population. FNP questions if " the Iphone appl. is the cause of the weight loss for other people as well?"

Mr. Santorini is a 71-year-old man with a 10-year history of hypertension. He is currently taking a fixed-dose lisinopril/hydrochlorothiazide tablet daily and reports excellent adherence. On today's visit, he is without complaint and physical examination is within normal limits, with BP=138/84. He mentions that, for the past three months, he has been sprinkling a "pinch" of cumin and coriander mixture on his food once a day to "help control my blood pressure" and reports "feeling like I have more energy" since starting this. You consider that: This mixture will likely interact with his antihypertensive medications. Using cumin or coriander could raise his blood pressure. Additional medications can be prescribed if his blood pressure control worsens. He is advocating for his health and the practice should be supported.

Ans: He is advocating for his health and the practice should be supported.

As part of Kevin's healthcare, consideration should be given for obtaining which of the following laboratory tests? Based on the below scenario: You see Kevin, a 25-year-old man with a BMI=38 kg/m2 and hyperpigmented plaques with a velvet-like appearance at the nape of the neck and axillary region. He states his skin has had this appearance since he was approximately age 13 and it has not changed significantly over time. He denies itch or pain in these areas. Hemoglobin A1C Serum transaminases Rapid plasma reagent Erythrocyte sedimentation rate Feedback

Ans: Hemoglobin A1C

According to the US medical eligibility criteria for contraceptive use which of the following clinical scenarios describes a Category 3 (exercise caution) situation for the use of a combined oral contraceptive? Presence of factor V Leiden mutation Report of ectopic pregnancy 2 years ago Treatment 3 years ago for high grade squamous intraepithelial neoplasia (HGSIL) with loop electrical excision procedure (LEEP) History of gastric bypass surgery

Ans: History of gastric bypass surgery Individual with hx of gastric bypass often lack of intrinsic factors due to the resection of gastric portion therefore; malabsorption is an issue for oral contraceptive effects is reduced, yielding high risk of pregnancy.

You anticipate finding the following on physical examination during an acute asthma flare with a markedly reduced FEV1? Intercostal retraction Inspiratory crackles Increased tactile fremitus Hyperresonance

Ans: Hyperresonance

our patient has been on combined oral contraceptives for 4 years and is now ready to start a family. She asks how long she should wait after stopping her pills. You advise that it is safe to conceive: Immediately. After 1-2 months. After 3-4 months. After 2 normal menstrual cycles.

Ans: Immediately.

A father presents his 3-year-old with viral gastroenteritis. He reports that she has been experiencing multiple episodes of diarrhea for the past 24 hours, with the most recent episode about 1 hour ago. Evaluation of the child suggests moderate dehydration. The NP recommends: Initiating oral rehydration therapy with oral rehydration solution. Referral for parenteral IV fluid. Offering child ginger ale in small sips. Offering child sports drink (e.g., Gatorade) diluted 1:1 with water in small sips.

Ans: Initiating oral rehydration therapy with oral rehydration solution.

Caleb, a 9-month-old boy, is presented for evaluation after sudden onset of intermittent severe abdominal pain. He often draws his knees to his abdomen when he appears to be in greatest pain. The mother reports that he has had episodes of diarrhea that contain a mixture of blood and mucous. He has not vomited during this time, but also has a lack of appetite. The most likely diagnosis for this child is: Pyloric stenosis. Small bowel obstruction. Giardiasis. Intussusception.

Ans: Intussusception.

Mr. Lam, a 78-year-old man with longstanding GERD, presents with a 1-month history of "feeling like the food gets stuck way down in my throat." This sensation occurs with meats and other solid food types and less likely with softer or liquid foods. He denies nausea, vomiting, constipation, diarrhea, aspiration, or melena. His physical examination is unremarkable. A hemogram today reveals a microcytic hypochromic anemia with an elevated RDW. The most likely etiology of this anemia is: Anemia of chronic disease Vitamin B12 deficiency Iron deficiency Acute blood loss

Ans: Iron deficiency

Juan is a 30-year-old man with a 10-year history of moderate persistent asthma who presents for a routine visit, requesting refills of his medium-dose budesonide per DPI and albuterol per MDI. He states, "I am doing great, no complaints here." His blood pressure, heart rate, temperature, and respiratory rate are within normal limits, he is in no distress, and his physical exam reveals no evidence of wheezing or crackles. You consider that: His current treatment regimen should be continued Due to patient report and today's objective findings, his asthma appears to be well-controlled His asthma therapy should be intensified More information is needed to determine his level of asthma control

Ans: More information is needed to determine his level of asthma control

You see a 62-year-old man, Mr. Tran, who is from Southeast Asia. He is accompanied in the exam room by his 39-year-old son. Both are fluent English speakers. Mr. Tran is seen in follow-up on type 2 diabetes and hypertension. As you speak with Mr. Tran, he smiles, nods his head in response to your questions. He frequently looks down at the floor. You consider that: Mr. Tran is conveying that he understands your questions. His downcast gaze should trigger an assessment for depression or other mood disorder. Directing your questions to Mr. Tran's son is likely a more appropriate approach to the visit. Mr. Tran is showing respect for the healthcare provider's authority.

Ans: Mr. Tran is showing respect for the healthcare provider's authority.

Which of the following is least likely to be noted in the 55-year-old man who presents with bladder cancer? Textile worker for the past 25 years 60 pack-year cigarette smoking history Report of intermittent painless gross hematuria Report of recent-onset intermittent acute urinary retention

Ans: Report of recent-onset intermittent acute urinary retention

A 90-year-old woman with moderate dementia, heart failure, and chronic renal insufficiency presents with a new onset of weakness. Further evaluation reveals marked hyperkalemia with a eGFR=22 mL/min/1.73 m2. Nephrology recommends hemodialysis. When discussing this option with the patient, she declines this treatment. She also inaccurately reports the current year, location, and name of the USA president. When asked why she does not want dialysis, she states, "I do not want to be on a machine. I am quite old and my health is not good." When asked what she believes will happen if she does not have dialysis, she states, "I guess I will die. That is OK." In reflecting on this patient, the NP appreciates that: She demonstrates competency in this situation and her wishes can be followed. Due to her inability to report accurate date, location, or current president, she is not competent to make healthcare decisions. A patient with a documented diagnosis of dementia is considered incompetent to make all healthcare decisions. Court-appointed guardianship should be sought.

Ans: She demonstrates competency in this situation and her wishes can be followed.

The most appropriate treatment option for Jackson is: Based on the below scenario: A mother brings in Jackson, her 4-year-old son, for evaluation after reporting that she stayed up all night with him because he was experiencing somewhat labored breathing and had a loud "barking-like" cough. The child has a hoarse voice, is without acute respiratory distress, and a temperature of 101.6ᵒF (38.7ᵒC). Short-term systemic corticosteroid Nebulized beta2-agonist Inhaled corticosteroid Guaifenesin

Ans: Short-term systemic corticosteroid

A 54-year-old woman presents for evaluation of a "cold sore" on her tongue. She has never had one in the past. She denies any pain, drainage, foul taste, or any other symptoms—just has an ulcer that has been there "for a while." The history is significant for a 62 pack-year smoking history. Physical examination reveals an ulcerated lesion with firm, palpable, indurated margins. The most likely diagnosis is: Aphthous stomatitis. Foreign body. Herpangina. Squamous cell carcinoma.

Ans: Squamous cell carcinoma.

The routine use of diuretics can be considered for patients with heart failure beginning at: Stage A. Stage B. Stage C. Stage D.

Ans: Stage C.

A 52-year-old female of European ancestry with a BMI=24 kg/m2 is diagnosed with new-onset hypertension. She has no significant medical history, no concomitant health problems, and takes no medications. Along with therapeutic lifestyle changes, therapy should begin with which of the following classes of drug? Aldosterone antagonist Beta-adrenergic antagonist Centrally-acting agent Thiazide diuretic

Ans: Thiazide diuretic

A 53-year-old woman complains of vaginal dryness and discomfort during sexual activity . She feels like she might have a urinary tract infection, but the urinalysis is negative. Pelvic examination reveals a lack of vaginal rugae. The nurse practitioner considers prescribing: Antimicrobial therapy. Oral metronidazole. Vaginal antifungal. Topical estrogen.

Ans: Topical estrogen.

Which of the following is unlikely to be reported by Mr. Kane, a 45-year-old man with a BMI=41 kg/m2 who presents with typical GERD symptoms? Unintended weight loss Hoarseness. A recurrent cough Chronic pharyngitis

Ans: Unintended weight loss

In advising a woman with menstruation-related migraine without aura and combined oral contraceptive use, the nurse practitioner considers that: A high-dose estrogen pill should be prescribed. Uninterrupted use can help minimize headache frequency and severity. Headache severity is likely to increase. Cardiovascular risk is markedly increased.

Ans: Uninterrupted use can help minimize headache frequency and severity.

A 33-year-old woman comes to your office and reports that she is "having a stroke." She has no medical history and no significant risk factors for cerebrovascular disease, but today she has sudden onset of inability to tightly close her eyelid and to frown or smile on the right side of her face. Her physical examination is otherwise unremarkable. The nurse practitioner recognizes that this likely represents paralysis of cranial nerve (CN): IV. V. VI. VII.

Ans: VII.

According to the EBP recommendations, nutraceutical options for the prevention of recurrent migraine include the use of all of the following except: Butterbur Feverfew Magnesium Vitamin C

Ans: Vitamin C

Age-related changes in older adults can result in a decreased drug effect for all of the following except: Inhaled muscarinic antagonists Warfarin DHP calcium channel blockers Beta2-agonists

Ans: Warfarin

A common complaint of patients who have been using medroxyprogesterone depot (Depo-Provera®) for more than 1 year is: Hypermenorrhea. Depression. Acne. Weight gain.

Ans: Weight gain.

An APRN prescribes clarithromycin for the treatment of sinusitis in a woman taking simvastatin. While on the antibiotic, the patient develops rhabdomyolysis. Can you see a correlation between the above events? Yes No

Ans: Yes

Your neighbor asks you to refill her high blood pressure medicine as she could not make her last office visit to her provider due to a family emergency. You are aware that she is going through a rough time personally and agree to call in the prescription. Is she now your patient? Yes No

Ans: Yes

Accordi The A1C goal for older adults who are frail or with limited anticipated life expectancy of ≤5 years is: ≤6.5% ≤7.0% ≤8.0% ≤9.0%

Ans: ≤8.0%

When counseling the parents of Joshua, a healthy term newborn, about sleeping safety, the NP advises the following: Position the baby on the side with a positioning wedge or back, as he is most comfortable. Place the baby in a face-up position for sleep. Place a soft bumper in the crib to minimize the risk of injury. Even when a caregiver is in attendance, the tummy-down position is not safe in the first two months of life

Ans: Chapter 16-A- Review of Questions- Primary Care of the Well and Sick Infant, Child, and Teen

Felicia is 18-weeks pregnant with her first child. Which of the following is most consistent with her gestational age? Uterine fundus palpable through the abdominal wall Nausea and breast tenderness Report of quickening Fetal heart tones detectable by abdominal Doppler

Ans: Report of quickening

Teresa's CURB-65 score __ Based on the scenario below: Teresa, a 38-year-old woman with no chronic health problems, presents with a chief complaint of "A cold I cannot shake for the past three weeks. She also reports an intermittent frontal headache and has taken acetaminophen with relief, as well as general malaise and a dry cough that is particularly problematic at night. She denies nausea, vomiting, chills, fever, or dyspnea. She has not taken an antimicrobial within the past year, underwent a bilateral tubal ligation approximately 10 years ago, and is allergic to penicillin with a hive-form reaction. Examination reveals the following: SaO2=97%, BP=114/70 mm Hg, T=98°F (36.7°C), HR=88, RR=20, bilateral coarse late inspiratory crackles without wheeze. She is in no acute distress. Chest x-ray demonstrates bilateral interstitial infiltrates. 0-1 1-2 2-3 3-4

Ans: 0-1

The child remains in "time out" for what period of time? 30 seconds for each year of life 1 minute for each year of life 2 minutes for each year of life 3 minutes for each year of life

Ans: 1 minute for each year of life

The nurse practitioner is counseling a patient who is new to insulin therapy. The patient is advised that when injecting a short-acting, rapid-onset formulation such as insulin aspart, the greatest risk time for hypoglycemia is: 15-60 minutes after injection. 1-3 hours after injection. 3-4 hours after injection. 4-5 hours after injection.

Ans: 1-3 hours after injection. Insulin Analog -- Aspart and Lispro (short-acting)-onset 15mins, peak 1hr, duration 3-4hrs Humilin R and Novolin R (regular)- 30mins, peak 2-3hrs, duration 3-6 hrs NPH (intermediate), Humilin N and Novolin N- 1-2 hrs, peak 6-14hrs, duration 16-24hrs Long-acting Lantus or Detemir- 1-2 hrs, peak and duration - none

Match each of the following findings with the associated disease state. An item can be used more than once. A. Rheumatoid arthritis B. Systemic lupus erythematosus C. Osteoarthritis 1. Anemia of chronic disease 2. Elevated C-reactive protein 3. Joint-space narrowing on x-ray 4. Positive antinuclear antibody titer 1-A & B 2-A & B 3-C 4-A & B 1-A 2-C 3-B & A 4- A & B 1-A & B 2-A 3-A 4-A & C 1-C 2-A 3-C 4-A & B

Ans: 1-A & B ( Anemia of chronic disease -autoimmune- RA and Lupus) 2-A & B (RA and Lupus CRP elevated )3-C (Osteoarthritis presents with joint-space is narrowing on Xray due to thinning of the cartilage) 4-A & B (ANA titers elevated in RA and Lupus as well)

Choose from the following (an option can be used more than once): A. Candida vulvovaginitis B. Trichomoniasis C. Bacterial vaginosis D. Trichomoniasis, Bacterial vaginosis 1. Clotrimazole cream __ 2. Oral metronidazole __ 3. Metronidazole gel __ 4. Clindamycin cream __ 1-A 2-D 3-C 4-C 1-B 2-C 3-D 4-A 1-D 2-D 3-C 4-A 1-A 2-D 3-B 4-C

Ans: 1-A ( Candida vulvovaginitis - use Clotrimazole) 2-D (BV and Trichomoniasis uses oral Metridonazole ) 3-C (BV uses metronidazole cream) 4-C 9 (metronidazole cream treats trichomoniasis)

Match the patient presentations with the following terms: A. Syncope B. Dizziness C. Vertigo 1."All of a sudden, I passed out and woke up on the floor." __ 2. "I feel lightheaded." __ 3. "The room is spinning." __ 1-B 2-C 3-A 1-A 2-B 3-C 1-C 2-B 3-A 1-A 2-C 3-B

Ans: 1-A (Sycope) 2-B (Dizziness) 3-C (the room is spinning)

Choose from the following: A. Bacterial vaginosis B. Candida vulvovaginitis C. Nongonococcal cervicitis/vaginitis 1. Clue cells with alkaline pH __ 2. Pseudohyphae __ 3. Abundant WBCs __ 1-A 2-B 3-C 1-B 2-A 3-C 1-C 2-B 3-A 1-A 2-C 3-B

Ans: 1-A (clue cells with alkaline pH) 2-B (pseudohyphae) 3-C (WBC's)

With an 8:00 AM dose of the following insulins, followed by inadequate dietary intake/excessive energy utilization, at what time would hypoglycemia occur? Select the time of onset of hypoglycemia Medication: A. 8:30AM-10:30AM B. 10:00AM-11:00AM C. 2:00 PM-10:00 PM 1. Lispro (Humalog®) __ 2. Regular insulin (Humulin R®) __ 3. NPH (Novolin N®, Humulin N®) __ 1-A 2-B 3-C 1-C 2-A 3-B 1-B 2-A 3-C 1-A 2-C 3-B

Ans: 1-A (onset: 30 mins, peak: 1-2 hr) 2-B ( onset: 1hr peak: 1-3 hrs 3-C (onset: 1-2 hrs; peak: 3-14hrs)

dentify the following children with AOM as a candidate for initial antimicrobial therapy or watchful waiting therapy. A. Antimicrobial therapy B. Watchful waiting therapy 1. A 5-month-old with unilateral AOM, T=101.2°F (38.4°C),. and 48 -h history of crankiness __ 2. A 3-year-old with bilateral AOM and, T=102.4°F (39.1°C), and a 8- h history of otalgia complaint __ 3. A 4-year-old with unilateral AOM, T= 100.6°F (38.1°C), and a 12 -h history of mild ear discomfort __ 4. A 2 ½-year-old with bilateral AOM, T=101.2°F (38.4°C), and a 3-day history of otalgia complaint __ 1-A 2-B 3-A 4-B 1-B 2-A 3-B 4-B 1-A 2-A 3-B 4-A 1-B 2-A 3-A 4-A

Ans: 1-A 2-A 3-B 4-A

dentify at which age each of the following characteristics would indicate a potential problem with a child's development. A. By 9 months B. By 12 months C. By 16 months 1. Fails to return a smile or other facial expression. __ 2. Lack of any spoken words. __ 3. Shows no response to his/her name". __ 1-A 2-C 3-B 1-B 2-A 3-C 1-A 2-B 3-C 1-C 2-A 3-B

Ans: 1-A 2-C 3-B

Match the following cancer types: A. Cervical cancer B. Endometrial cancer C. Ovarian cancer D. Cervical cancer, Endometrial cancer E. Endometrial cancer, Ovarian cancer F. All 1. Risk factors include obesity and personal history of PCOS __ 2. Risk factors include long-term infection with HPV 16 and/or 18 __ 3. Risk factors include BRCA1 and BRCA2 gene mutation __ 4. Clinical presentation includes unexplained abdominal bloating and constipation __ 5. Clinical presentation includes abnormal vaginal bleeding __ 6. Often without significant clinical signs and symptoms until later disease __ 7. Screening includes Pap test __ 8. No specific screening tests available or recommended __ 1-D 2-F 3-A 4-C 5-E 6-B 7-A 8-C 1-B 2-A 3-C 4-C 5-D 6-F 7-A 8-E 1-E 2-F 3-A 4-C 5-E 6-B 7-A 8-C 1-A 2-E 3-B 4-D 5-B 6-A 7-F 8-D

Ans: 1-B ( PCOS and obesity increases risks of endometrial cancer ) 2-A risks of longterm infection with HPV 16/18 increases risks of cervical cancer 3-C (BRCA1 and BRCA2 gene mutation increase risks of ovarian cancer) 4-C (ovarian cancer has clinical presentation include unexplained abdominal bloating and constipation) 5-D (cervical and endometrial cancers often have abnormal vaginal bleeding) 6-F ( ovarian, endometrial and cervical are often without significant clinical s/s until later disease 7-A Screening for cervical cancer recommends only 8-E (No recommended screening for endometrial and ovarian at present time)

When counseling a patient with hypertension and/or dyslipidemia on recommended physical activity, all of the following are appropriate goals to strive for except: Aerobic physical activity (e.g., brisk walking) of at least 40 minutes at a time. Exercise of at least 3‒4 times per week. Achieving a peak heart rate of 150 BPM or greater. No more than 48 hours without exercise.

Ans: Achieving a peak heart rate of 150 BPM or greater.

Match the following. A. Benzoyl peroxide B. Isotretinoin (Accutane®) C. Tretinoin (Retin-A®) D. Combined oral contraceptive 1. Indicated for treatment of cystic acne __ 2. Most cost-effective topical antibacterial in mild acne __ 3. Used as a keratolytic in acne treatment __ 4. Use results in reduction of androgen levels __ 1-A 2-B 3-C 4-D 1-B 2-A 3-C 4-D 1-C 2-A 3-D 4-B 1-B 2-C 3-A 4-D

Ans: 1-B (Accutane) 2-A (Benzoyl peroxide) 3-C (Tretinoin Retin-A) 4-D(Combined oral contraceptives)

According to the recommendations found in the Beers Criteria, the use of certain medications and the other members of its class should be avoided in the older adult. Match the medication with the rationale for avoiding or using with caution in the elder. A. Zolpidem (Ambien®) B. Amitriptyline (Elavil®) C. Naproxen sodium (Aleve®, Anaprox®) D. Sertraline (Zoloft®) 1. Significant risk of orthostatic hypotension __ 2. Increase in fall and fracture risk __ 3. Potential to promote fluid retention and minimize effect of many anti-HTN medications __ 4. Increased risk for hyponatremia, especially when used with diuretic __ 1-C 2-B 3-A 4-D 1-B 2-D 3-C 4-A 1-B 2-A 3-C 4-D 1-D 2-C 3-B 4-A

Ans: 1-B (Amtriptiline) 2-A (Ambien) 3-C (Naproxen sodium) 4-D (Sertraline-Zoloft)

Match the diagnosis with its most appropriate treatment option. A. Tricyclic antidepressant B. Cilostazol (Pletal®) C. Compression stockings 1. Peripheral artery disease __ 2. Venous insufficiency __ 3. Peripheral neuropathy __ 1-B 2-C 3-A 1-A 2-C 3-B 1-B 2-A 3-C 1-C 2-B 3-A

Ans: 1-B (Cilostazol) 2-C (compression stockings) 3-A (TCA-Neuropathy)

Match the following. An item can be used more than once. A. Paroxetine B. Fluoxetine C. Citalopram D. Escitalopram 1. Most energizing SSRI __ 2. Most sedating SSRI __ 3. Dose limitation due to potential QT prolongation __ 4. Least drug interaction potential __ 5. Longest T ½ __ 1-B 2-A 3-C 4-D 5-B 1-D 2-B 3-C 4-A 5-B 1-B 2-A 3-D 4-A 5-C 1-A 2-C 3-B 4-D 5-A

Ans: 1-B (Flouxetine- most energizing) 2-A (Paroxitine- most sedating) 3-C Citalopram (increase QT prolongation) 4-D (Escitalopram- least interactions with other meds) 5-B (Flouxetine- longest half-life 84 hours)

A 32-year-old landscaper with a history of seasonal allergic rhinitis presents with a variety of symptoms. Match each symptom with the appropriate medication. A. Oral antihistamine B. Decongestant nasal spray C. Corticosteroid nasal spray 1. Acute relief of nasal congestion __ 2. Relieve pharyngeal itch __ 3. Prevent allergy symptoms __ 1-A 2-B 3-C 1-C 2-B 3-A 1-B 2-A 3-C 1-A 2-C 3-B

Ans: 1-B (decongestant nasal spray) 2-A (oral histamine) 3-C ( prevent allergy symptoms)

Who is a combined oral contraceptive (estrogen/progestin) candidate? A. Category 1 B. Category 2 C. Category 3 D. Category 4 E. Category 1 for Initiation & Category 2 for continuation F. Category 3 for Initiation & Category 4 for continuation 1. 22-year-old who smokes 1 PPD__ 2. 29-year-old with pelvic inflammatory disease __ 3. 45-year-old with recurrent tension-type headache: Non-migrainous headache __ 4. Age ›40 years __ 5. 36-year-old woman with well controlled primary hypertension who is taking ARB and HCTZ: HTN with adequate control __ 6. HTN with poor control __ 1-A 2-B 3-E 4-C 5-C 6-D 1-B 2-A 3-E 4-B 5-C 6-D 1-C 2-A 3-D 4-F 5-B 6-A 1-F 2-D 3-A 4-A 5-A 6-C

Ans: 1-B (no contraindications - smoking put pt in Cat 2) 2-A (No contraindications) 3-E (Age >35yrs, hz of non-migrainous headache; if this individual has migrain headache then she is in Cat 4 -absolute risk of stroke or blood clots) 4-B (Age > 35yrs but no other contraindications) 5-C (caution use with hx of HTN but her HTN is controlled; increased caution of OCC use due to tendency to increase blood pressure) 6-D (Cat 4 Absolute contraindications for OCC use regardless of age)

Indicate the appropriate neonatal/-infant reflex for each description. A. Moro reflex B. Palmar grasp C. Babinski reflex D. Stepping reflex E. Rooting reflex 1. Walking motion made with legs and feet when held upright and feet touching the ground. Appears for first 3-4 months, then reappears at 12-24 months. __ 2. Turning of head and sucking when cheek is stroked. No longer seen by 6-12 months. __ 3. Throwing out arms and legs followed by pulling them back to the body following a sudden movement or loud noise. No longer seen by 16 weeks. __ 4. Stroking the sole of the foot elicits fanning of the toes. No longer seen by 6 months. __ 1-D 2-E 3-A 4-C 1-A 2-B 3-D 4-E 1-D 2-E 3-B 4-A 1-C 2-D 3-B 4-A

Ans: 1-D (stepping reflex) 2-E (rooting reflex) 3-A (moro reflex) 4-C (babinski reflex)

Match each clinical presentation with one of the following conditions. A. Erosive gastritis B. Acute pancreatitis C. Duodenal ulcer D. Cholecystitis E. Diverticulitis 1. A 45-year-old man who drinks 8-10 beers/day with a 12-hour history of acute-onset epigastric pain radiating into the back with bloating, nausea, vomiting. Objective: Epigastric tenderness, hypoactive bowel sounds, abdomen distended and hypertympanic. Laboratory evaluation reveals elevated lipase and amylase. __ 2. A 64-year-old woman with a 3-day history of intermittent left lower quadrant (LLQ) abdominal pain accompanied by fever, cramping, nausea, and 4-5 loose stools per day. Objective: Abdomen soft, +bowel sounds, tenderness to LLQ abdominal palpation, negative Blumberg's sign. Laboratory evaluation reveals leukocytosis with neutrophilia. __ 3. A 34-year-old man w/3-month history of intermittent upper abdominal pain described as epigastric burning, gnawing pain about 2-3 h PC. Relief with foods, antacids. Awakening at 1-2 AM with symptoms. Objective: Tender at the epigastrium, LUQ, slightly hyperactive bowel sounds. __ 4. A 52-year-old woman who was recently laid off from her job, taking 3-4 doses of ibuprofen/day for the past 2-3 months to help with headaches; 1-month history of intermittent nausea, burning, and pain, limited to upper abdomen, often worse with eating. Objective: Tender at the epigastrium, LUQ, slightly hyperactive bowel sounds. __ 5. A 54-year-old woman who presents with a 24-h history of significant epigastric and RUQ abdominal pain that is constant with 2‒3-minute periods of increased pain, accompanied by nausea, 2 episodes of vomiting, and intermittent fever. Objective: Tenderness at the epigastrium and abdominal RUQ, positive Murphy's sign, moderately elevated AST, ALT, and ALP. __ 1-B 2-C 3-A 4-E 5-D 1-A 2-D 3-C 4-B 5-E 1-B 2-E 3-C 4-A 5-D 1-C 2-A 3-E 4-B 5-D

Ans: 1-B 2-E 3-C 4-A 5-D

Match each test or sign with the relevant condition below: A. Drop arm test B. Finkelstein test C. McMurray test D. Lachman test E. Spurling test F. Straight-leg raising test G. Talar tilt H. Tinel's Sign Conditions: 1. Meniscal tear __ 2. Ankle instability __ 3. Carpal tunnel syndrome __ 4. Anterior cruciate ligament tear __ 5. Lumbar nerve root compression __ 6. Cervical nerve root compression __ 7. Rotator cuff evaluation __ 8. De Quervain's tenosynovitis __

Ans: 1-C (McMurray Test- Meniscal tear of the knee) 2-G (Talar Tilt test for ankle instability) 3-H (Tinel's sign for Carpal tunnel syndrome - two hand place against with hyperflexion of the wrists, if numbness or tingling sensation is elicited test is +) 4-D (Lachman Test for ACL injury of the knee where tendon is twisted causing injury; patient cannot squat, very painful) 5-F (straigth leg raising test_ leg is raised 45 degrees, ask pt to hyperextend against the examiner's hand if pt is experienced pain, test is +) 6-E (Spruling test - test for range of motion (flexion.. of neck) 7-A (Drop-arm test for rotator cuff evaluation) 8-B (Finkelstein test)

Which of the following is most consistent with pyloric stenosis (upper GI obstruction [PS]) or intussusception (lower GI obstruction [I]) or both? A. Pyloric stenosis B. Intussusception C. Both 1. Significantly more common in males. __ 2. Sudden onset, colicky, severe, and intermittent abdominal pain, often with knees drawn to chest during most intense discomfort. __ 3. Accompanied by loose stools that are often described as currant jelly appearance (mixture of blood and sloughed mucous). __ 4. Most common time for symptom onset is= Approximately age 3 weeks. __ 5. Post-fed projectile vomiting is present, with the baby eager to eat again immediately post emesis. __ 6. Accompanied by a sausage-shaped abdominal mass. __ 7. Olive-shaped RUQ abdominal mass occasionally noted. __ 8. Usually occurs between ages 6-12 months. __ 9. Ultrasonography is usually first-line diagnostic study. __ 1-A 2-B 3-A 4-A 5-B 6-C 7-C 8-B 9-A 1-C 2-B 3-B 4-A 5-A 6-B 7-A 8-B 9-C 1-C 2-A 3-B 4-A 5-B 6-B 7-A 8-B 9-A 1-B 2-B 3-C 4-C 5-C 6-A 7-B 8-A 9-B

Ans: 1-C (both) 2-B (Intussusception) 3-B (Intussusception) 4-A (pyloric stenosis) 5-A (pyloric stenosis) 6-B (intussusception) 7-A (pyloric stenosis) 8-B (intussusception) 9-C (both)

Treatment for which of the following STIs? An option can be used more than once. A. Ceftriaxone B. Injectable penicillin C. Imiquimod D. Trichloroacetic acid (TCA) 1. External genital warts in a 25-year-old man __ 2. Gonococcal urethritis in a 22-year-old man __ 3. Syphilis in a 45-year-old man __ 4. External genital warts in a 28-year-old pregnant woman __ 1-C 2-A 3-B 4-D 1-D 2-A 3-B 4-C 1-A 2-B 3-C 4-D 1-B 2-D 3-A 4-C

Ans: 1-C (use Imiquimod cream to treats external genital warts ) 2-A (Ceftriaxone 2 gr IM treats gonococcal urethritis ) 3-B (Injectable Penicillin 2 Mill VK treats Syphillis) 4-D (Trichloacetic acid uses to treat external genital warts for 28yr pregnant woman)

Match each medication used to manage dyslipidemia with its major anticipated change in the lipid profile. An answer can be used more than once. A. Decreases triglycerides B. Decreases triglycerides, increases HDL-C C. Decreases LDL-C 1. Statins __ 2. Niacin __ 3. Fibrates __ 4. Fish oil __ 5. PCSK9 inhibitor __ 1-C 2-A 3-B 4-B 5-A 1-C 2-B 3-B 4-A 5-C 1-B 2-C 3-A 4-B 5-A 1-A 2-B 3-C 4-C 5-A

Ans: 1-C (↓ triglycerides) 2-B ↓triglycerides, ↑ HDL 3-B (↓ triglycerides, ↑HDL) 4-A ( ↓trigylcerides ) 5-C (↓LDL)

Match each of the following signs and symptoms with the appropriate dehydration status for a younger child. A. Mild dehydration B. Moderate dehydration C. Severe dehydration 1. Reduced blood pressure, Fontanels depressed, lethargic __ 2. Normal blood pressure, capillary refill of <1.5 seconds, slightly dry lips and thick saliva __ 3. Slightly sunken eyes, normal blood pressure, capillary refill of ~2 seconds __ 1-B 2-A 3-C 1-A 2-B 3-C 1-C 2-B 3-A 1-C 2-A 3-B

Ans: 1-C severe dyhration (reduced BP, fontanels depressed and lethargic) 2-A mild dehydration (normal BP, cap refill<1.5seconds, slightly dry lips and thick saliva) 3-B moderate dehydration ( slightly sunken eyes, normal BP, cap refill < 2 sec)

Match the following: A. Hydrocele B. Varicocele C. Testicular torsion D. Phimosis E. Paraphimosis F. Cryptorchidism 1. Retracted foreskin that cannot be brought forward to cover the glans __ 2. A palpable "nest of worms" scrotal mass that is only evident in standing position __ 3. Collection of serous fluid that causes painless scrotal swelling, easily recognized by transillumination __ 4. With this, the foreskin cannot be pulled back to expose the glans __ 5. Characterized by scrotal pain and loss of the cremasteric reflex __ 6. Testicle located in inguinal canal or abdomen __ 1-B 2-C 3-A 4-D 5-E 6-F 1-D 2-A 3-B 4-C 5-F 6-E 1-E 2-B 3-A 4-D 5-C 6-F 1-A 2-F 3-E 4-C 5-D 6-B

Ans: 1-E (paraphimosis) 2-B (varicocele) 3- (hydrocele) A 4- phimosis D 5- testicular torsion C 6- cryptochidism

Match each type of urinary incontinence with its most appropriate management option [Each condition can have multiple answers]. A. Availability of an assistant to help with toileting activities B. Identify and treat the underlying condition C. Antimuscarinic therapy D. Pelvic floor exercises E. Antimuscarinic therapy or pelvic floor exercises 1. Urge incontinence __ 2. Stress incontinence __ 3. Functional incontinence __ 4. Transient incontinence __ 1-B 2-C 3-D 4-A 1-A 2-D 3-C 4-B 1-E 2-D 3-A 4-B 1-E 2-A 3-C 4-B

Ans: 1-E urge incontinence (antimuscarinic therapy or pelvic floor exercises) 2-D stress incontinence - pelvic floor exercises (Kegel) 3- A functional incontinence - availabilty of an assistant to help with toileting activities 4-B transient incontinence -usually associated with acute illness/ identify and treat underlying condition.

Pediatric immunizations: True or false? 1. MMR should not be given to a 12-month-old whose mother is pregnant. __ 2. A 6-month-old who is taking amoxicillin for acute otitis media (AOM) should have immunizations delayed until the antimicrobial course is completed. __ 3. Preterm infants are usually immunized at the schedule that corresponds with their birth or extrauterine age. __ 4. One of the best ways to protect infants younger than 6 months of age from influenza is to make sure members of their household and their caregivers are vaccinated against the disease. __ 5. Pregnant women should receive inactivated influenza vaccine to protect both mother and unborn child. __ 6. The risk of autism can be reduced through the use of an early childhood vaccination schedule that minimizes the number of immunizations given at a single visit. __ 7. In order to avoid post-vaccine discomfort, younger children should be given a weight- and age-appropriate dose of an antipyretic, such as acetaminophen or ibuprofen, prior to receiving immunizations. __ 8. Children 6-11 months of age who are traveling outside the United States should receive 1 dose of MMR. __ 1-True 2-True 3-False 4-True 5-False 6-True 7-False 8-False 1-False 2-True 3-True 4-True 5-False 6-True 7-False 8-True 1-False 2-False 3-True 4-True 5-True 6-False 7-False 8-True 1-False 2-True 3-False 4-True 5-False 6-True 7-False 8-True

Ans: 1-False 2-False 3-True 4-True 5-True 6-False 7-False 8-True

True/False In keeping with HIPAA's requirements: 1. You cannot announce a patient's name in a waiting room.__ 2. If interrupted during a patient visit, prior to leaving the exam room, the healthcare provider should lock or otherwise disable the electronic health record (EHR) to minimize the risk of unauthorized access to information.__ 3. A healthcare provider can leave a patient a message on his or her answering machine unless specifically asked not to do so but the amount of information disclosed should be limited.__ 4. Unless asked not to do so by the patient, a healthcare provider is allowed to leave a message with a family member or other person who answers the phone when the patient is not home.__ 5. If interrupted during a patient visit, prior to leaving the exam room, the healthcare provider should take the paper healthcare record with him/her, in order to minimize the risk of unauthorized access to information.__ 6. Unless under the control of the healthcare provider or staff, paper records should be kept in a secure location, such as a locked desk, locked filing cabinet or office with appropriate staff.__ 7. An employee of a healthcare facility is only able to access patient records for legitimate, job-related purposes.__ 8. If a patient requests a paper copy of his/ her record, the healthcare facility can charge a reasonable fee for this service.__ 1-False 2-True 3-True 4-True 5-True 6-True 7-True 8-True 1-False 2-False 3-True 4-False 5-True 6-True 7-True 8-True 1-True 2-True 3-True 4-False 5-False 6-True 7-True 8-False

Ans: 1-False 2-True 3-True 4-True 5-True 6-True 7-True 8-True

ndicate whether each characteristic is better associated with irritable bowel syndrome (IBS) or inflammatory bowel disease (IBD). 1. Intestinal ulceration present. 2. Absence of rectal bleeding. 3. CRP and ESR levels are elevated. 4. Surgical intervention often needed. 1-IBD 2-IBS 3-IBS 4-IBD 1-IBS 2-IBS 3-IBS 4-IBD 1-IBD 2-IBS 3-IBD 4-IBD 1-IBS 2-IBD 3-IBS 4-IBD

Ans: 1-IBD 2-IBS 3-IBD 4-IBD

When a patient is seen by a nurse practitioner, the visit can be charged to Medicare using "incident-to" billing. If the criteria for this type of visit are fulfilled, then anticipated reimbursement is at what percentage of the physician rate? 75 80 90

Ans: 100%

Indicate whether each event is likely due to normal age-related mental changes (N) or is a possible warning sign of Alzheimer-type dementia (W). 1. Occasionally adding the wrong amount of an ingredient when following a recipe. __ 2. Getting lost while driving to usual house of worship. __ 3. Placing the house keys in the freezer. __ 4. Forgetting a son's birthday until late in the day. __ 5. Incorrectly identifies the current US President. __ 6. Cannot immediately remember what they had for dinner the night before but remembers this later. __ 1-N 2-N 3-W 4-N 5-W 6-W 1-W 2-W 3-N 4-N 5-W 6-W 1-N 2-W 3-W 4-N 5-W 6-N 1-N 2-W 3-N 4-N 5-W 6-W

Ans: 1-N (Age-related change- occasionally adding wrong ingredient...) 2-W ( Dementia related - getting lost while driving to familiar house...) 3-W (placing the house keys in the freezer) 4-N (forgetting son's birthday until later in the day) 5-W (incorrectly identifies the current US president) 6-N (cannot remember what they had for dinner last night but remembers it now)

Adolescent issues: True or false? 1. Although adolescents tend to drink alcohol less frequently than adults, they drink considerably more alcohol per occasion of drinking. __ 2. USPSTF recommends depression screening using a validated questionnaire in adolescents (ages 12-18 years). __ 3. The majority of states require either parental consent or notification for teenagers younger than 18 to have a pregnancy termination. __ 1-True 2-True 3-True 1-False 2-True 3-True 1-False 2-True 3-False 1-True 2-False 3-False

Ans: 1-True 2-True 3-True

Which of the following is unlikely to be noted in Tom, a 62-year-old man with lumbar spinal stenosis? Report of back pain worsening with standing Bilateral leg numbness Pain improvement with flexed forward position Absent pedal pulses

Ans: Absent pedal pulses

True or false? When approaching the provision of primary care for LGBT youth, the NP considers that: 1. The healthcare provider should ask the adolescent how he/she self-identifies. __ 2. Therapy which attempts to change one's sexual orientation or gender identity is inconsistent with current standards of medical care. __ 3. Because victimized LGBT youth are at increased risk of depression and suicidality, providers should screen for these mental health issues and intervene as appropriate. __ 4. In adolescents, sexual orientation and gender identity are relatively fixed constructs. __ 1-True 2-True 3-True 4-True 1-False 2-True 3-True 4-True 1-True 2-True 3-True 4-False 1-True 2-False 3-True 4-False

Ans: 1-True 2-True 3-True 4-False

Which of the following are potential candidates to receive the PCV13 pneumococcal vaccine? (YES or NO) 1. A 67-year-old man who received the PPSV23 vaccine 1 year ago __ 2. A healthy 58-year-old woman who is receiving the seasonal influenza vaccine today __ 3. A 48-year-old man who uses smokeless tobacco __ 4. A 36-year-old woman with HIV infection who received the PPSV23 vaccine 3 years ago __ 1-Yes 2-No 3-Yes 4-No 1-No 2-No 3-Yes 4-No 1-Yes 2-No 3-No 4-No 1-Yes 2-No 3-No 4-Yes

Ans: 1-Yes 2-No 3-No 4-Yes

Of the following, who should receive Tdap (tetanus, diphtheria, acellular pertussis) vaccine? (Yes or no) 1. A 28-year-old woman who is 29-weeks pregnant and received Tdap approximately 3 years ago __ 2. The spouse of a woman in the 2nd trimester of pregnancy who provides documentation of receiving Tdap 2 years ago __ 3. A 70-year-old man who received a Td about 8 years ago, slated to be one of his son's newborn's caregivers __ 1-Yes 2-No 3-Yes 1-Yes 2-Yes 3-Yes 1-No 2-No 3-Yes 1-No 2-Yes 3-Yes

Ans: 1-Yes 2-No 3-Yes

Indicate whether each of the suggested long-acting reversible contraceptives (LARCs) for the following patients is (yes) or is not (no) appropriate. 1. A 19-year-old woman to receive the levonorgestrel-releasing IUS (Mirena®) __ 2. A 30-year-old woman who smokes 1 PPD to receive the etonogestrel implant (Nexplanon®) __ 3. A 38-year-old woman with a history of large uterine fibroids and heavy menses to receive the copper-containing IUD (ParaGard®) __ 4. A 28-year-old woman who is currently taking antiepileptic medications to receive the etonogestrel implant (Nexplanon®) __ 1-No 2-Yes 3-No 4-Yes 1-No 2-No 3-No 4-Yes 1-Yes 2-Yes 3-No 4-Yes 1-Yes 2-No 3-No 4-Yes

Ans: 1-Yes 2-Yes 3-No 4-Yes

Who is a Cu-IUD (ParaGard®) or LNG-IUS (Mirena®) candidate (category 1 or 2)? Yes or No 1. 45-year-old nulliparous woman 2. 33-year-old who smokes 2 PPD 3. 25-year-old with seizure disorder 4. 33-year-old woman with HIV (excludes AIDS-defining illness) 1-Yes 2-Yes 3-No 4-No 1-Yes 2-Yes 3-Yes 4-Yes 1-No 2-Yes 3-No 4-Yes 1-No 2-No 3-Yes 4-No

Ans: 1-Yes 2-Yes 3-Yes 4-Yes Cu_IUD (ParaGard) and LNG -IUS (levonogestrol - progestin-based only, no estrogen intrauterine device) therefore, no contraindications for all these women.

Match each of the following patients with the most appropriate diagnosis. A. Increased intracranial pressure B. Migraine with aura C. Tension-type headache D. Cluster headache 1. A 33-year-old man who reports a 3-week history of recurrent headaches that awaken him during the night. Pain lasts up to 2 hours and is focused primarily behind the right eye. __ 2. A 29-year-old woman with a 2-year history of recurrent, unilateral, pulsating headaches that are often accompanied by nausea/vomiting and photophobia. She reports seeing "squiggles before my eyes" about 10 minutes before a headache occurs. __ 3. A 54-year-old man reporting occasional bilateral, pressing, nonpulsatile headaches of moderate intensity without nausea, photophobia, or phonophobia. __ 4. A 45-year-old man who reports a pressing, nonpulsatile headache that is greatest in severity upon awakening each morning and lessens as the day progresses. __ 1-A 2-C 3-B 4-D 1-D 2-B 3-C 4-A 1-B 2-A 3-D 4-C

Ans: 1-cluster head ( 4. A 45-year-old man who reports a pressing, nonpulsatile headache that is greatest in severity upon awakening each morning and lessens as the day progresses) 2- Migraine w aura (A 29-year-old woman with a year history of recurrent, unilateral, pulsating headaches that are often accompanied by nausea/vomiting and photophobia. She reports seeing "squiggles before my eyes" about 10 minutes before a headache occurs. 3-tension-type h/a (A 33-year-old man who reports a 3-week history of recurrent headaches that awaken him during the night. Pain lasts up to 2 hours and is focused primarily behind the right eye) 4- Increased intracranial pressure ( A 45-year-old man who reports a pressing, nonpulsatile headache that is greatest in severity upon awakening each morning and lessens as the day progresses).

In the above example, the annual incidence of PAD in this population is approximately: 1%. 10%. 500. 1/2000 persons.

Ans: 1/2000 persons if there 500 new cases out of 10,000 people with PAD are diagnosed each year. Calculation : 1,000,000 / 500 = 2000 persons

In the preceding question, the dependent variable is the: iPhone® application. 10% weight loss. Gender. Sample Size.

Ans: 10%

Identify the two children that are at greatest risk for iron deficiency anemia. 3-month-old who takes about 24 oz (0.71 L) of iron-fortified formula per day 11-month-old, breastfed about 7 times a day, taking iron-fortified cereal, fruit, and vegetables three times per day 16-month-old who drinks about 1 qt (0.95 L) whole milk per day and a few solids 4-year-old who eats small amounts of meat, fruit, and cheese 6-month-old who was born at 30 weeks' gestation and is exclusively breastfed without additional supplements

Ans: 16-month-old who drinks about 1 qt (0.95 L) whole milk per day and a few solids 6-month-old who was born at 30 weeks' gestation and is exclusively breastfed without additional supplements

The American Academy of Pediatrics recommends screening for autism at which of the following times in early childhood? 6 and 12 months 12 and 18 months 18 and 24 months 24 and 30 months

Ans: 18 and 24 months

n teaching a family about "time out," this method is appropriate to introduce the concept at about the age of _____________. 12-18 months. 18-24 months. 24-30 months. 30-36 months.

Ans: 18-24 months.

The typical international normalized ratio (INR) goal during warfarin therapy for a 65-year-old patient with atrial fibrillation is: 1.0‒2.0. 1.5‒2.5. 2.0‒3.0. 2.5‒3.5.

Ans: 2.0‒3.0.

For a patient who is >60 years old, with hepatic impairment, or taking concomitant cimetidine, the recommended daily dose of citalopram should not exceed: 10 mg 20 mg 40 mg 60 mg

Ans: 20 mg

Thomas is a neonate who was born at 32 weeks' gestation. The adjusted age calculation should be used to assess his development until age: 6 months. 12 months. 18 months. 24 months.

Ans: 24 months.

When counseling the parents of a healthy 2-week-old boy about supervised tummy time, you recommend slowly building up to a total of _______ per day. 15 minutes 30 minutes 1 hour 2 hours

Ans: 30 minutes

A full-term pregnancy is one where the birth occurs between: 37 weeks and 38 weeks of gestation. 37 weeks and 38 weeks plus 6 days of gestation. 39 weeks and 40 weeks plus 6 days of gestation. 41 weeks and 41 weeks plus 6 days of gestation.

Ans: 39 weeks and 40 weeks plus 6 days of gestation.

Mr. Spaulding's current medications include an inhaled corticosteroid, a long-acting beta2-agonist, an ACE inhibitor with a thiazide diuretic, a statin, and low-dose aspirin, as well as a short-acting beta2-agonist as needed. Laboratory testing reveals BUN and hematocrit=WNL. The preferred choice of antimicrobial therapy for Mr. Spaulding is a: Based on the scenario below: Mr. Spaulding is a 70-year-old man with a 50 pack-year cigarette smoking history, chronic obstructive pulmonary disease, and hypertension, who presents with a 24-hour history of increasing dyspnea and productive cough with white-yellow sputum. He is alert, oriented, and answers questions with ease. Physical examination reveals the following: Alert, breathing slightly labored at rest, BP=130/78 mm Hg, T=99.8°F (37.7°C), HR=96, RR=22, dullness to percussion over the left base with increased tactile fremitus and tubular breath sounds as well as crackles in the right base. Cardiac examination reveals no S3, no S4, no murmur, with nondistended neck veins. 7-day course of oral clarithromycin 5-day course of oral levofloxacin 10-day course of oral amoxicillin-clavulanate 3-day course of injectable ceftriaxone

Ans: 5-day course of oral levofloxacin

You see a well child who resists being placed in a supine position on the exam table, cries loudly when her parent is out of view, and has visibly erupting lower central incisors. This child is approximately what age? 6-8 weeks 3-4 months 7-8 months 2-3 years

Ans: 7-8 months

Physiologic gynecomastia is usually found in which of the following? A 14-year-old male who is at Tanner stage 3 A 12-year-old male who is at Tanner stage 2 A 17-year-old male who is at Tanner stage 5 A 10-year-old male who is at Tanner stage 1

Ans: A 14-year-old male who is at Tanner stage 3

Anticipated clinical findings in acute appendicitis for Jordan, an otherwise well 24-year-old man with a BMI=32 kg/m2 include all of the following except: Leukocytosis with neutrophilia and bandemia Positive obturator and psoas signs A 2-hour history of sudden onset of vomiting and generalized abdominal pain accompanied by fever A 12-hour history of epigastric discomfort and anorexia that gradually shifts to nausea and right lower quadrant abdominal pain

Ans: A 2-hour history of sudden onset of vomiting and generalized abdominal pain accompanied by fever

Which of the following is likely to cause cardiac dysrhythmia and seizures when taken in an intentional ingestion equivalent to a typical adult therapeutic dose? A 4-week supply of fluoxetine (SSRI) A 2-week supply of nortriptyline (TCA) A 3-week supply of venlafaxine (SNRI) A 3-day supply of diazepam (BENZO)

Ans: A 2-week supply of nortriptyline

Of the following in need of an antidepressant, who is the best candidate for fluoxetine (Prozac®) therapy? An 80-year-old woman who is taking multiple medications and who presents with depressed mood and agitation A 45-year-old man with anorgasmia who is an occasional marijuana user A 28-year-old woman who occasionally "skips a dose" of her prescribed medication and is using a progestin implant (Nexplanon®) for contraception A 44-year-old woman with decreased appetite who is on hydrochlorothiazide for the treatment of hypertension

Ans: A 28-year-old woman who occasionally "skips a dose" of her prescribed medication and is using a progestin implant (Nexplanon®) for contraception

All of the following are appropriate candidates for tuberculosis (TB) testing except: A 28-year-old with HIV infection. A 55-year-old man who recently emigrated from a country where TB is endemic. A 33-year-old woman who works at a child care center. A 56-year-old man who was recently released from prison.

Ans: A 33-year-old woman who works at a child care center.

The use of opioid analgesia is most appropriate in which of the following clinical situations? A 40-year-old man with acute low back pain × 2 days A 28-year-old woman with a 24-hour history of migraine headache A 48-year-old woman who is 1 day postoperative after abdominal hysterectomy A 54-year-old woman with 5-year history of fibromyalgia

Ans: A 48-year-old woman who is 1 day postoperative after abdominal hysterectomy

Which of the following is the most common pathogen implicated in acute bronchitis? S. pneumoniae H. influenzae M. pneumoniae Respiratory virus

Ans: Respiratory virus

A father presents his 2-year-old daughter with suspected gastroenteritis. She has had two episodes of vomiting and several episodes of diarrhea beginning 24 hours ago. Her last diarrheal episode was 2 hours ago and her diaper is wet. She has been able to consume small amounts of fluid but shows signs of mild dehydration. An appropriate treatment option to prevent further dehydration is a single oral dose of: An antidiarrheal agent (e.g., bismuth salicylate [Pepto Bismol®]). An antimotility agent (e.g., loperamide [Imodium®]). A 5-HT3 antagonist (e.g., ondansetron [Zofran®]). Antiparasitic antimicrobial (e.g. metronidazole [Flagyl®]).

Ans: A 5-HT3 antagonist (e.g., ondansetron [Zofran®]).

The results of her laboratory testing do not show alarm findings. In treating Teresa, you prescribe the following. Based on the scenario below: Teresa, a 38-year-old woman with no chronic health problems, presents with a chief complaint of "A cold I cannot shake for the past three weeks. She also reports an intermittent frontal headache and has taken acetaminophen with relief, as well as general malaise and a dry cough that is particularly problematic at night. She denies nausea, vomiting, chills, fever, or dyspnea. She has not taken an antimicrobial within the past year, underwent a bilateral tubal ligation approximately 10 years ago, and is allergic to penicillin with a hive-form reaction. Examination reveals the following: SaO2=97%, BP=114/70 mm Hg, T=98°F (36.7°C), HR=88, RR=20, bilateral coarse late inspiratory crackles without wheeze. She is in no acute distress. Chest x-ray demonstrates bilateral interstitial infiltrates. A 7-day course of oral doxycycline A 10-day course of oral moxifloxacin A 3-day course of parenteral vancomycin Timed inhaled short-acting beta2-agonist use

Ans: A 7-day course of oral doxycycline

A 76-year-old man with COPD, who uses an inhaled LAMA daily on a set schedule and SABA via MDI as needed for symptom relief, presents with a COPD exacerbation. Which of the following describes the role of imaging in the evaluation of COPD exacerbation? Chest x-ray is a routine part of the evaluation of a person with COPD exacerbation In COPD exacerbation, chest x-ray should not be obtained due to radiation risk A chest x-ray should be ordered in COPD exacerbation in the patient with fever and/or low SaO2 to help rule out concomitant pneumonia A thoracic CT is the preferred imaging study to order in a COPD exacerbation

Ans: A chest x-ray should be ordered in COPD exacerbation in the patient with fever and/or low SaO2 to help rule out concomitant pneumonia

Type 2 diabetes mellitus is best described as: A genetically-base autoimmune process. A consequence of obesity. A condition largely dictated by environmental factors. A disorder of insulin resistance with eventual insulin deficiency.

Ans: A disorder of insulin resistance with eventual insulin deficiency.

Jack is a 16 year-old varsity basketball player who presents for a sports clearance physical examination. His health history is unremarkable and reports excellent activity tolerance. Which of the following would be a finding on Jack's cardiac examination that would warrant immediate evaluation? A split second heart sound that increases on inspiration and closes on expiration accompanied by a point of maximum impulse (PMI) at the 5th intercostal space (ICS) midclavicular line (MCL) A grade 2/6 midsystolic murmur that increases in intensity with position change from supine to standing accompanied by a loud S4 PMI with palpable single impulse at the 5th intercostal space accompanied by S1 louder than S2 at the apex A grade 2/6 systolic murmur noted over the precordium without radiation that disappears with position change from supine to standing accompanied by S2 louder than S1 at the base

Ans: A grade 2/6 midsystolic murmur that increases in intensity with position change from supine to standing accompanied by a loud S4

A prostate cancer lesion detectable by digital rectal examination will likely be described as: A hard, immobile induration. A bilateral enlargement. A painful flocculent t lesion. A rubbery gland with central depression.

Ans: A hard, immobile induration.

our neighbor asks you to refill her high blood pressure medicine as she could not make her last office visit to her provider due to a family emergency. You are aware that she is going through a difficult time personally and agree to call in the prescription. As a result: A healthcare provider-patient relationship has been formed. The Good Samaritan Act will protect you from liability since you volunteered to help her out. You should charge your neighbor for a standard office visit in order to keep your relationship with her on a professional level. You realize this action is acceptable as her antihypertensive medication is not a controlled substance.

Ans: A healthcare provider-patient relationship has been formed.

ou see a 45-year-old woman with major depressive disorder (MDD) who started taking standard dose sertraline one week ago. She returns today with a chief complaint of "not really feeling any better." "In fact, I think I might feel worse. I have this on-and-off headache right over my eyes since I started the medication." Results of physical examination reveal the following: Well-groomed and appropriately dressed for the occasion, PERLA, fundi WNL, CN 2-12 intact, clear, fluid speech. Today, you provide the following advice, choosing two of the following responses. The sertraline dose is likely inadequate and needs to be increased. She should be switched to a SNRI such as venlafaxine. Due to her headache report, the sertraline should be immediately discontinued. A lag of a number of weeks in the onset of SSRI therapeutic effect is expected. Frontal headache is a common short-term problem with early SSRI use.

Ans: A lag of a number of weeks in the onset of SSRI therapeutic effect is expected. Frontal headache is a common short-term problem with early SSRI use.

The NP is counseling a 51-year-old male patient who is considering undergoing a PSA test to check for prostate cancer. In explaining the possibility of a false negative result of the test, the NP explains that this can occur when: A patient without cancer gets a negative test result. A patient without cancer gets a positive test result. A patient with cancer gets a negative test result. A patient with cancer gets a positive result.

Ans: A patient with cancer gets a negative test result.

diagnostic test that is used to identify patients with colon cancer is determined to have a high sensitivity. Which of the following statements about the test is most accurate? A negative test result is not a reliable indicator for the absence of colon cancer. The test reliably provides a negative result for those without colon cancer. A positive test result reliably identifies someone with colon cancer. A positive test result is not a reliable indicator for the presence of colon cancer.

Ans: A positive test result reliably identifies someone with colon cancer.

Michelle's physical examination is consistent with an asthma flare triggered by viral RTI. Her medication regimen should be adjusted to include which of the following? Choose all that apply. Based on the scenario below: You see Michelle, a 38-year-old woman with moderate persistent asthma who is using medium-dose inhaled fluticasone daily and albuterol via MDI as needed. In a typical month, she uses albuterol 2 puffs "about 2 times, when I feel my chest getting a little tight, and it works right away." In an average month, she has no episodes of nocturnal awakening with cough or wheeze, typically has excellent activity tolerance. An oral systemic bronchodilator titrated to therapeutic level A short course of oral corticosteroid An oral macrolide antimicrobial A single dose of injectable corticosteroid

Ans: A short course of oral corticosteroid

Mrs. Perry presents her 19-month-old for evaluation of a 2-day history of crankiness and fever. Physical examination is significant for a sustained fever (up to 103.4ᵒF or 39.7ᵒC). The tympanic membranes are erythematous around the edges but otherwise normal. The chest is clear to auscultation and her respiratory rate is within normal limits. The child is alert and age-appropriately resists examination. The remainder of the physical examination is normal. Initial assessment should include: A chest radiograph. A lumbar puncture. A sterile urine specimen for culture and sensitivity. Stool for ova, parasites, and culture.

Ans: A sterile urine specimen for culture and sensitivity.

During a well neonate visit for Christopher, a healthy 2-week-old boy born at 41 weeks' gestation, you anticipate that the baby will have: A visual preference for the human face. Hear low-pitched voices best. Will not react to the cry of other neonates. A poorly-developed sense of smell.

Ans: A visual preference for the human face.

All of the following diagnostic studies are available as part of Mr. Lam's evaluation. Rank from 1 (most helpful) to 3 (least helpful) as these tests pertain to the evaluation of Mr. Lam's symptoms. Based on the scenario below: Mr. Lam, a 78-year-old man with longstanding GERD, presents with a 1-month history of "feeling like the food gets stuck way down in my throat." This sensation occurs with meats and other solid food types and less likely with softer or liquid foods. He denies nausea, vomiting, constipation, diarrhea, aspiration, or melena. His physical examination is unremarkable. A hemogram today reveals a microcytic hypochromic anemia with an elevated RDW. A. Barium swallow __ B. Upper endoscopy __ C. Abdominal ultrasound __ A-2 B-1 C-3 A-1 B-2 C-3 A-3 B-1 C-2 A-2 B-3 C-1

Ans: A-2 B-1 C-3

Adolescent issues: By high school senior year (Match the following). 20% 40% 50% 70% A. Tried alcohol. __ B. Used an illegal/restricted drug including marijuana. __ C. Smoked a cigarette. __ D. Used a prescription drug for a nonmedical purpose. __ A-50% B-70% C-40% D-20% A-70% B-40% C-50% D-20% A-40% B-50% C-20% D-70% A-70% B-50% C-40% D-20%

Ans: A-70% B-50% C-40% D-20%

Which of the following vaccines should be offered to him today? Choose all that apply. Based on the below scenario: Maria is a G1P0AB0 woman with is 28-weeks pregnant. Her pregnancy to date has been uneventful and she has no chronic health problems. Maria has not received any immunizations in the past year. Maria's 28-year-old spouse, who is generally in good health, also presents today. A. Tdap B. Injectable influenza vaccine C. Pneumococcal conjugate vaccine D. Zoster vaccine

Ans: A. Tdap B. Injectable influenza vaccine

The nurse practitioner is seeing a patient who is starting on an HMG-CoA reductase inhibitor. Laboratory assessment should include an initial measurement of: LDH. ALT. ALP. GGT.

Ans: ALT.

TMP-SMX should be avoided in elders who are being concomitantly treated with: Moderate- to high-intensity statin therapy ARBs or ACEIs 2nd-generation antihistamine SSRIs

Ans: ARBs orACIs due to increase risks of hypotension and cardiac events

All of the following would support initial testing for type 2 diabetes mellitus in a 12-year-old overweight child except: Family history of type 2 diabetes mellitus. African ethnicity. Absence of signs for start of puberty. Presence of acanthosis nigricans.

Ans: Absence of signs for start of puberty.

Mr. Woods is a 63-year-old man who presents for initial evaluation at your practice. He has a 45 pack-year smoking history and COPD that is currently being treated with daily oral theophylline and albuterol via MDI prn. He complains of shortness of breath and chronic cough with sputum production. He states, "I can barely walk to my car without getting out of breath." Spirometric assessment reveals an FEV1 of 45% predicted and a FEV1:FVC of 62%. His health history reveals 3 exacerbations in the past year. The best course of action for Mr. Woods to minimize COPD exacerbation risk is to: Continue on theophylline therapy. Advise using albuterol on a set schedule rather than prn. Switch to a long-acting beta2-agonist as needed for shortness of breath. Add therapy with a long-acting muscarinic antagonist uses on a set schedule.

Ans: Add therapy with a long-acting muscarinic antagonist uses on a set schedule.

36-year-old man with asthma has been treating his asthma with albuterol once or twice a week for the last few years. He reports that for the last month or so he has had to use it much more often—"not every day, but almost." The nurse practitioner should consider all of the following except: Administering the Asthma Control Test questionnaire. Spirometric assessment. A complete symptom assessment. Adding an inhaled long-acting beta2-agonist.

Ans: Adding an inhaled long-acting beta2-agonist.

Which of the following best demonstrates the intent of informed consent? Discussing the risks of surgery with the parent of a 20-year-old Advising a 48-year-old woman about the likely course of illness if no treatment is rendered Conferring with the husband of a 30-year-old woman who has pneumonia about the benefits of antimicrobial therapy Recommending a course of treatment to a 70-year-old man with prostate cancer

Ans: Advising a 48-year-old woman about the likely course of illness if no treatment is rendered

A 52-year-old man presents for evaluation of fatigue. His physical examination is significant only for intention tremor and asymptomatic pharyngeal redness. He reports a history of recurrent gastritis. Laboratory examination results are as follows: -Hb=14.3 g/dL (143 g/L) -Hct=48% (.48 proportion) -MCV=109 fL -HDL-C=58 mg/dL (1.5 mmol/L) -LDL-C=118 mg/dL (3.06 mmol/L) -Triglycerides=318 mg/dL (3.59 mmol/L) These findings are most consistent with: Pernicious anemia. Iron deficiency anemia. Alcohol abuse. Normal findings.

Ans: Alcohol abuse.

Treatment of atopic dermatitis present on the cheeks of an otherwise healthy 4-month-old infant can include all of the following except: Moisturizing cream to hydrate the skin. Antihistamines to reduce itching. Alcohol-based lotion to prevent infection. Topical corticosteroid to control flares.

Ans: Alcohol-based lotion to prevent infection.

A 14-month-old girl is brought in for evaluation by her mother. She reports that her daughter has been fussy for the past 3 days and tugging at her left ear. Evaluation reveals moderate bulging of the tympanic membrane and her temperature is 102.6ºF (39.2ºC). The child has no reported medication allergies and has not received any antimicrobials in the past 3 months. You recommend: Watchful waiting and follow-up in 3 days. Amoxicillin (Amoxil®). Clarithromycin (Biaxin®). Levofloxacin (Levaquin®).

Ans: Amoxicillin (Amoxil®).

Jared is a 17-year-old with no known medication allergy who has suspected infectious mononucleosis. He is febrile and complains of acute otalgia on the left for the past three days. Physical examination reveals a left tympanic membrane that is red and bulging. When considering therapy for Jared, which of the following should not be prescribed? Acetaminophen Ibuprofen Amoxicillin Azithromycin

Ans: Amoxicillin (secondary reaction due to Mononucleosis infection)

A woman brings in her 23 month-old grandson who has been fussy for the past two days with frequent holding of both ears and crying for the past 2 days. Evaluation reveals bulging and erythema of both tympanic membranes and a temperature of 100.4ºF (38.0ºC). The boy has no reported allergy to penicillin and is otherwise healthy. The most appropriate course of action is: Analgesics and watchful waiting. Amoxicillin. Cefpodoxime. Azithromycin.

Ans: Amoxicillin.

Which of the following patients with acne is the best candidate for oral isotretinoin (Accutane®)? A 15-year-old with 10-15 comedones and has not responded adequately to benzoyl peroxide. A 17-year-old with type 1 diabetes and a total lesion count of 80-100 and has a sulfa allergy An 18-year-old with 5-10 nodules and ~60 inflammatory lesions who has not responded to multiple prior therapies A 19-year-old woman with severe acne (>125 total lesions) who does not wish to initiate contraceptive therapy

Ans: An 18-year-old with 5-10 nodules and ~60 inflammatory lesions who has not responded to multiple prior therapies

Who among the following patients is at the greatest risk for suicide? A 95-year-old female whose only child died last week An 85-year-old male who lives alone A 15-year-old female who has been using marijuana A 12-year-old male who is questioning his gender identity

Ans: An 85-year-old male who lives alone

A researcher wishes to evaluate the effectiveness of Antimicrobial Z for complicated skin infections by analyzing patient charts over the past 3 years. This is generally considered what type of study? Double-blind Single-blind Prospective Retrospective

Ans: Retrospective

Which of the following patients should have screening for HPV-associated cancer? An annual Pap test for cervical cancer screening in a 21-year-old woman A cervical Pap test every 3 years for a 40-year-old woman who has undergone a total hysterectomy for uterine fibroids An anal Pap in a 45-year-old man with a history of anal and genital warts An anal Pap in a 21-year-old woman who has received HPV-9 vaccine

Ans: An anal Pap in a 45-year-old man with a history of anal and genital warts

A mother who is breastfeeding her 2-day-old daughter expresses concern because the child makes a "clicking sound" when nursing. On physical examination of the baby, the NP notices that she is able to thrust her tongue beyond the lip margins with ease. The NP explains that the noise is most likely due to: An incomplete latch. Immature motor control. Ankyloglossia. An occult cleft palate.

Ans: An incomplete latch.

Adam, a 7-year-old boy with a prior diagnosis of moderate persistent asthma, presents with his parents for a well-child visit. He is new to your practice and has not had a healthcare visit in the past year. Adam's mom mentions that he took a "pill to control his breathing, but we ran out." "Right now, he uses the albuterol pump once or twice a day. This keeps his cough under pretty good control. The inhaler works quickly most of the time." You advise the following: Add a twice-a-day long-acting beta2-agonist as needed to ensure Adam has better cough control. An inhaled corticosteroid should be added to Adam's treatment regimen. A leukotriene modifier is an acceptable first-line controller medication for Adam. No additional medication is needed as Adam has adequate symptom control with the current albuterol dose and frequency.

Ans: An inhaled corticosteroid should be added to Adam's treatment regimen.

One of the preferred pharmacologic options of persistent cough in an otherwise well 25-year-old woman with acute uncomplicated bronchitis would most likely include: Antibiotic therapy to cover atypical pathogens. An inhaled short-acting muscarinic antagonist. An inhaled corticosteroid. A dextromethorphan-based cough suppressant.

Ans: An inhaled short-acting muscarinic antagonist.

Given Katie's clinical presentation, you prescribe which of the following? Choose all that apply. Based on the Scenario Below: Katie is a 33-year-old woman who reports a 10-year history of unilateral, pulsing headache that lasts about 6-10 hours, occurring 3?4 times per month. The headache is typically preceded by a gradual onset of paresthesia affecting the ipsilateral face and arm, which lasts about 20 minutes. She reports severe photophobia and phonophobia as well as left-sided cephalgia during the 6-10 h headache duration. She states the headaches appear randomly. As a result of her headaches, Katie typically needs to either call in sick or leave work early at least once a month due to headache. Katie has used OTC medications with partial relief of pain but continued photo-and phonophobia. She is currently headache-free and neurological exam is within normal limits. An oral triptan Migraine prophylactic therapy An oral NSAID A short course of a systemic corticosteroid

Ans: An oral triptan, Migraine prophylactic therapy An oral NSAID

A 41-year-old woman was recently diagnosed with systemic lupus erythematous. She complains of feeling generally fatigued and sometimes palpitations. Her hemogram results are as follows: -Hb=9.5 g/dL (95 g/L) -Hct=28% (.28 proportion) -MCV=86 fL -RDW= 12.5% -Reticulocytes=0.9% (.09 proportion) These findings are most consistent with: Iron deficiency anemia. Anemia of chronic disease. Pernicious anemia. Alpha thalassemia minor.

Ans: Anemia of chronic disease.

Ms. Hannigan is a 70-year-old woman with well-controlled hypertension, dyslipidemia, and hypothyroidism. Her current medications include a statin, ACE inhibitor, a thiazide diuretic, and levothyroxine replacement. She presents today with a chief complaint of a 2-month history of fatigue and aching sensation accompanied by morning stiffness in her hips and shoulders. During this time, she has lost 8 lb (3.6 kg) "without even trying" and reports struggling to get dressed, especially when putting on a shirt or pants. Physical exam reveals full limb strength, decreased active range of motion in the hips and shoulders, cool, smooth joints, and no excessive muscle tenderness. Laboratory test results include: Hb=10.8 g/dL (108 g/L) Hct=32% (0.32 proportion) MCV=86 fL RDW=12.2% (0.122 proportion) ESR=112 mm/h (ULN=20 mm/h) The result of her hemogram reveals: Iron deficiency anemia. Pernicious anemia. Anemia of chronic disease. Age-relative normative findings.

Ans: Anemia of chronic disease.

A 40-year-old man complains of a recurrent dry cough. He is not a smoker, has no airway disease, and his history is significant only for recently diagnosed hypertension. He admits having just started taking medication for his high blood pressure, but doesn't know what it is called. The nurse practitioner suspects that he is most likely taking a(n): Angiotensin-converting enzyme (ACE) inhibitor. Calcium channel antagonist. Angiotensin receptor blocker. Beta-adrenergic antagonist.

Ans: Angiotensin-converting enzyme (ACE) inhibitor.

When choosing pharmacologic intervention to prevent recurrence of duodenal ulcer, you prescribe: A proton pump inhibitor Timed antacid use Antimicrobial therapy A H2-receptor antagonist

Ans: Antimicrobial therapy

The mother of a 12-month-old son is expected to bring him to clinic for vaccinations later that day. She asks if she can give him some ibuprofen prior to the visit to minimize pain and fever from the vaccinations. The NP responds: Acetaminophen is preferred over ibuprofen. Ibuprofen should be administered about 1 hour prior to the visit. Half a dose can be given before the visit and half immediately following the visit. Antipyretic use is not recommended prior to vaccinations.

Ans: Antipyretic use is not recommended prior to vaccinations.

A 12-year-old boy presents with his mother for a well-child visit. What is the most helpful approach to this visit? Interview and examine the child in the absence of the mother. Interview the child with the mother, asking her to leave for the examination. Ask the child if he wishes his mother to be there for the interview and examination. Ask the mother if she wishes to be included in the interview and examination

Ans: Ask the child if he wishes his mother to be there for the interview and examination.

A 50-year-old man of Croatian ancestry has a follow-up appointment after cardiac bypass surgery. The patient brings his father with him into the examination room. They are both fluent English speakers. The NP provides culturally sensitive care by the following action. Ask the patient's father if he has any questions regarding his son's care. Request the patient's father to leave the room due to confidentiality issues. Acknowledge the father's presence and complete the visit while reporting all findings to the patient. Perform the clinical evaluation, then report to the patient's father the examination findings.

Ans: Ask the patient's father if he has any questions regarding his son's care.

The nurse practitioner knows that all patients with an acute eye complaint should have: Dilated pupil examination. Assessment of visual acuity. Everted lid examination. Fluorescein staining.

Ans: Assessment of visual acuity.

When initiating pharmacologic treatment for osteoporosis in women, the American College of Physicians recommends bone mineral density monitoring: Annually. At 6 months following initiation of treatment, then annually thereafter. Every 2 years. At the completion of therapy.

Ans: At the completion of therapy

Clinical evaluation of a 43-year-old man with a BMI=42 kg/m2 who presents with recurrent genital Candida albicans infection should include all of the following except: Treatment with oral or topical antifungals. Offering testing for HIV antibodies. Avoidance of soap on the genitals. Checking A1C.

Ans: Avoidance of soap on the genitals.

Intervention for Josh should include information about: Based on the below scenario: Josh is a 14-year-old basketball player who presents with anterior knee pain that has occurred intermittently over the past 3 months. The pain worsens with squatting and walking up or down stairs, and is better with rest. He denies fever, weight loss, joint redness, or skin rash. Physical examination reveals a Tanner 3 male in no acute distress with a tender, swollen tibial tuberosity in the affected knee. Pain can be reproduced with resisted active extension and passive hyperflexion of the knee. No effusion is present. Curtailing his participation in sporting activities. Avoiding sports that involve heavy quadriceps loading or deep knee bending. The benefit of intraarticular corticosteroid injection for the control of discomfort. The likely need for surgical correction of the defect.

Ans: Avoiding sports that involve heavy quadriceps loading or deep knee bending.

Treatment options for uncomplicated Chlamydiatrachomatis infection in pregnancy include: Clarithromycin. Doxycycline. Azithromycin. Ofloxacin.

Ans: Azithromycin

First-line therapy for the treatment of community-acquired pneumonia in a 5-year-old child with a presumed atypical pathogen is: Amoxicillin (Amoxil®). Azithromycin (Zithromax®). Oseltamivir (Tamiflu®). Cefpodoxime (Vantin®).

Ans: Azithromycin (Zithromax®).

A 15-week-old infant presents with a fever of 100.5ᵒF (38.1ᵒC) and bilateral erythematous tympanic membranes. The infant is alert with excellent skin turgor, no evidence of difficulty breathing, acknowledges her mother's face, and is wearing a wet diaper. Her parents report that she is vigorously nursing every 3 hours without vomiting or excessive stooling. The appropriate management would be to: Counsel the mother to observe for 72 hours and return to clinic if there is no improvement. Start topical therapy with otic antibiotic drops. Initiate a sepsis workup. Begin a systemic antibiotic regimen.

Ans: Begin a systemic antibiotic regimen.

While walking in the parking lot of a grocery store, an NP notices an elderly man fall and hit his head on the ground. The NP quickly rushes to provide aid to the man. This is an example of: Beneficence Autonomy Veracity Utilitarianism

Ans: Beneficence

The NP considers that which of the following dietary components is the least likely to trigger migraine. Sourdough bread Cheddar cheese Pickled herring Ricotta cheese

Ans: Ricotta cheese

A patient with newly-diagnosed type 2 diabetes mellitus who has an A1C=8.2% will most likely begin therapy with a medication from which of the following drug classes? GLP-1 agonist such as exenatide DPP-4 inhibitor such as saxagliptin Thiazolidinedione such as pioglitazone Biguanide such as metformin

Ans: Biguanide such as metformin

A 21 year old woman with normal BMI prior to pregnancy is now 38-weeks pregnant. Her weight gain up to 28 weeks was 12 lbs (5.45 kg) and she has gained 2 lbs (0.9 kg) from 29 weeks to 38 weeks. Her weight gain pattern is most likely to affect her baby's: Birth weight. Head circumference. Birth length. Apgar scores.

Ans: Birth weight

Which of the following is a potential serious complication of Mrs. Jensen's presumptive diagnosis? Based on the scenario below: Mrs. Jensen is an 82-year-old woman with generalized osteoarthritis and systolic hypertension who presents with a 3-day history of right-sided headache with accompanying right-sided jaw pain on chewing. OTC analgesics have provided little relief. She states she is eating little due to the pain on chewing but is taking liquids without difficulty. Mrs. Jensen reports "I can hardly wash my hair, my scalp is so sore." Physical examination reveals: BP=168/88 mm Hg bilateral, P=88 regular, RR=18, alert, appears uncomfortable with poorly-groomed hair on the right only PERLA, cranial nerve function II‒XII intact, remaining neurological exam WNL, carotid upstroke within normal limits without bruit. There is a tender, palpable, pulseless structure in the right temple area. Hemiparesis Arthritis Blindness Anterior uveitis

Ans: Blindness

Which of the following two tests should be periodically checked while on olanzapine (Zyprexa®)? BUN and creatinine Creatine kinase and potassium Complete blood count and platelet count Blood sugar and lipid profile

Ans: Blood sugar and lipid profile

Teresa, a 38-year-old woman with no chronic health problems, presents with a chief complaint of "A cold I cannot shake for the past three weeks. She also reports an intermittent frontal headache and has taken acetaminophen with relief, as well as general malaise and a dry cough that is particularly problematic at night. She denies nausea, vomiting, chills, fever, or dyspnea. She has not taken an antimicrobial within the past year, underwent a bilateral tubal ligation approximately 10 years ago, and is allergic to penicillin with a hive-form reaction. Examination reveals the following: SaO2=97%, BP=114/70 mm Hg, T=98°F (36.7°C), HR=88, RR=20, bilateral coarse late inspiratory crackles without wheeze. She is in no acute distress. Chest x-ray demonstrates bilateral interstitial infiltrates. Which of the following describes the recommended additional diagnostic testing for Teresa? Choose all that apply. Blood urea nitrogen (BUN) and creatinine (Cr) CBC with WBC differential Blood culture Sputum culture

Ans: Blood urea nitrogen (BUN) and creatinine (Cr), CBC with WBC differential

The most helpful imaging study in the evaluation of appendicitis for Jordan is an abdominal: Based on the scenario below: Jordan, an otherwise well 24-year-old man with a BMI=32 kg/m2 and acute appendicitis. Flat plate CT with contrast MRI Radionuclide scan

Ans: CT with contrast

When initiating treatment with an antihypertensive medication for a 72-year-old male with mild renal impairment, you consider all of the following except: Sodium depletion can result with the use of a thiazide diuretic. An increased risk of hyperkalemia is associated with ARB use. The risk of hyperkalemia with the use of an aldosterone antagonist is increased when combined with an ACE inhibitor. Calcium channel blockers are appropriate to use in the presence of heart failure.

Ans: Calcium channel blockers are appropriate to use in the presence of heart failure.

Absorption of which of the following is least affected by long-term PPI use? Iron Calcium citrate Vitamin B12 Magnesium

Ans: Calcium citrate

You see a 54-year-old woman, Senora Sanchez, who is not fluent in speaking English, who comes into the emergency department of a 300-bed hospital with a chief complaint of abdominal pain and is accompanied by Tomas, her 10-year-old nephew. Tomas states, "I am here to help my aunt since she does not speak English." Which of the following is the most appropriate way in conducting this clinical encounter? Direct questions to Tomas, asking him to translate the information to his aunt. Direct questions to Senora Sanchez, asking Tomas to translate. Advise that Senora Sanchez return with an adult interpreter. Call the hospital's interpreter services department for assistance.

Ans: Call the hospital's interpreter services department for assistance.

When counseling parents on 'tummy time' for a newborn, the NP mentions that tummy time should be encouraged until the infant: Is able to sit solo. Can pull up to a standing position. Can roll from tummy-to-back and back-to-tummy with ease. Can crawl for at least 5 feet.

Ans: Can roll from tummy-to-back and back-to-tummy with ease.

The relationship between obesity and type 2 diabetes is best described as: Causal Correlational Indirect Qualitative

Ans: Causal

A father brings in his 7-year-old daughter for evaluation. She has experienced moderate pain in both ears for the past 2 days and has a fever (103.1ºF [39.5ºC]). Examination reveals bilateral moderate bulging tympanic membranes. She has a history of penicillin allergy. You prescribe therapy with: Analgesics only and follow-up in 2 days. Amoxicillin-clavulanate (Augmentin®). Cefuroxime axetil (Ceftin®). Ciprofloxacin (Cipro®).

Ans: Cefuroxime axetil (Ceftin®).

A characteristic feature of erythema migrans is: Blanching to pressure. Central clearing. Rapid response to topical corticosteroids. Intense pruritus.

Ans: Central clearing.

When a pregnant woman takes a teratogenic drug, the fetal effects are usually seen in: Certain target organs in a predictable manner. Random body systems without a predictable pattern. Select organ systems with random outcomes. A pattern of systemic injury.

Ans: Certain target organs in a predictable manner.

A 12-day-old infant who is otherwise well presents with a 2-day history of irritation of both eyes. He was born at a local birth center and received standard newborn care including ocular chemoprophylaxis. Examination reveals bilateral lid swelling, chemosis, and mucoid eye discharge. The most likely cause of this condition is: Chemical irritation from neonatal ocular chemoprophylaxis. Chlamydial (inclusion) conjunctivitis. Gonococcal conjunctivitis. Neonatal adenovirus infection.

Ans: Chlamydial (inclusion) conjunctivitis.

Janet, a 47-year-old well woman who is 64" (163 cm) tall with BMI=25 kg/m2, presents for a periodic health evaluation and to establish care in your practice. She reports drinking approximately 1 to 2 mixed drinks containing 1.5 oz. (44.4 mL) 80-proof liquor per week and takes approximately 1 to 2 doses of acetaminophen per month. Her last visit with a healthcare provider was more than 5 years ago; she was told that her "liver tests were a bit higher than normal." She felt well and decided not to return for follow-up. She denies history of injection drug use and has no tattoos. Her last vaccines were administered more than 10 years ago. She is without complaint. Abdominal examination is within normal limits. Janet's laboratory results are as follows. HBsAg=positive (evidence HBV on board, Ag=Always growing, HBV persists, in acute or chronic hepatitis B) Anti-HAV=positive (immune to HAV, include HAV IgG, G=Gone) Anti-HCV=negative (no evidence of past or present HCV infection) AST=56 unit/L (0-40) (modest elevation, with ALT>AST) ALT=98 unit/L (0-40) (modest elevation, with ALT>AST) These findings are most consistent with which of the following? Choose two that apply: Chronic hepatitis B Evidence of hepatitis B infection immunity Evidence of hepatitis A immunity Acute hepatitis B infection Evidence of hepatitis C infection

Ans: Chronic hepatitis B, Evidence of hepatitis A immunity

What type of professional liability insurance policy covers you only if the injury occurs within the policy period and the claim is filed during the period the policy is in effect? Claims-made Occurrence Interval coverage Incident petition

Ans: Claims-made

Managed care is best defined as a: Collaborative approach to provide coordination of the healthcare services to a defined population. System for determining what healthcare services are reimbursable. Process of reviewing the appropriateness of select healthcare interventions. Method of review to ensure adherence to evidence-based practice.

Ans: Collaborative approach to provide coordination of the healthcare services to a defined population.

Katie requests advice on family planning. She has not been pregnant in the past. Given her history, which of the following is the least desirable contraceptive form for Katie? Based on the Scenario Below: Katie is a 33-year-old woman who reports a 10-year history of unilateral, pulsing headache that lasts about 6-10 hours, occurring 3?4 times per month. The headache is typically preceded by a gradual onset of paresthesia affecting the ipsilateral face and arm, which lasts about 20 minutes. She reports severe photophobia and phonophobia as well as left-sided cephalgia during the 6-10 h headache duration. She states the headaches appear randomly. As a result of her headaches, Katie typically needs to either call in sick or leave work early at least once a month due to headache. Katie has used OTC medications with partial relief of pain but continued photo-and phonophobia. She is currently headache-free and neurological exam is within normal limits. Levonorgestrel IUD (Skyla®, Lilleta®, Mirena®) Copper IUD (ParaGard®). Combined estrogen-progestin oral contraceptive. A progestin-only implant (Nexplanon®).

Ans: Combined estrogen-progestin oral contraceptive.

A 23-year-old- woman is breastfeeding her healthy newborn. She wishes to use hormonal contraception. Which of the following represents an unacceptable choice? Combined oral contraception initiated at 2 weeks Progestin-only oral contraception initiated at 3 weeks Medroxyprogesterone acetate (Depo-Provera®) given day 1 postpartum CU-containing IUD (ParaGard®)

Ans: Combined oral contraceptives initiated at 2 weeks.

When initiating pharmacologic therapy for the patient with first diagnosis of major depression disorder, the nurse practitioner advises the patient that treatment should: Generally be given for about 4-6 months before improvement is noted. Continue for at least 6 months after remission is achieved. Be continued indefinitely after therapeutic goals are met. Be started at a higher dose then titrated to a lower dose once symptom relief is achieved.

Ans: Continue for at least 6 months after remission is achieved.

A mother brings in Jackson, her 4-year-old son, for evaluation after reporting that she stayed up all night with him because he was experiencing somewhat labored breathing and had a loud "barking-like" cough. The child has a hoarse voice, is without acute respiratory distress, and a temperature of 101.6ᵒF (38.7ᵒC). The most likely diagnosis is: Upper airway foreign body. Croup. Asthma. Bronchiolitis.

Ans: Croup.

A mother presents her 3-year-old son with a 2-day history of cough and runny nose without fever. The NP explains that this is likely a viral infection and treatment should focus on symptom management. However, the mother is insistent on getting a prescription for antibiotics despite the NP's explanation that antibiotics will have limited effectiveness and may cause adverse effects. In this scenario, which of the following best demonstrates the ethical principle of nonmaleficence? Providing a prescription for an antibiotic regimen. Providing a 2-day sample of antibiotics with follow-up at 3 days to determine if additional antibiotics are needed. Denying a prescription for antibiotics. Getting approval from a physician prior to giving the antibiotic prescription

Ans: Denying a prescription for antibiotics.

Ms. Hopkins is a 78-year-old woman with a history of hypertension, dyslipidemia, and stable angina pectoris who presents for follow-up care. She is a former smoker who quit 25 years ago with a 40 pack-year history. She denies chest pain or shortness of breath. Her current medications include a beta-adrenergic antagonist, sustained-release nitrate, angiotensin-converting enzyme inhibitor, statin, low-dose aspirin, and a thiazide diuretic. She complains of feeling a "catch or cramp" in her lower posterior legs bilaterally when she walks for extended periods. This is promptly relieved by rest. Which of the following do you anticipate finding on examination of her lower extremities? Hyperpigmentation with bilateral ankle edema Diminished bilateral pedal pulses with thinning of the skin Extensive dry skin with evidence of lichenification on the plantar aspects of both feet Diminished sensory perceptions and abnormal monofilament examination

Ans: Diminished bilateral pedal pulses with thinning of the skin

When examining a 1-day-old boy who was born at 39-weeks' gestation, the NP notes a flat facial profile, hypotonia, hyperflexible joints, a single palmar crease in both hands, and Brushfield spots. This is most suggestive of: Cerebral palsy. Down syndrome (Trisomy 21). Edwards syndrome (Trisomy 18). Fragile X syndrome.

Ans: Down syndrome (Trisomy 21).

An appropriate treatment option for community-acquired pneumonia in a 45-year-old woman with no significant comorbidities, no recent history of antimicrobial use (>3 months), and no known drug allergies is: Ampicillin. Doxycycline (Doryx®). Cefuroxime (Ceftin®). Moxifloxacin (Avelox®).

Ans: Doxycycline (Doryx®).

The nurse practitioner is evaluating a 34-year-old male who reports a 3-month history of epigastric pain that he describes as intermittent, burning, and improves with eating then returns 2-3 hours after meals. The leading diagnosis is: Cholecystitis. Gastritis. Esophageal reflux. Duodenal ulcer.

Ans: Duodenal ulcer.

The NP is counseling a 67-year-old patient with severe COPD who is considering the use of long-term oxygen therapy. The patient is currently taking an inhaled corticosteroid (ICS) with a long-acting beta2-agonist (LABA). The NP advises that: Oxygen therapy can be used in place of the ICS Effective treatment will require at least 4‒6 hours of therapy per day Effective treatment will require at least 15 hours of therapy per day Consistent use of oxygen therapy can improve FEV1 to near normal values

Ans: Effective treatment will require at least 15 hours of therapy per day

Reasonable goals for using headache prophylactic therapy include all of the following except: Reduce the frequency of headaches Reduce the severity of headaches Eliminate the incidence of headaches Allow headache medications to work more effectively

Ans: Eliminate the incidence of headaches

Which information concerning the use of email messages is included in the Health Insurance Portability and Accountability Act? Email messages are not an acceptable form of communication under any circumstances. Email messages are suitable only if this communication contains medical record numbers, instead of patient names. Email messages are acceptable between healthcare providers, but not between healthcare providers and patients. Email messages containing private patient information are acceptable if the information is encrypted.

Ans: Email messages containing private patient information are acceptable if the information is encrypted.

n considering jaundice in the neonate, the NP considers that: Neonatal jaundice usually starts first in the trunk and extremities and then progresses to the face. Using visual evaluation of bilirubin levels to estimate the degree of jaundice is an acceptable way to evaluate the newborn. The onset of physiologic jaundice is usually within the first 12 hours of life. Encouraging feedings at minimum every 2?3 hours at the breast per day while avoiding dextrose and water feedings will help minimize the newborn's risk of hyperbilirubinemia.

Ans: Encouraging feedings at minimum every 2?3 hours at the breast per day while avoiding dextrose and water feedings will help minimize the newborn's risk of hyperbilirubinemia.

You see a 10-year-old boy who is obese and has elevated triglycerides. You recommend: Initiating treatment with a statin. Initiating treatment with a fibrate. Initiating treatment with fish oil. Encouraging weight loss.

Ans: Encouraging weight loss.

The mechanism of action of saxagliptin includes: Increase in hepatic glucose utilization. Enhanced insulin release. Facilitation of renal glucose excretion. Enhancement of insulin receptor site activity.

Ans: Enhanced insulin release. Metformin - Biguanide - 1st line therapy- insulin sensitizer; neutral weight loss. Complication: can cause GI symptoms (diarrhea) or renal acidosis; monitor renal function. Glipizide, glipizide..- Sulfonuleas class- constant insulin release- causes weight gain, high risk of hypoglycemia, monitor in adult patients. Sitagliptin- DDP-4 (-gliptins) - insulin release post glucose rise in the body- neutral weight control- rare side effects Pioglitazones (TZDs)- insulin sensitizers- modest gain weight, avoid use in CHF causing edema, increase risk of fractures. Exenatide (SQ Injection)- insulin release post glucose rise- GI symptoms Canagliflozin - SGLT2- glucose excretion (through urine) post glucose rise- modest weight loss, increase chance of urethral/vaginal yeast infection, dehydration or fractures, monitor renal function.

All of the following are characteristics of knee osteoarthritis except: Limited range of motion. Morning stiffness. Erythema surrounding the joint. Crepitus.

Ans: Erythema surrounding the joint.

Which of the following represents the best choice of initial test to support Mrs. Jensen's presumptive diagnosis? Based on the scenario below: Mrs. Jensen is an 82-year-old woman with generalized osteoarthritis and systolic hypertension who presents with a 3-day history of right-sided headache with accompanying right-sided jaw pain on chewing. OTC analgesics have provided little relief. She states she is eating little due to the pain on chewing but is taking liquids without difficulty. Mrs. Jensen reports "I can hardly wash my hair, my scalp is so sore." Physical examination reveals: BP=168/88 mm Hg bilateral, P=88 regular, RR=18, alert, appears uncomfortable with poorly-groomed hair on the right only PERLA, cranial nerve function II‒XII intact, remaining neurological exam WNL, carotid upstroke within normal limits without bruit. There is a tender, palpable, pulseless structure in the right temple area. Erythrocyte sedimentation rate Enhanced contrast brain MRI Head CT without contrast Arterial biopsy

Ans: Erythrocyte sedimentation rate

Gastroparesis is a contraindication to the use of which of the following medications in type 2 diabetes mellitus? Exenatide (Byetta®) Pioglitazone (Actos®) Sitagliptin (Januvia®) Glyburide (Glynase®)

Ans: Exenatide (Byetta®)

Mr. Miller is a 60-year-old man with hypertension. On today's office visit, his blood pressure is noted to be 122/78. The rest of his history and physical examination are unremarkable. He mentions that, in addition to his antihypertensive medication you have prescribed, he uses a teaspoon of hot pepper sauce in a glass of warm water daily to help "clean my blood and lower the pressure. I feel much better when I take it." Your most appropriate response is to: Advise Mr. Miller to stop using the hot pepper sauce. Explore what Mr. Miller means by "clean my blood." Inform Mr. Miller that this practice can result in serious gastrointestinal complications. Ask Mr. Miller to stop the hot pepper sauce use until you can examine the contents of the product he uses.

Ans: Explore what Mr. Miller means by "clean my blood."

A 63-year-old patient presents for evaluation of fever. She reports that 3 days ago she went to the urgent care and was diagnosed with acute bacterial rhinosinusitis (ABRS). She was given a prescription for antibiotics which she is taking as ordered, but is concerned because she has a fever today. Which of the following findings suggests a serious complication of ABRS? Tender preauricular lymph nodes Foul-tasting post nasal drip Facial tenderness Eyelid erythema and edema

Ans: Eyelid erythema and edema

One site for atopic dermatitis usually only noted during infancy is the: Face. Antecubital fossa. Buttocks. Hands.

Ans: Face.

Billing and coding are basically the same thing. True False

Ans: False

Once I have achieved national certification, I can practice as a nurse practitioner. True False

Ans: False

The NP certification exam will contain a number of questions about my state's NP practice act. True False

Ans: False

You are seeing 17-year-old Cynthia. As part of the visit, you consider her risk factors for type 2 diabetes mellitus would likely include all of the following except: Obesity. Pacific Islander ancestry. Family history of type 1 diabetes mellitus. Personal history of polycystic ovary syndrome (PCOS).

Ans: Family history of type 1 diabetes mellitus.

Treatment with a second-generation antipsychotic (SGA) is being initiated for a 37-year-old woman with schizophrenia who is otherwise healthy. Following initiation of the SGA, the NP understands that regular monitoring should be performed for which of the following? AST/ALT Fasting blood glucose Hct TSH

Ans: Fasting blood glucose due to weight gain

Which of the following medications would be ineffective in treating Joe's episode of acute gouty arthritis? Based on the below scenario: Joe is a 54-year-old man with COPD who presents with an 18-hour history of sudden-onset monoarticular pain consistent with acute gouty arthritis. He denies trauma to the area and has taken acetaminophen 1 g × 2 doses with little effect. Intraarticular corticosteroid injection Naproxen sodium (Aleve®, Anaprox®) Febuxostat (Uloric®) Colchicine (Colcrys®)

Ans: Febuxostat (Uloric®)

Ms. Yancy is a 56-year-old woman with a history of depression and has been taking citalopram for about 6 months. Four days ago, she decided to discontinue the citalopram as she states her depressive symptoms have been much improved. Today, she presents with "not feeling well," with a 2-day history of persistent frontal headache. When assessing Ms. Yancy for additional findings of antidepressant discontinuation syndrome, the NP anticipates that Ms.Yancy will report all of the following except: Nausea. Fever. Flu-like symptoms. Anxiety.

Ans: Fever.

A mother presents her 5-year-old son who has a 3-day history of a reticular-form pink-red rash originating on his face that is spreading to his trunk and extremities. The mother reports that he had a mild fever and headache as well but is without GI symptoms other than slightly reduced appetite. A number of other children in his preschool have similar signs and symptoms. The most likely diagnosis for this patient is: Croup. Measles. Roseola. Fifth's disease.

Ans: Fifth's disease.

A 56-year-old woman who is being treated for COPD, hypertension, and dyslipidemia is recently diagnosed with moderate depression and requires antidepressant therapy. Which of the following agents is the least favorable choice due to concerns about potential drug-drug interactions? Citalopram (Celexa®) Escitalopram (Lexapro®) Bupropion (Wellbutrin®) Fluoxetine (Prozac®)

Ans: Fluoxetine (Prozac®)

A 68-year-old woman presents for evaluation. She reports a recent history of fatigue and says she can barely go up the steps without becoming short of breath. Hemogram results are as follows: -Hb=8.7 g/dL (87 g/L) -Hct=27% (.27 proportion) -MCV=70 fL -RDW=18% (.18 proportion) A critical diagnosis to consider is: Vitamin B12 deficiency. Folic acid deficiency. Renal insufficiency. Gastrointestinal blood loss.

Ans: Gastrointestinal blood loss.

Mrs. Jensen is an 82-year-old woman with generalized osteoarthritis and systolic hypertension who presents with a 3-day history of right-sided headache with accompanying right-sided jaw pain on chewing. OTC analgesics have provided little relief. She states she is eating little due to the pain on chewing but is taking liquids without difficulty. Mrs. Jensen reports "I can hardly wash my hair, my scalp is so sore." Physical examination reveals: BP=168/88 mm Hg bilateral, P=88 regular, RR=18, alert, appears uncomfortable with poorly-groomed hair on the right only PERLA, cranial nerve function II‒XII intact, remaining neurological exam WNL, carotid upstroke within normal limits without bruit. There is a tender, palpable, pulseless structure in the right temple area. Mrs. Jensen's clinical presentation is most consistent with: Postherpetic neuralgia. Transient ischemic attack. Giant cell arteritis. Acute venous occlusion.

Ans: Giant cell arteritis.

You are rounding in the nursery and see the neonate of a mother who is HBsAg-positive. Your most appropriate action is to: Check the baby for HBsAb. Inform the mother that she should not breastfeed. Administer hepatitis B immunization to mother and infant. Give hepatitis B immunization and hepatitis B immune globulin to the newborn.

Ans: Give hepatitis B immunization and hepatitis B immune globulin to the newborn.

The nurse practitioner is beginning pharmacologic management of dyslipidemia in a 47-year-old male patient whose lipid panel is as follows: -Total cholesterol=249 mg/dL (6.45 mmol/L) -HDL-C=39 mg/dL (1.01 mmol/L) -TG=279 mg/dL (3.15 mmol/L) -LDL-C=191 mg/dL (4.94 mmol/L) The appropriate class of agent is a(n): Omega-3 fatty acid such as fish oil. Fibric acid derivative such as fenofibrate. Cholesterol absorption inhibitor such as ezetimibe. HMG-CoA reductase inhibitor such as simvastatin.

Ans: HMG-CoA reductase inhibitor such as simvastatin.

You see a 13-year-old boy with no immunizations documented. In updating this child's immunizations, which of the following should be omitted? Hepatitis B Haemophilus influenzae type B (Hib) Tetanus, diphtheria, acellular pertussis (Tdap) Measles, mumps, rubella (MMR)

Ans: Haemophilus influenzae type B (Hib)

Meeting the eligibility requirements for national NP certification signifies that the nurse practitioner: Attained the required academic and clinical skills for specialty practice. Graduated from an accredited nurse practitioner educational program. Has achieved select prerequisites determined by a nongovernmental professional organization. Has permission to practice as a nurse practitioner.

Ans: Has achieved select prerequisites determined by a nongovernmental professional organization.

A 22-year-old man presents for hepatitis screening. He is without symptoms but needs the testing for a job in the food service industry. Laboratory results are as follows: -Anti-HCV with HCV RNA present -HBsAg=positive -Anti-HAV=negative You advise the patient that he: Has chronic hepatitis A, B, and C. Is immune to hepatitis A and B but has hepatitis C. Has chronic hepatitis B and C and needs immunization against hepatitis A. Had hepatitis A in the past, is immune to hepatitis B, and has chronic hepatitis C.

Ans: Has chronic hepatitis B and C and needs immunization against hepatitis A.

When counseling a mother on how to assess if a newborn is nursing adequately, the NP advises that the newborn should: Be able to sleep 2-3 hours between feedings. Have at least 1-2 bowel movements per day. Have 6-8 wet diapers per day. Cease wanting to nurse after about 10 minutes of feeding.

Ans: Have 6-8 wet diapers per day.

You see a 29-year-old woman with a BMI=38 kg/m2 and hyperpigmented plaques with velvet-like appearance at the nape of the neck and axilla (see image). Laboratory assessment should include which of the following tests? Hemoglobin A1C Serum electrolytes HIV antibody screening Hemogram

Ans: Hbg A1C

Which of the following describes the ethical principle of utilitarianism? Healthcare resources are allocated so that the best is done for the greatest number of people. The healthcare provider must be truthful and avoid deception. The healthcare provider has an obligation to be faithful to commitments made to self and others. The responsibility of the healthcare provider is to treat all people in the same fair manner.

Ans: Healthcare resources are allocated so that the best is done for the greatest number of people.

The nurse practitioner is evaluating a patient with recurrent heartburn. When considering gastroesophageal reflux as a differential diagnosis, all of the following symptoms are anticipated except: Eructation. Cough. Dysphonia. Hematemesis.

Ans: Hematemesis.

A 63-year-old man who resides at the homeless shelter is brought to urgent care with a chief complaint of chronic cough and chest pain. His temperature is 102.3°F (39.1°C), respiratory rate of 45 bpm, and blood pressure of 135/85 mm Hg. Which of the following findings would best support a diagnosis of active tuberculosis infection? Erythematous plaques on the extremities Hemoptysis Dry cough with bronchospasm Unexplained weight gain

Ans: Hemoptysis

Thomas is a 45-year-old man who has a past history of injection drug use, currently with 5 years of sobriety. He is unsure about his vaccine history but believes he had "some when I was a child." Results of recent laboratory tests reveal the following. HCV RNA=positive (evidence of current HCV infection) Anti-HBs/HBSab=positive (evidence of HBV immunity, HBSab= B="Bye" as HBV no longer on board) Anti-HAV=negative (no evidence of HAV infection or immunity, negative= "Never had HAV") AST=45 unit/L (0-40) (modest elevation, with ALT>AST)ALT=72 unit/L (0-40) (modest elevation, with ALT>AST) You offer the following immunization(s) against: Hepatitis B Hepatitis A Hepatitis A and B Seasonal influenza, tetanus, diphtheria, and acellular pertussis

Ans: Hepatitis A

You see Sharon for a well-child visit. She is a 12-year-old who is at Tanner stage 2-3 and states unhappily, "I am the shortest girl in my class."When reviewing her growth chart, you notice she has been consistently between the 10th and 15th percentile for height and weight during her childhood. The rest of her examination is within normal limits. You advise that: She should have an evaluation by a pediatric endocrinology specialist. Her growth spurt will start soon. Due to her age, she is likely near her adult height. X-ray determination of bone age should be obtained.

Ans: Her growth spurt will start soon.

Evaluation of a diagnostic test for the presence of rheumatoid arthritis is shown to very reliably provide negative results for those without the disease. The test is said to have: High specificity. High sensitivity. Low specificity, high sensitivity. Low rate of false positives.

Ans: High specificity

The nurse practitioner is evaluating laboratory studies in a patient with newly-diagnosed type 2 diabetes mellitus. Which lipid abnormality is most likely? Low total cholesterol High triglycerides Low LDL-C High HDL-C

Ans: High triglycerides

All of the following are potential causes of secondary hypertriglyceridemia except: Untreated or undertreated hypothyroidism. High-dose ACE inhibitor use. Poorly-controlled DM. Excessive alcohol use.

Ans: High-dose ACE inhibitor use.

You see a Tanner stage 4 14-year-old male who you suspect has Fragile X syndrome because of the notation of all of the following except: Macroorchidism. Large body habitus. History of learning differences. Hip and breast enlargement.

Ans: Hip and breast enlargement. Fragile X-syndrome: In males, large forehead, ears, prominent jaw, tendency to avoid eye contacts, large testicles (macrochidism), noted after beginning after puberty, large body habitus, behavioral and learning differences common. In females, less prominent findings, occurs in all racial ethnic groups, common cause for autism in both genders. Klinefelter syndrome (XXY) only in males, with specific physical habitus, low testicular volume, hip and breast enlargement, infertility most commonly is language development Turner syndrome: (XO) : usually is short stature <5ft, evident by age 5, wide webbed neck broad shield shaped-chest absent menses, infertility, most noted often at birth with high-arched palate, low-set ears, edema of hands and feet,

Which of the following do you not expect to find in the examination of a full-term healthy newborn? Holding the baby about 16-20 inches (41-51 cm) away from the caregiver's face takes advantage of the newborn's visual range. Bluish scleral tint is noted regardless of ethnicity or eye color. The newborn's eyes are quite light and glare sensitive. If an object goes towards the newborn's eye, the baby will likely react with a defensive blink reflex.

Ans: Holding the baby about 16-20 inches (41-51 cm) away from the caregiver's face takes advantage of the newborn's visual range.

You ask a patient to follow your finger, while not moving her head, through the positions of gaze. This is in part an evaluation of cranial nerves: II, III, VI. I, II, III. V, VI, VII. III, IV, VI.

Ans: III, IV, VI. I - oldfactory nerve -smell - SENSORY II - optic (vision) -MOTOR- test for accommodation, visual field, pupillary reaction III- oculomotor nerve - eye move up or down; outward or inward; drooping lid-reflag. MOTOR IV- trochlear nerve- eye movement down and inward - MOTOR V- trigeminal nerve - corneal blink reflex, sensation (wisp of cotton, light vs. dull touch) BOTH VI- Aducens nerve- eye movements (ability to move eyes bilaterally) MOTOR VII- Facial nerve - facial expression, clench teeth, puff cheeks out, production of saliva and tears, raise eyebrown, close eyes against resistance- MOTOR Usually V and VII are tested together. VIII- Acoustic nerve- hearing -Wiber test (lateralization) Rinne test (air vs bone conduction) SENSORY IX- glossaphareangeal - test gag reflex (say Ahh)- BOTH X- Vagus nerve- swallowing or gag reflex, voice - BOTH XI- Spinal accessory - test shoulder movements like shrugging, push up against resistance - MOTOR XII- Hypoglossal - tongue movements (sticks tongue out, move tongue side-to-side) MOTOR

A 45-year-old woman with hypertension and dyslipidemia is currently taking moderate-intensity statin therapy and she is at LDL-reduction goal. She is feeling well. As part of her ongoing healthcare, the NP considers that: A. Serum transaminases should be checked periodically. Evaluation of serum creatine kinase is needed. A CBC with WBC and platelet count is advised. In the absence of symptoms or concerns, no particular laboratory monitoring is recommended

Ans: In the absence of symptoms or concerns, no particular laboratory monitoring is recommended.

he mechanism of action of a GLP-1 agonist includes: Attenuation of insulin resistance. Promotion of renal glucose excretion. Increase in insulin release in response to rising glucose. Decrease in gastric emptying.

Ans: Increase in insulin release in response to rising glucose.

Age-related changes in the elder include all of the following except: Decrease in relative kidney weight Body weight as fat increases for both men and women Increase in serum albumin Decrease in percent body weight as water

Ans: Increase in serum albumin

Jannetta is a 16-year-old who presents with a 3-day history of pharyngitis and fatigue. Findings include exudative pharyngitis, minimally tender anterior and posterior cervical lymphadenopathy, and right and left upper quadrant abdominal tenderness. Per Jannetta's record, she is up-to-date with all recommended vaccinations. This is most consistent with: S. pyogenes pharyngitis. Infectious mononucleosis. Hodgkin disease. Gonococcal pharyngitis.

Ans: Infectious mononucleosis.

The use of which of the following antibiotics is most likely to reduce combined oral contraceptive effectiveness? Amoxicillin Ciprofloxacin Doxycycline Rifampin

Ans: Rifampin is CYP450 3A4 Isoenzymers to break down the substrates of oral contraceptive therefore OCC is ineffective.

Jane is a 27 year-old woman who presents for a "pap test." She is new to your practice, has not significant health history and reports excellent exercise tolerance, running up to 5 miles 5-7 days a week and recently was first place in her age division in a 6K road race. Jane reports taking a low-dose combined oral contraceptive and a daily multivitamin. Physical examination reveals a BMI=22.1 kg/m2, mild pectus excavatum, PMI WNL with both heart sounds intact. There is a midsystolic click with late systolic murmur and the murmur moves forward with position change from supine to stand. The rest of her physical examination is unremarkable. The next most appropriate step in her care is to: Prescribe an antibiotic that she should take prior to dental work. Inform her that she should curtail her running program. Advise Jane that she should discontinue the use of a combined oral contraceptive immediately. Inform Jane that obtaining an echocardiogram is a prudent next step.

Ans: Inform Jane that obtaining an echocardiogram is a prudent next step.

Which of the following agents/drug classes used in the management of COPD provides one of the first-line drug therapies used to minimize the risk of exacerbation? Oral xanthine (e.g., theophylline [Theolair®]) Inhaled long-acting muscarinic antagonists (LAMA) (e.g., tiotropium bromide [Spiriva®]) Inhaled short-acting beta2-agonist (SABA) (e.g., albuterol [ProAir®]) Oral PDE-4 inhibitor (e.g., roflumilast [Daliresp®])

Ans: Inhaled long-acting muscarinic antagonists (LAMA) (e.g., tiotropium bromide [Spiriva®])

You are seeing Benjamin, an 18-month-old, who presents with his mother for a sick visit. His last well-child visit was at age 5 months, when he was up-to-date for recommended immunizations. Mom states that Benjamin has not been seen by another healthcare provider nor received vaccines since his last visit at your practice. He now presents with a 2-day history of crankiness and fever. Benjamin has had a poor appetite for the past 2 days but has not vomited and has been taking small amount of fluids. His last wet diaper was approximately 2 hours ago. Exam reveals T=39.6ºC (103.4ºF), P=150 BPM, RR=45/min. Additional findings include slightly dry mucous membranes, capillary refill of <2 seconds, oropharyngeal redness, bilateral, red, immobile TMs, and a clear chest. The child has a high-pitched cry, is difficult to console, and does not regard his mother's face. Your next best action is to: Start the child on high-dose PO amoxicillin and oral analgesia with ibuprofen. Give Benjamin an age- and weight-appropriate dose of an oral antipyretic with plans to reassess after 1 hour. Initiate an evaluation for sepsis and consider for inpatient admission. Administer a single dose of IM ceftriaxone and arrange for revisit tomorrow.

Ans: Initiate an evaluation for sepsis and consider for inpatient admission.

Ms. Leonardo is a 68-year-old woman of Italian ancestry who presents with a new diagnosis of primary hypertension. Recent lab analysis reveals the following: A1C=5.3% (0.053 proportion), calculated GFR=98 mL/min/1.73 m2. Physical examination findings include the following: BMI=26.4 kg/m2, no S3, S4, or murmur, PMI at 5th ICS, MCL. Funduscopic examination is within normal limits. In the past 6 months, she has increased physical activity by walking for at least 40 minutes 5-6 days per week and has cut back on dietary sodium. Which of the following represent(s) the best advice? Choose all that apply. Advise that beta-blocker therapy should be started Initiate therapy with a thiazide diuretic Encourage weight reduction Advise Ms. Leonardo that drug therapy will be initiated when there is evidence of target organ damage

Ans: Initiate therapy with a thiazide diuretic, Encourage weight reduction

The most appropriate next step in Ms. Hannigan's plan of care is: Based on the below scenario: Ms. Hannigan is a 70-year-old woman with well-controlled hypertension, dyslipidemia, and hypothyroidism. Her current medications include a statin, ACE inhibitor, a thiazide diuretic, and levothyroxine replacement. She presents today with a chief complaint of a 2-month history of fatigue and aching sensation accompanied by morning stiffness in her hips and shoulders. During this time, she has lost 8 lb (3.6 kg) "without even trying" and reports struggling to get dressed, especially when putting on a shirt or pants. Physical exam reveals full limb strength, decreased active range of motion in the hips and shoulders, cool, smooth joints, and no excessive muscle tenderness. Laboratory test results include: Hb=10.8 g/dL (108 g/L) Hct=32% (0.32 proportion) MCV=86 fL RDW=12.2% (0.122 proportion) ESR=112 mm/h (ULN=20 mm/h) Referral to physical therapy. Rest and ice to the affected areas. Initiating a course of systemic corticosteroids. Prescribing oral low-dose bisphosphonate therapy.

Ans: Initiating a course of systemic corticosteroids

For an 87-year-old man with a history of cardiovascular disease and requires an LDL-C reduction of approximately 33%, the NP recommends: Initiating atorvastatin 20 mg daily Initiating rosuvastatin 20 mg daily Initiating simvastatin 10 mg daily No statin therapy as there is likely no significant benefit at his age

Ans: Initiating atorvastatin 20 mg daily (moderate intensity statin therapy reduction of <33% of LDL level)

Jane is a 56-year-old woman who undergoes DXA testing of the spine and reveals a T-score of ‒2.9. Based on recommendations by the American College of Physicians, the NP suggests: No treatment needed at this time. Initiating treatment with risedronate. Initiating treatment with estrogen therapy. Initiating treatment with raloxifene.

Ans: Initiating treatment with risedronate.

Radiographic assessment of a patient with early osteoarthritis will most likely reveal: Soft tissue swelling. Involvement of the metacarpals. Osteophyte formation. Joint space narrowing on X-ray.

Ans: Joint space narrowing on X-ray.

Timmy is a 4-year-old boy who presents with his Mom today for a sick visit. For the past 8 days, he has had intermittent fever as high as 104.5ºF (40.3ºC) and has complained of a sore throat and increased throat pain with swallowing, but without difficulty taking fluids. He has little appetite, but his mother denies nausea, vomiting, diarrhea, or constipation. On examination, you note he is alert, appears ill without acute distress, and has extensive cervical lymphadenopathy, injected conjunctivaoral erythema, and a peeling rash on his hands. You consider a diagnosis of: Infectious mononucleosis. Fifth's disease. Hand, foot, and mouth disease. Kawasaki disease.

Ans: Kawasaki disease.

Metformin is contraindicated in the patient with a GFR <45 mL/min/1.73 m2 because of the risk of: Hypokalemia. Worsening renal function. Hypocalcemia. Lactic acidosis.

Ans: Lactic acidosis.

A 61-year-old woman sees you in follow-up after being discharged from the hospital. She recently had an inferior wall myocardial infarction and is asking about supervised cardiac rehabilitation. She is shocked that she had a heart attack, and says she never even knew that her cholesterol was so high. She hasn't seen a healthcare provider since having her last child 27 years ago. Tertiary prevention strategy for this patient will include: Referral for mammography. Lipid-lowering therapy with an HMG-CoA reductase inhibitor (statin). Influenza and pneumococcal vaccination. Screening for diabetes.

Ans: Lipid-lowering therapy with an HMG-CoA reductase inhibitor (statin).

Which of the following classes of agents used in the management of COPD best provides a protracted duration of bronchodilation? Short-acting beta2-agonists (SABA) Inhaled corticosteroids PDE-4 inhibitor Long-acting beta2-agonists (LABA)

Ans: Long-acting beta2-agonists (LABA)

As you develop a treatment plan for Mrs. Jensen, you consider that likely: Based on the scenario below: Mrs. Jensen is an 82-year-old woman with generalized osteoarthritis and systolic hypertension who presents with a 3-day history of right-sided headache with accompanying right-sided jaw pain on chewing. OTC analgesics have provided little relief. She states she is eating little due to the pain on chewing but is taking liquids without difficulty. Mrs. Jensen reports "I can hardly wash my hair, my scalp is so sore." Physical examination reveals: BP=168/88 mm Hg bilateral, P=88 regular, RR=18, alert, appears uncomfortable with poorly-groomed hair on the right only PERLA, cranial nerve function II‒XII intact, remaining neurological exam WNL, carotid upstroke within normal limits without bruit. There is a tender, palpable, pulseless structure in the right temple area. NSAIDs will be helpful Reducing her blood pressure is important Long-term, high-dose systemic corticosteroid therapy will be needed Opioid use is contraindicated

Ans: Long-term, high-dose systemic corticosteroid therapy will be needed

An 18-year-old male presents with profound scrotal pain. Which of the following findings supports a diagnosis of testicular torsion? Positive Prehn's sign High fever Profound neutrophilia Loss of the cremasteric reflex

Ans: Loss of the cremasteric reflex

You are evaluating Karen, a 48-year-old African American woman who is a nonsmoker and drinks 2 glasses of wine per week. She has a longstanding intermittent lumbosacral strain that she attributes in part to her work, which requires her to be physically active. Today she presents with a 2-week history of shooting pain down the right leg and occasional "dragging" of the right foot with walking. Karen denies any recent trauma or precipitating event. Examination reveals abnormal straight-leg raising, diminished right patellar reflex, and difficulty performing heel walking. She relates, "I am really uncomfortable. This is different than my usual back pain." Her presentation is most consistent with: Lumbar radiculopathy. Exacerbation of chronic lumbar-sacral strain. Vertebral compression fracture. Spondyloarthropathy.

Ans: Lumbar radiculopathy.

Match each image with the correct diagnosis. A. Metatarsus adductus B. Club foot C. Polydactyly D. Syndactyly

Ans: Metatarsus adductus (forefoot turns inward, high arch and wide gap between big toes and second). Club foot (foot is turned inward, often with bottom of foot facing sideways). Polydactyly (presence of extra digits). Syndactyly (fusion of 2 or more digits or webbing of the skin between digits).

A 51-year-old woman presents with a 1-day history of fever, abdominal pain, and vomiting. The diagnosis of acute cholecystitis would be confirmed by: Isolated lymphocytosis. Elevated amylase and lipase. Right upper quadrant abdominal ultrasound. Abdominal plain films.

Ans: Right upper quadrant abdominal ultrasound.

According to the World Health Organization Medical Eligibility for Contraceptive Use, which of the following is a Category 4 (use represents unacceptable health risk) for the use of a combined oral contraceptive? Controlled hypertension Migraine with focal neurologic symptoms Age >35 years and smoking <10 cigarettes per day. BMI ≥30 kg/m2

Ans: Migraine with focal neurologic symptoms

When prescribing an SSRI, the nurse practitioner considers that: There will be full therapeutic effect within 2-3 days of the first dose. These drugs are potentially photosensitizing. These medications cannot be used in patients with cardiovascular disease. Mild GI upset is often reported in first weeks of use.

Ans: Mild GI upset is often reported in first weeks of use.

Richard is a 62 year-old man w/ HTN heart disease and HF. He presents today for follow up and is without new symptoms. Physical examination reveals the following: PMI @ 5th ICS AAL with a sustained impulse, Gr 3/6 blowing holosystolic murmur with radiation to axilla. The murmur accentuated by rolling patient onto left side, softens when going from supine to standing position, louder with hand grip, 2d heart sound not preserved, and full carotid upstroke bilaterally is noted. These findings are most consistent with: Mitral valve prolapse. Aortic stenosis. Pulmonic stenosis. Mitral regurgitation.

Ans: Mitral regurgitation MR-ASS or MS-ARD Mitral regurgitation -Aortic stenosis = Systolic murmurs (PMI at 5th ICS) Mitral stenosis- Aortic stenosis = Diastolic murmurs (2nd USB radiating to right side of the neck)

Mr. Kelly is a 66-year-old man with type 2 DM and HTN who presents with the following lipid profile: HDL=35 mg/dL (0.9 mmol/L) LDL=150 mg/dL (3.9 mmol/L) Triglycerides=210 mg/dL (2.4 mmol/L) Which of the following represents the best choice of dyslipidemia therapy for Mr. Kelly? Optimized oral selective cholesterol absorption inhibitor therapy Moderate-intensity oral statin therapy Injectable PCSK9 therapy High-intensity oral omega-3 fatty acid therapy

Ans: Moderate-intensity oral statin therapy

Taylor is a 2½-year-old otherwise well child who presents today for a sick visit. According to his father, Taylor has vomited approximately 10 times in the past 18 hours, with the last episode about 4 hours ago. He has retained sips of water during the last hour. He also developed watery stools approximately 8 hours ago with 4 episodes in total. The last stooling episode was about 1 hour ago. Fever is reported to be elevated to 38.4ºC (101.2ºF) on a number of occasions during the past day. On assessment, he is lying in his father's arms, is alert and nods his head when asked if he is thirsty. His lips are dry and slightly cracked and there is a small amount of saliva in the oral cavity. His heart rate is 110 BPM and capillary refill is <2 seconds. Dad tells you Taylor's diaper was dry when he awoke 4 hours ago but due to watery stools, he is unsure if Taylor has voided this morning. Currently, Taylor is wearing a diaper that is slightly wet in the front. Based on these findings, you consider that Taylor appears to be: Mildly dehydrated. Moderately dehydrated. Severely dehydrated. Adequately hydrated.

Ans: Moderately dehydrated.

Parents of Asian ancestry brings in her healthy 1-month-old daughter born at 40-weeks' gestation for a well child evaluation. Upon examination, you notice multiple non-tender blue-black macular lesions on the lower back and buttocks. The most likely diagnosis is: Port wine lesion. Milia. Keratosis pilaris. Mongolian spot.

Ans: Mongolian spot

Mr. Spaulding returns in 2 days. He states that he is feeling somewhat better, with less shortness of breath but with continued fatigue and production of small amounts of white-yellow sputum. He states he has taken his antimicrobial therapy as advised without difficulty. Physical examination reveals the following: Alert, BP=130/78 mm Hg, T=97.8°F (36.6°C), HR=88, RR=18, dullness to percussion over the left base with increased tactile fremitus and tubular breath sounds. Cardiac examination reveals no S3, no S4, no murmur, with nondistended neck veins. At this point, you consider the following two best options: Based on the scenario below: Mr. Spaulding is a 70-year-old man with a 50 pack-year cigarette smoking history, chronic obstructive pulmonary disease, and hypertension, who presents with a 24-hour history of increasing dyspnea and productive cough with white-yellow sputum. He is alert, oriented, and answers questions with ease. Physical examination reveals the following: Alert, breathing slightly labored at rest, BP=130/78 mm Hg, T=99.8°F (37.7°C), HR=96, RR=22, dullness to percussion over the left base with increased tactile fremitus and tubular breath sounds as well as crackles in the right base. Cardiac examination reveals no S3, no S4, no murmur, with nondistended neck veins. A repeat chest x-ray should be obtained today His antimicrobial needs to be changed to an agent with wider spectrum of activity Mr. Spaulding should be advised to complete his current course of therapy Pneumococcal and seasonal influenza vaccines should be updated today as needed

Ans: Mr. Spaulding should be advised to complete his current course of therapy Pneumococcal and seasonal influenza vaccines should be updated today as needed

A 52-year-old woman presents to your office as a new patient. She reports a long history of high blood pressure, and admits she does not regularly take her medication. Which of the following is an anticipated funduscopic finding? Narrowing of arterioles An increased cup:disk ratio Macular degeneration Cotton-wool spots

Ans: Narrowing of arterioles

You see a 56-year-old male who is newly diagnosed with dyslipidemia. Prior to initiating statin treatment, his hepatic enzyme levels are measured to establish baseline values. In the absence of symptoms of liver injury, which of the following is the recommended schedule for hepatic enzyme monitoring during statin use? Every 3 months Every 6 months Every 12 months No additional hepatic enzyme monitoring is needed

Ans: No additional hepatic enzyme monitoring is needed

You examine a thriving 4-week-old boy who was born at 39 weeks' gestation and note a painless, tense, non-reducible, relatively symmetric scrotal enlargement that brightly and evenly transilluminates. The parents report that the scrotum "always looks like this," without change in size during the course of the day. Bilateral testes, approximately 1 cm in length, are palpable and held within the scrotum. The penis is approximately 4 cm in length. You consider these findings are most consistent with: A normal examination. Bilateral inguinal hernias. Micropenis. Noncommunicating hydrocele.

Ans: Noncommunicating hydrocele.

Upon examining a 67-year-old woman, the NP detects a palpable thyroid nodule that is 1.5 cm in diameter. The most appropriate next step for this patient is: Schedule follow-up in 3 months to determine if the size changes. Obtain TSH measurement and thyroid ultrasound. Schedule MRI imaging of the head and neck. Referral to an oncologist.

Ans: Obtain TSH measurement and thyroid ultrasound.

You are seeing your 27-year-old female patient for contraceptive care. A levonorgestrel-IUD (Mirena®) was inserted 10 months ago. You anticipate that the patient will report: Dyspareunia. Vaginal dryness. Oligomenorrhea. Breast tenderness.

Ans: Oligomenorrhea.

A patient with hypothyroidism is being started on thyroid hormone replacement therapy. The nurse practitioner advises that the medication should be taken: With 8 oz (236.6 mL) of milk. On an empty stomach. At bedtime. Before a meal.

Ans: On an empty stomach.

A 61-year-old retired schoolteacher presents with moderate persistent allergic rhinitis that is impacting his ability to sleep most nights. He has a history of hypertension and suffered an NSTEMI 3 years ago. Which of the following should be avoided for this patient? Antihistamine nasal spray Leukotriene modifier Oral decongestant Intranasal corticosteroid

Ans: Oral decongestant

Matthews is a 78-year-old woman with severe COPD, who is currently using tiotropium bromide via inhaler and medium-dose inhaled fluticasone with salmeterol via MDI, and who presents for an acute care visit. Her current medications include enalapril, hydrochlorothiazide, lovastatin, and low-dose aspirin. She reports increasing dyspnea and a productive cough with small amounts of yellow-green sputum for the past 24 hours. She states, "I hardly slept at all last night. I kept waking up coughing." She denies nausea, vomiting, or fever. Physical examination reveals bilateral expiratory wheezes and rhonchi with hyperresonance to percussion without increased tactile fremitus or dullness to percussion. SaO2=98%, T=97.6°F (36.4°C), BP=136/84 mm Hg, P=92, regular, RR=20. When considering pharmacologic therapy to treat Ms. Matthews, the NP prescribes: Oral cefpodoxime and a single dose of an injectable, sustained-release corticosteroid Oral clarithromycin with an increase of her inhaled corticosteroid/LABA via MDI by 2 puffs per day IM ceftriaxone and an opioid-containing cough suppressant Oral doxycycline with a short-course of an oral corticosteroid

Ans: Oral doxycycline with a short-course of an oral corticosteroid

Appropriate outpatient treatment for a 62-year-old woman with mild diverticulitis who is without vomiting and suitable for outpatient therapy is: Oral or IV linezolid (Zyvox®) Oral metronidazole (Flagyl®) Oral levofloxacin (Levaquin®) plus oral metronidazole (Flagyl®) IV daptomycin (Cubicin®) plus oral clindamycin (Cleocin®)

Ans: Oral levofloxacin (Levaquin®) plus oral metronidazole (Flagyl®)

A 2-week-old boy is presented by his mother for evaluation. The boy was born at 39 weeks' gestation and appears healthy. The mother expresses concern about an area of abnormal growth under his left ear. She states that he was born with a pinkish "blotch" in the area but has noticed a significant change over the past few days. Upon examination, you note a red slightly-protruding spongy mass approximately 0.5 cm in diameter. For the infant described above, an appropriate initial treatment option would be: Corticosteroid injection. Oral propranolol. Mohs surgery. Laser therapy.

Ans: Oral propranolol.

Josh is a 14-year-old basketball player who presents with anterior knee pain that has occurred intermittently over the past 3 months. The pain worsens with squatting and walking up or down stairs, and is better with rest. He denies fever, weight loss, joint redness, or skin rash. Physical examination reveals a Tanner 3 male in no acute distress with a tender, swollen tibial tuberosity in the affected knee. Pain can be reproduced with resisted active extension and passive hyperflexion of the knee. No effusion is present. Josh's presentation is most consistent with: Osgood-Schlatter disease. Prepatellar bursitis. Meniscal tear. Reactive arthritis.

Ans: Osgood-Schlatter disease.

Mr. Spaulding's location of treatment should be __. Based on the scenario below: Mr. Spaulding is a 70-year-old man with a 50 pack-year cigarette smoking history, chronic obstructive pulmonary disease, and hypertension, who presents with a 24-hour history of increasing dyspnea and productive cough with white-yellow sputum. He is alert, oriented, and answers questions with ease. Physical examination reveals the following: Alert, breathing slightly labored at rest, BP=130/78 mm Hg, T=99.8°F (37.7°C), HR=96, RR=22, dullness to percussion over the left base with increased tactile fremitus and tubular breath sounds as well as crackles in the right base. Cardiac examination reveals no S3, no S4, no murmur, with nondistended neck veins. Inpatient Outpatient

Ans: Outpatient

Teresa's location of treatment should be __ Based on the scenario below: Teresa, a 38-year-old woman with no chronic health problems, presents with a chief complaint of "A cold I cannot shake for the past three weeks. She also reports an intermittent frontal headache and has taken acetaminophen with relief, as well as general malaise and a dry cough that is particularly problematic at night. She denies nausea, vomiting, chills, fever, or dyspnea. She has not taken an antimicrobial within the past year, underwent a bilateral tubal ligation approximately 10 years ago, and is allergic to penicillin with a hive-form reaction. Examination reveals the following: SaO2=97%, BP=114/70 mm Hg, T=98°F (36.7°C), HR=88, RR=20, bilateral coarse late inspiratory crackles without wheeze. She is in no acute distress. Chest x-ray demonstrates bilateral interstitial infiltrates. Inpatient Outpatient

Ans: Outpatient

Which of the following is a worrisome finding noted during pelvic examination on a 62-year-old woman? Flattening of the vaginal rugae Vaginal pH=5.6 Ovary palpable on bimanual examination Scant white vaginal discharge

Ans: Ovary palpable on bimanual examination

A 84-year-old woman recently started a medication to help with overactive bladder symptoms. Since taking the medication, she has developed dry mouth and worsening constipation. Oxybutynin (Ditropan ®). Tamsulosin (Flomax®). Mirabegron (Myrbetriq®). Finasteride (Proscar®).

Ans: Oxybutynin (Ditropan ®).

As part of a well-child visit, you advise the parents of a well 6-month-old that a mild fever of 1-2 days in duration is most likely to occur after the baby receives: Inactivated polio virus (IPV) vaccine. Haemophilus influenzae type B (Hib) vaccine. Pneumococcal conjugate 13-valent vaccine (PCV13). Injectable influenza vaccine.

Ans: PCV 13

Michelle returns for an urgent care visit three weeks later with a 2-day history of URI symptoms. She has a dry cough and reports, "The albuterol is not working as well as usual. I was up all night coughing. When I cough, a little bit of white phlegm sometimes comes up." She denies fever, nausea, or vomiting. Which of the following is the most important clinical parameter in assessing Michelle's asthma flare? Based on the scenario below: You see Michelle, a 38-year-old woman with moderate persistent asthma who is using medium-dose inhaled fluticasone daily and albuterol via MDI as needed. In a typical month, she uses albuterol 2 puffs "about 2 times, when I feel my chest getting a little tight, and it works right away." In an average month, she has no episodes of nocturnal awakening with cough or wheeze, typically has excellent activity tolerance. SaO2=97%. Peak expiratory flow=55% of personal best Presence of bilateral expiratory wheezes Patient report of reduced response to beta2-agonist use

Ans: Peak expiratory flow=55% of personal best

Genital Candida albicans infection in men typically presents with all of the following except: Penile discharge. Balanitis. Groin-fold involvement. Scrotal excoriation.

Ans: Penile discharge.

Risk factors for the development of type 2 diabetes in women include: History of early menarche. Cigarette smoking. Personal history of polycystic ovary syndrome. Caucasian race.

Ans: Personal history of polycystic ovary syndrome.

A 20-year-old male complains of being fatigued for the past week. The physical examination is significant for anterior and posterior cervical lymphadenopathy and right and left upper quadrant abdominal tenderness. A white blood cell differential is significant for lymphocytosis with reactive forms. Which of the following additional findings does the nurse practitioner anticipate? Pharyngitis with exudate Supraclavicular lymphadenopathy Diffuse maculopapular rash Marked sinus tenderness

Ans: Pharyngitis with exudate

With Kevin's skin condition, lesions are most often found in all of the following areas except: Based on the below scenario: You see Kevin, a 25-year-old man with a BMI=38 kg/m2 and hyperpigmented plaques with a velvet-like appearance at the nape of the neck and axillary region. He states his skin has had this appearance since he was approximately age 13 and it has not changed significantly over time. He denies itch or pain in these areas. Groin folds. Over the knuckles. Elbows. Plantar surface of the feet.

Ans: Plantar surface of the feet.

Developmental assessment of a healthy 11-month-old born at 32-weeks' gestation is expected to reveal an infant who: Follows a two-step command. Plays "peek-a-boo." Walks short distances solo. Says "mama, dada" specifically.

Ans: Plays "peek-a-boo."

Ms. Hannigan's clinical presentation is most consistent with: Based on the below scenario: Ms. Hannigan is a 70-year-old woman with well-controlled hypertension, dyslipidemia, and hypothyroidism. Her current medications include a statin, ACE inhibitor, a thiazide diuretic, and levothyroxine replacement. She presents today with a chief complaint of a 2-month history of fatigue and aching sensation accompanied by morning stiffness in her hips and shoulders. During this time, she has lost 8 lb (3.6 kg) "without even trying" and reports struggling to get dressed, especially when putting on a shirt or pants. Physical exam reveals full limb strength, decreased active range of motion in the hips and shoulders, cool, smooth joints, and no excessive muscle tenderness. Laboratory test results include: Hb=10.8 g/dL (108 g/L) Hct=32% (0.32 proportion) MCV=86 fL RDW=12.2% (0.122 proportion) ESR=112 mm/h (ULN=20 mm/h) Rheumatoid arthritis. Fibromyalgia. Osteoarthritis. Polymyalgia rheumatica.

Ans: Polymyalgia Rheumatica

Which of the following provides the most accurate information on assessing for alcohol abuse in a 55-year-old man who states, "I drink 5‒6 beers every night but I get to work every day." Elevated ALT/AST to ≥6 times upper limit of normal Positive response to two items on the CAGE questionnaire Modest RBC macrocytosis Elevated serum triglycerides

Ans: Positive response to two items on the CAGE questionnaire

A 17-year-old woman presents complaining of left lower abdominal and groin pain. She admits that she has had a "light period everyday" for nearly 3 weeks. The presumptive diagnosis of ectopic pregnancy is supported by which of the following? Positive urine hCG Negative pelvic ultrasound Abdominal rebound tenderness Breast pain

Ans: Positive urine hCG

You start a patient on spironolactone who is also on an angiotensin-converting enzyme inhibitor. You advise the patient to return in 4 weeks to check which of the following laboratory parameters? Sodium Calcium Potassium Chloride

Ans: Potassium

A 19-year-old female comes to your office to discuss birth control. She has just become sexually active and wants to be responsible. She has heard a lot from her friends about "the pill" (combined oral contraception {COC}) and is asking you for advice. You tell her that: Premenstrual syndrome symptoms are often improved with COC use. Long-term use is discouraged as the body needs a COC "rest" from time to time. The menstrual flow volume is typically increased with COC use. There is an increase in the rate of uterine cancer after protracted COC use.

Ans: Premenstrual syndrome symptoms are often improved with COC use.

Mr. Jones, a 52-year-old African-American man, presents for treatment of hypertension. He is currently taking a calcium channel blocker for the past 2 months. He is feeling well with no complaint and physical examination is within normal limits. His blood pressure today is 160/94 mm Hg bilaterally. The next best step is to: Add an ACEI and have the patient follow-up in 2 weeks Discontinue the calcium channel blocker and start an aldosterone antagonist Advise Mr. Jones that his blood pressure is in an acceptable range and he should follow-up in approximately 2 months Prescribe a thiazide diuretic and advise a 1-month follow-up

Ans: Prescribe a thiazide diuretic and advise a 1-month follow-up

A malpractice suit concerning a delayed cancer diagnosis is filed naming a NP in a primary care practice. Will the APRN be held to standard of care of an APRN with the same level of education and practice in primary care or in an oncology practice? Primary Care Oncology Practice

Ans: Primary Care

An appropriate time to educate parents about floor safety (i.e., potential dangers of what a child can find on the floor) is: Prior to the time when a child learns to walk. Prior to the time when the child can roll from tummy-to-back and back-to-tummy. Around the time the baby can pull to stand. Prior to child being able to sit solo.

Ans: Prior to the time when the child can roll from tummy-to-back and back-to-tummy.

A 20-year-old woman requests information about emergency contraception using high-dose progestin after having unprotected sexual intercourse approximately 18 hours ago. Today is day 12 of her normally 27-29 day menstrual cycle. You respond that: The use of this type of emergency contraception reduces the risk of pregnancy by approximately 33% if taken at this point. All forms of emergency contraception must be used within 18 hours of unprotected intercourse to be effective. The likelihood of pregnancy is minimal and emergency contraception is not indicated. Progestin-based emergency contraceptive is likely helpful in reducing pregnancy risk when used as long as 120 hours post intercourse.

Ans: Progestin-based emergency contraceptive is likely helpful in reducing pregnancy risk when used as long as 120 hours post intercourse.

When considering evidence-based practice (EBP) recommendations for the use of prophylactic migraine treatment, which of the following is the preferred agent? Propranolol. Ergotamine. Rizatriptan. Verapamil.

Ans: Propranolol.

All of the following are characteristics of long-acting muscarinic antagonists (LAMA) when used in the management of COPD except: Provide relief of acute bronchospasm Offer a protracted duration of bronchodilation Minimize the risk of COPD exacerbation Are usually used on a daily set schedule

Ans: Provide relief of acute bronchospasm

While counseling a patient about the adverse effects of first-generation antihistamines, the nurse practitioner advises that all of the following usually occur except: Sedation. Dry mouth. Pruritus. Urinary retention.

Ans: Pruritus.

Which of the following medications should be avoided in a 45-year-old man with poorly-controlled hypertension who asks, "What medications can I take when I have a cold?" Dextromethorphan Chlorpheniramine Pseudoephedrine Guaifenesin

Ans: Pseudoephedrine

A clinical trial is studying whether a vaccine prevents disease. Among those who receive the vaccine, 10% get the disease compared to 40% who received placebo. In this case, the RRR is: 10% 30% 75% 300%

Ans: RRR is the relative risk (10/40) x 100 = 25%; 100-25% = 75% Of 40% who received placebo, only 10% received the vaccine which is proportionally equated to 0.25% who have prevented from diseases; remaining 75% who do not receive the vaccine can be infected with diseases. When RRR is > 1, risk of good or bad outcome of the intervention or treatment is increased. When RRR< 1 the risk of good or bad outcome of the intervention is decreased; when RRR = 1 no change.

he preferred therapeutic option to resolve the underlying pathology in Graves' disease is: Non-cardioselective beta blockade. Surgical resection of hyperfunctioning nodule. Radioactive ablation of overactive thyroid tissue. Levothyroxine suppression therapy.

Ans: Radioactive ablation of overactive thyroid tissue.

Which of the following is most consistent with a normal developmental exam for a thriving 6-month-old infant born at 32 weeks' gestation? Responds to own name and sits without support Reaches for toy with one hand and recognizes familiar people and objects at a distance Babbles mamama, bababa and transfers objects hand-to-hand without difficulty Vocalizes "ah" and "oh" sounds, and is able to lift head briefly when positioned on the tummy and turn it from side-to-side

Ans: Reaches for toy with one hand and recognizes familiar people and objects at a distance

Jackson is a 13-month-old who is in for a well-child visit. Height and weight are at approximately 40th percentile and he is on target developmentally. His diet consists of approximately 18 oz (0.53 L) of whole cow's milk per day and a variety of vegetables, fruits, lean meats, and grains. You advise that Jackson: Is eating a well-balanced diet and no nutritional supplements are needed. Should receive iron supplement equivalent to 1 mg/kg/d. Receive vitamin D 400 IU as a daily oral supplement. Should be taking in more calcium via increased cow's milk intake to ≥28 fl oz (0.83 L) per day.

Ans: Receive vitamin D 400 IU as a daily oral supplement.

A 27-year-old woman presents for care. She reports a 3-day history of intense vaginal itch, burning with urination, and white, clumping discharge. She denies a foul odor but says her genitals "smell a little musty." Which of the following history findings supports your diagnosis? New sexual partner Swimming in a local lake Recent antibiotic use Recent insertion of levonorgestrel-releasing IUD

Ans: Recent antibiotic use

Post-menopausal hormone replacement therapy is indicated for: Prevention of cerebrovascular events. Prevention of cardiovascular disease. Reduction in frequency and severity of vasomotor symptoms. Reduction of cardiovascular events.

Ans: Reduction in frequency and severity of vasomotor symptoms.

Which of the following represents the most appropriate next step in Karen's plan of care? Based on the below scenario: You are evaluating Karen, a 48-year-old African American woman who is a nonsmoker and drinks 2 glasses of wine per week. She has a longstanding intermittent lumbosacral strain that she attributes in part to her work, which requires her to be physically active. Today she presents with a 2-week history of shooting pain down the right leg and occasional "dragging" of the right foot with walking. Karen denies any recent trauma or precipitating event. Examination reveals abnormal straight-leg raising, diminished right patellar reflex, and difficulty performing heel walking. She relates, "I am really uncomfortable. This is different than my usual back pain." Advise a 3-day course of bedrest Refer to physical therapy Order a lumbosacral MRI Obtain a standing lumbosacral x-ray

Ans: Refer to physical therapy

Samantha, a healthy 32-year-old woman who is taking combined oral contraceptives, is here to review the results of her recent liquid-based Pap screening that revealed atypical squamous cells of unknown significance (ASCUS) and high-risk HPV positive. She has no history of previous abnormal cervical cytology, with her last screening obtained approximately 2 years ago. After explaining the significance of these findings, the most appropriate next step is to: Advise that she return in 6-12 months for a repeat Pap with HPV cotesting. Obtain screening tests for N. gonorrhoeae and C. trachomatis infection. Referral for colposcopy. Counsel that the usual cervical cancer screening guidelines should be followed.

Ans: Referral for colposcopy.

A 64-year-old man presents complaining of difficulty initiating urination, a weak urine stream, and inability to completely empty the bladder. Suspecting benign prostatic hyperplasia, you prescribe tamulosin and advise the patient that this will: Decrease the size of the prostate. Relax the bladder neck muscles to ease urination. Eradicate bacteria that cause inflammation of the prostate. Reverse hormonal changes that cause prostate enlargement.

Ans: Relax the bladder neck muscles to ease urination.

Tina is an otherwise well 15-year-old who presents with her mother. They report that Tina has had a one-day history of "sore throat and swollen glands" as well as a low-grade fever and rash. Examination reveals a diffuse maculopapular rash, mildly tender posterior cervical and postauricular lymphadenopathy, and pharyngeal erythema without exudate. The remainder of her history and review of systems is unremarkable. Per her mother's report, Tina has not received any immunizations since age 6 months. The most likely diagnosis is: Scarlet fever. Roseola. Rubella. Rubeola.

Ans: Rubella.

The nurse practitioner has diagnosed acute bacterial rhinosinusitis (ABRS) and is considering antibiotic choices. He recognizes that the most common causative pathogen is: H. influenzae. S. aureus. S. pneumoniae. M. catarrhalis.

Ans: S. pneumoniae.

In peripheral arterial disease, all of the following are recommended except: Referral to vascular surgery. Sclerotherapy. Oral cilostazol (Plental®) to reduce symptoms. Management of cardiovascular risk factors

Ans: Sclerotherapy.

Choosing all that apply, you offer Janet the following immunizations against: Based on the scenario below: Janet, a 47-year-old well woman who is 64" (163 cm) tall with BMI=25 kg/m2, presents for a periodic health evaluation and to establish care in your practice. She reports drinking approximately 1 to 2 mixed drinks containing 1.5 oz. (44.4 mL) 80-proof liquor per week and takes approximately 1 to 2 doses of acetaminophen per month. Her last visit with a healthcare provider was more than 5 years ago; she was told that her "liver tests were a bit higher than normal." She felt well and decided not to return for follow-up. She denies history of injection drug use and has no tattoos. Her last vaccines were administered more than 10 years ago. She is without complaint. Abdominal examination is within normal limits. Janet's laboratory results are as follows. HBsAg=positive (evidence HBV on board, Ag=Always growing, HBV persists, in acute or chronic hepatitis B) Anti-HAV=positive (immune to HAV, include HAV IgG, G=Gone) Anti-HCV=negative (no evidence of past or present HCV infection) AST=56 unit/L (0-40) (modest elevation, with ALT>AST) ALT=98 unit/L (0-40) (modest elevation, with ALT>AST). Hepatitis B Hepatitis A Seasonal influenza Tetanus, diphtheria, and acellular pertussis

Ans: Seasonal influenza Tetanus, diphtheria, and acellular pertussis

Which of the following psychotropic medications would be preferred in the elderly due to its low anticholinergic and sedation effect? Amitriptyline Nortriptyline Sertraline Paroxetine

Ans: Sertraline

Gary is a 26-year-old graduate student who is diagnosed with major depressive disorder (MDD) without psychotic features. His medical history is unremarkable and he states that he does not wish to undergo psychotherapy. An appropriate first-line medication is: Sertraline (Zoloft®). Bupropion (Wellbutrin®). Amitriptyline (Elavil®). Trazodone (Oleptro®).

Ans: Sertraline (Zoloft®).

You see an 81-year-old woman with type 2 DM. Today she presents for follow-up care. Results of recent laboratory assessment include the following: GFR=45 mL/min/1.73 m2, A1C=7.2% (0.072 proportion), K+=4.8 mEq/L (4.8 mmol/L). Her current medications include metformin, glyburide, lisinopril, and simvastatin. She is generally feeling well, is self-sufficient in ADL, and states, "I can get around in my home with my cane. I am a little less certain on my feet if I do not use the cane." Due to age and impaired renal function, her risk of adverse effects from metformin is increased. Glipizide is preferred over glyburide as a sulfonylurea in the older adult. With poor hydration, she is at risk for hyperkalemia. She should be treated to an A1C goal of <7% (0.07 proportion).

Ans: She should be treated to an A1C goal of <7% (0.07 proportion).

Actinic keratoses can be described as: Hyperpigmented macules on sun-exposed areas. Vesicular lesions along a dermatomal distribution. Ulcerating lesions in groin folds. Slightly rough, pink or flesh-colored lesions on the face.

Ans: Slightly rough, pink or flesh-colored lesions on the face.

Select components of substance-abuse disease include all of the following except: Craving or a strong desire to use the problematic substance. Over time, diminished effect with the same amount of the substance. Set of characteristic withdrawal symptoms. Social or occupational function remains intact.

Ans: Social or occupational function remains intact.

A 2½-year-old boy is brought by his mother who reports that, approximately 10 minutes ago, the child was injured when he pulled a pot of boiling water off the stove. The child is alert and crying, and age-appropriately resists the examination. Skin survey reveals approximately 18% body surface area of moist, red skin with peeling borders, largely involving the posterior thighs, buttocks, and scrotum. No other injury is noted. Identify the two most important considerations in his care. The injury should be débrided as soon as possible. The affected area should be promptly washed with an antiseptic solution. Specialty burn care should be promptly sought. Child protective services should be immediately notified. A course of oral antibiotics should be initiated.

Ans: Specialty burn care should be promptly sought. Child protective services should be immediately notified.

Actinic keratosis can be a precursor to: Malignant melanoma. Acanthosis nigricans. Basal cell carcinoma. Squamous cell carcinoma.

Ans: Squamous cell carcinoma.

Which of the following is inconsistent with benign prostatic hyperplasia? Obliterated median sulcus Size ≥2.5 cm × 3 cm Symptoms improved with use of an alpha-1 receptor blockade such as tamsulosin. Surgical intervention should be offered early in the disease process.

Ans: Surgical intervention should be offered early in the disease process.

Which of the following is an acceptable team sport for a college-age young adult with a bleeding disorder to participate in? Lacrosse Soccer Swimming No team sport is acceptable.

Ans: Swimming

You see a 26-year-old college student diagnosed with major depressive disorder. He has been treated with a SSRI for the past 6 months with minimal clinical effect. You recommend as the next course of action: Adding a benzodiazepine. Adding a second-generation antipsychotic (SGA). Switching to a SNRI. Switching to a tricyclic antidepressant (TCA).

Ans: Switching to a SNRI.

The murmur of mitral valve insufficiency is best described as: Mid-late systole with a "snap." Diastolic and generalized across the precordium. Diastolic with radiation to the neck. Systolic with radiation to the axillae.

Ans: Systolic with radiation to the axillae.

Laboratory assessment of a patient with Hashimoto's thyroiditis typically includes the presence of: Polycythemia. TPO antibodies. Leukocytosis. Hypokalemia.

Ans: TPO antibodies. (thyroid protease antibodies)

A 38-year-old woman presents complaining of weight loss, agitation, and new-onset tremor. The nurse practitioner suspects hyperthyroidism. Laboratory assessment will most likely include: TSH=8.9 mIU/L (0.4-4.0 mIU/L), free T4 =15 pmol/L (10-27 pmol/L). TSH<0.15 mIU/L (0.4-4.0 mIU/L), free T4 =79 pmol/L (10-27 pmol/L). TSH=24 mIU/L (0.4-4.0 mIU/L), free T4 =3 pmol/L (10-27 pmol/L). TSH=1.9 mIU/L (0.4-4.0 mIU/L), free T4 =22 pmol/L (10-27 pmol/L).

Ans: TSH<0.15 mIU/L (0.4-4.0 mIU/L), free T4 =79 pmol/L (10-27 pmol/L).

A patient presenting with Graves' disease will most likely demonstrate which laboratory finding? Free T4 =3 pmol/L (NL=10-27 pmol/L) TSH=0.04 mIU/L (NL=0.15-4.0 mIU/L) ESR=42 mm/h (NL <15 mm/h) TWBC=12,400 cells/mm3 with 70% neutrophils

Ans: TSH=0.04 mIU/L (NL=0.15-4.0 mIU/L)

Laboratory assessment of a patient with subclinical hypothyroidism is likely to reveal: TSH=0.1 mIU/L (0.4-4.0 mIU/L); total T4=7.2 mcg/dL (4.5-12.0 mcg/dL). TSH=2.6 mIU/L (0.4-4.0 mIU/L); total T3=310 ng/dL (95-190 ng/dL). TSH=7.7 mIU/L (0.4-4.0 mIU/L); free T4=22 pmol/L (10-27 pmol/L). TSH=0.2 mIU/L (0.4-4.0 mIU/L); free T3=1.7 ng/dL (0.2-0.5 ng/dL).

Ans: TSH=7.7 mIU/L (0.4-4.0 mIU/L); free T4=22 pmol/L (10-27 pmol/L).

You provide the following information to Taylor's father. Based on the below scenario: Taylor is a 2½-year-old otherwise well child who presents today for a sick visit. According to his father, Taylor has vomited approximately 10 times in the past 18 hours, with the last episode about 4 hours ago. He has retained sips of water during the last hour. He also developed watery stools approximately 8 hours ago with 4 episodes in total. The last stooling episode was about 1 hour ago. Fever is reported to be elevated to 38.4ºC (101.2ºF) on a number of occasions during the past day. On assessment, he is lying in his father's arms, is alert and nods his head when asked if he is thirsty. His lips are dry and slightly cracked and there is a small amount of saliva in the oral cavity. His heart rate is 110 BPM and capillary refill is <2 seconds. Dad tells you Taylor's diaper was dry when he awoke 4 hours ago but due to watery stools, he is unsure if Taylor has voided this morning. Currently, Taylor is wearing a diaper that is slightly wet in the front. Taylor can go home now on rehydration therapy with an appropriate oral rehydration solution and clear liquids. Taylor should be started on rehydration therapy with an appropriate oral rehydration solution in the office now with a goal of demonstrating ability to tolerate oral fluids. Given Taylor's hydration status, he should be hospitalized for parenteral fluid replacement. Taylor is able to go home on a diet of dry toast, mashed bananas, applesauce and white rice along with sips of clear liquids.

Ans: Taylor should be started on rehydration therapy with an appropriate oral rehydration solution in the office now with a goal of demonstrating ability to tolerate oral fluids.

The adult daughter of an 88-year-old man telephones the nurse practitioner to inquire about her father's medications. The daughter reports that "my father relies upon me to explain everything to him." The nurse practitioner's initial response is to: Ask the daughter to provide a copy of the patient's advance directive document. Assure the daughter that the NP can share the requested information. Inform the daughter that she must come to the clinic to discuss her father's care. Tell the daughter that the NP can discuss the information only with the patient.

Ans: Tell the daughter that the NP can discuss the information only with the patient.

During a 12-month well-child checkup, the nurse practitioner notices that the child is able to pull to stand, haltingly cruises, but is not walking independently. The mother is very concerned and insists that her other three children were all walking by the age of 1 year. The most appropriate action would be to: Tell the mother that while on average children are walking by 1 year, many children do so later; schedule a follow-up in 1-2 months to assess progress. Reassure the mother that children with older siblings often do not start walking until later; schedule the next routine well-child checkup in 3 months. Counsel the mother that while there is no need to panic, this does represent a developmental delay and the child should see a pediatric orthopedic specialist. Request a radiographic assessment of the spinal cord, pelvis, and femurs to rule out structural bony anomalies that frequently delay walking.

Ans: Tell the mother that while on average children are walking by 1 year, many children do so later; schedule a follow-up in 1-2 months to assess progress.

During a well child visit of a 14-year-old boy, the mother expresses concern that he is the shortest boy on the soccer team. Physical examination reveals a small amount of coarse, pigmented hair at the base of the penis and the scrotal sac is slightly reddened and rugated. You advise: Testing for serum human growth hormone level. That he is likely near his adult height. That his growth spurt will start soon. Performing a genetic test to detect a chromosomal abnormality.

Ans: That his growth spurt will start soon.

In initial prescription of thyroxine therapy, which of the following statements is true? TSH should be checked about 2 days after dose adjustment. The anticipated thyroxine dose is usually calculated using an age- and weight-based formula. TSH should be suppressed to undetectable measures when seeking therapeutic effect. The therapeutic dose of thyroxine needed by the elderly is approximately the same as that needed by the younger adult and child.

Ans: The anticipated thyroxine dose is usually calculated using an age- and weight-based formula.

You are providing counseling for the parents and caregivers of a healthy full-term newborn who is being breastfed. The counseling should include information on all of the following except: The baby should make at least 6 wet diapers a day. Newborns often lose up to 10% of birth weight in the first week of life. A breastfed baby usually has 4 or more bowel movements per day. The baby should be back up to birth weight by age 3-4 weeks.

Ans: The baby should be back up to birth weight by age 3-4 weeks.

A mother brings in her 4-year-old son for a well child visit. The mother explains that he is recovering from a cold, but he is in generally good health with no complaints. Upon examination, the NP notices a reddened tympanic membrane but an absence of any bulge. The NP explains: A follow-up visit is needed in 2 days to determine if the infection clears. Antimicrobial therapy with amoxicillin should be initiated. Antimicrobial therapy with azithromycin should be initiated. The benefits of antimicrobial therapy do not outweigh potential risks of adverse effects and resistance development.

Ans: The benefits of antimicrobial therapy do not outweigh potential risks of adverse effects and resistance development.

Which of the following describes the ethical principle of veracity? Healthcare resources are allocated so that the best is done for the greatest number of people. The healthcare provider must be truthful and avoid deception. The healthcare provider has an obligation to be faithful to commitments made to self and others. The responsibility of the healthcare provider is to treat all people in the same fair manner.

Ans: The healthcare provider must be truthful and avoid deception.

Which of the following describes the ethical principle of beneficence? The right of the competent person to choose a personal plan of life and action. The obligation of the healthcare provider to help people in need. The duty of healthcare provider to do no harm. The responsibility of the healthcare provider to treat all in the same fair manner.

Ans: The obligation of the healthcare provider to help people in need.

When communicating healthcare decisions through a healthcare proxy, the NP recognizes: The proxy has the same rights to request or refuse treatment that the patient would have if he or she were capable of making and communicating decisions. Only management decisions that are considered life-saving are required to be communicated to the healthcare proxy. The proxy has the right to refuse treatment except in situations when treatment is necessary to preserve life. The closest relative to the patient is assumed to take the proxy responsibilities.

Ans: The proxy has the same rights to request or refuse treatment that the patient would have if he or she were capable of making and communicating decisions.

The most appropriate next diagnostic step for Caleb is: Based on the below scenario: Caleb, a 9-month-old boy, is presented for evaluation after sudden onset of intermittent severe abdominal pain. He often draws his knees to his abdomen when he appears to be in greatest pain. The mother reports that he has had episodes of diarrhea that contain a mixture of blood and mucous. He has not vomited during this time, but also has a lack of appetite. Ultrasound of the abdomen Flat plate of the abdomen Stool culture with susceptibility testing Stool sample to check for ova and parasites

Ans: Ultrasound of the abdomen

Which of the following is an absolute contraindication to post-menopausal hormone therapy? Unexplained vaginal bleeding Seizure disorder Endometriosis Cigarette smoking

Ans: Unexplained vaginal bleeding

A 78 Year-old woman with a 30-year history of hypertension and dyslipidemia and a 10-year history of type 2 diabetes presents with suspected acute coronary syndrome. She is most likely to report a recent onset of: Retrosternal chest pain with radiation to the left arm. Syncope. Unusual fatigue. Nausea and vomiting.

Ans: Unusual fatigue

Mrs. Little currently resides with her daughter's family and they voice a desire for this to continue "as long as possible and safe." When evaluating Mrs. Little's healthcare needs, choose the two options that do not apply. Based on the below scenario: Mrs. Little is a 78-year-old woman with recently-diagnosed Alzheimer-type dementia (AD) who is here today for an office visit with her 55-year-old daughter. According to her daughter, Mrs. Little struggles with word-finding and has difficulty following directions. She appears "not to care about what is going on around her," while other times is engaged in family activities. According to her daughter, Mrs. Little will have an angry verbal outburst that is triggered by a minor problem. Her daughter states, "This is not like my mother. Usually she is very patient." A home safety evaluation should be conducted and appropriate modifications carried out. If Mrs. Little has a sudden change in mental status, her healthcare provider should be contacted as soon as possible. Behavioral difficulties often arise in individuals with AD if their usual routine is disrupted. The use of a cholinesterase inhibitor will likely improve her mental status to a point that is nearly equivalent to her predementia baseline. The most common adverse effects from cholinesterase inhibitor use include nausea and diarrhea. A second-generation antipsychotic such as risperidone should be started to help minimize the angry outburst risk.

Ans: The use of a cholinesterase inhibitor will likely improve her mental status to a point that is nearly equivalent to her predementia baseline. A second-generation antipsychotic such as risperidone should be started to help minimize the angry outburst risk.

A 16-year-old young woman presents to you the day after a "condom break." She is concerned that she might become pregnant and is asking about emergency contraception. Appropriate counseling about the use of hormonal emergency contraception (EC) includes all of the following except: An established pregnancy will not be interrupted. There is about a 50% reduction in pregnancy risk with properly-timed use. There is no increased risk of birth defect if pregnancy occurs. Ulipristal (ella®) is more effective than levonorgestrel in days 3-5 following unprotected intercourse.

Ans: There is about a 50% reduction in pregnancy risk with properly-timed use.

For the newborn described above, you advise: A referral to a dermatologist who specializes in pulse dye laser therapy. These lesions will likely lighten over time and disappear during childhood. Evaluation at 6-month intervals due to increased malignancy potential. Initiating a course of corticosteroid cream for the affected area.

Ans: These lesions will likely lighten over time and disappear during childhood.

One week into sertraline (Zoloft®) therapy, a 34-year-old woman complains of a recurrent dull frontal headache that is relieved with acetaminophen. You advise her that: This is a common, transient adverse effect of SSRI therapy. Bupropion (Wellbutrin®) should be substituted. Desipramine (Norpramin®) should be added. She should discontinue the medication.

Ans: This is a common, transient adverse effect of SSRI therapy.

You are seeing an 8-year-old healthy boy who is brought in by his mother who states, "He is in trouble in school and failing two classes. His teacher thinks he is too hyper and will not sit still and wants him to be evaluated. I do not understand this. He is fine at home." When considering the diagnosis of attention deficit/hyperactivity disorder in this child, you understand that: In the majority of children diagnosed with ADHD, the symptoms and behaviors will resolve by early adulthood. Psychostimulant use in the child with ADHD can lead to substance abuse disorders in adolescence. To confirm the diagnosis of ADHD, assessment of the child's behavior at home and school should be conducted using standardized scales. When psychostimulants are prescribed with the diagnosis of ADHD, concomitant behavioral therapy is usually not needed.

Ans: To confirm the diagnosis of ADHD, assessment of the child's behavior at home and school should be conducted using standardized scales.

Physical examination of a patient with symptomatic Graves' disease will likely reveal: Generalized hypopigmentation. Tremor. Blunted pupil response. Thinning hair.

Ans: Tremor.

A person who is Medicaid eligible in one state might be deemed to be ineligible in another state. True False

Ans: True

Adults born from 1945 through 1965 should be encouraged to be screened for HCV regardless of HCV risk factors. True False

Ans: True

Impoverishment is the primary requirement for Medicaid eligibility. True False

Ans: True

Medicaid is the largest payer for nursing facility care. True False

Ans: True

The Medicare program pays for any services that are covered by Medicare before any payments are made by the Medicaid program, since Medicaid is always the payer of last resort. True False

Ans: True

The charge of the State Board of Nursing is to ensure public safety. True False

Ans: True

The services rendered under Medicaid that is provided by one state can differ considerably in amount, duration, or scope from services provided in similar or neighboring states. True False

Ans: True

Matthew is a 29-year-old man who presents with a 6-hour history of sudden onset of inability to raise his eyebrow or smile on the right side. He also reports decreased lacrimation in the right eye and difficulty closing the right eyelid. The rest of his health history and physical examination is otherwise unremarkable. This likely represents paralysis of cranial nerve (CN): III. VIII. IV. VII.

Ans: VII

Performing Weber and Rinne tests is part of the evaluation of cranial nerve (CN): V. VI. VII. VIII.

Ans: VIII.

A patient who underwent CABG following a MI develops a surgical site infection. Upon reviewing the patient's records, the NP notices that standard antimicrobial prophylaxis was not provided prior to surgery. The ethical principle of ________ requires the NP to report this finding. Nonmaleficence Justice Veracity Utilitarianism

Ans: Veracity

Which of the following WBC response would be most consistent in a patient with acute appendicitis? WBC 5000/mm3, Neutrophils 32%, Bands 4% WBC 17,500/mm3, Neutrophils 72%, Bands 10% WBC 14,000/mm3, Neutrophils 24%, Bands 3% WBC <1000/mm3, Neutrophils 52%, Bands 6%

Ans: WBC 17,500/mm3, Neutrophils 72%, Bands 10%

Sam is a 15-year-old with a BMI=40 kg/m2 who presents with a lipid profile that reveals low HDL, elevated triglycerides, and an acceptable A1C.Which of the following is the recommended treatment option? Oral niacin Oral fibrate therapy Weight loss Oral statin therapy

Ans: Weight loss

While evaluating a patient with psoriasis vulgaris, the nurse practitioner expects to find: Lichenification in the post-auricular region. Maculopapular lesions over the upper thorax. Scaling lesions within the nasolabial folds. Well-demarcated plaques and coalescing papules on the knees.

Ans: Well-demarcated plaques and coalescing papules on the knees.

When evaluating a patient with low back pain without radiculopathy, the nurse practitioner will most likely identify: Discomfort to direct palpation of paraspinal muscles. A positive straight leg raise. Abnormal lower extremity DTR. Pain relief with lumbar flexion.

Ans: When evaluating a patient with low back pain without radiculopathy, the nurse practitioner will most likely identify:

You are examining a 21-year-old man who is seeking a sports clearance physical examination. He is generally healthy with an unremarkable medical history. During cardiovascular examination, you identify a physiologic split S2 and realize that this: Is usually first noticed in early adulthood. Can progress to a pathologic condition during adulthood. Will be more pronounced with inspiration. Is associated with S4 heart sound.

Ans: Will be more pronounced with inspiration.

As part of pediatric primary care, parents and caregivers should be counseled to schedule the child's first dental visit: When the child has erupting molars. With eruption of the first tooth. At age 3 years. When the child has been weaned from breast or bottle.

Ans: With eruption of the first tooth.

The preferred technique for initial diagnosis of osteoarthritis of the knee is: X-ray. MRI. CT scan. Arthroscopy

Ans: X-ray.

The preferred technique for initial diagnosis of osteoarthritis of the knee is: X-ray. MRI. CT scan. Arthroscopy.

Ans: Xray

Mr. Nelson is a 75-year-old man in need of surgery. Mr. Nelson has been treated unsuccessfully with conservative therapy for a bowel obstruction. Documented on his record is his consent to surgery that was obtained by the chief surgical resident. During your preoperative visit with Mr. Nelson, he is alert and relatively comfortable. His wife mentions that her husband had a dose of promethazine (Phenergan®) to manage his nausea around the time the surgical consent was obtained. Mrs. Nelson asks, "Is the consent OK since my husband clearly does not feel well?" You respond: Since the chief surgical resident obtained the surgical consent, this should be considered valid. Legal counsel should be obtained prior to proceeding. That surgical consent should be obtained from Mrs. Nelson as she is Mr. Nelson's de facto healthcare proxy. You will call the chief surgical resident to clarify the issue of consent.

Ans: You will call the chief surgical resident to clarify the issue of consent.

Which of the following is demonstrated to provide the most symptom relief in treating vasomotor symptoms? Clonidine Paroxetine Conjugated estrogen Venlafaxine

Ans: conjugated estrogen

Which of the following is the most important time to screen for hearing defects? In the first days of life During the time of most intense speech formation Before the child enters school Once antimicrobial therapy for AOM is completed

Ans: in the first days of life

Which of the numbered areas on the chest x-ray best correlates with Mr. Spaulding's physical examination findings? Based on the scenario below: Mr. Spaulding is a 70-year-old man with a 50 pack-year cigarette smoking history, chronic obstructive pulmonary disease, and hypertension, who presents with a 24-hour history of increasing dyspnea and productive cough with white-yellow sputum. He is alert, oriented, and answers questions with ease. Physical examination reveals the following: Alert, breathing slightly labored at rest, BP=130/78 mm Hg, T=99.8°F (37.7°C), HR=96, RR=22, dullness to percussion over the left base with increased tactile fremitus and tubular breath sounds as well as crackles in the right base. Cardiac examination reveals no S3, no S4, no murmur, with nondistended neck veins.

Ans: left lower lobe with consolidation (picture in module) look for Xray film images online.

The most common contraceptive method used by teens is: Male condom. Combined oral contraceptives. DMPA injection (Depo-Provera®). Withdrawal.

Ans: male condoms

You examine a healthy 2-month-old boy and note that his foreskin cannot be retracted. You consider that: The foreskin should be forcibly retracted to facilitate cleaning. In most instances, the foreskin is not easily retractable until the child is about 3 years old. The risk for hypospadias or epispadias is increased in the presence of this finding. Persistent maternal hormonal influences contribute to this problem.

Ans: n most instances, the foreskin is not easily retractable until the child is about 3 years old.

A chest x-ray is ordered on a 47-year-old man with suspected active tuberculosis. The radiograph supports this diagnosis with findings of a/an: (Picture 2 Ch 10 Tuberculosis) Infiltrate in the left upper lobe Infiltrate in the right upper lobe Pleural effusion in the left upper lobe Pleural effusion in the right upper lobe

Ans: nfiltrate in the right upper lobe ( picture in the module or online)

All of the following are recommended by the American Academy of Orthopaedic Surgeons (AAOS) for the management of OA of the knee except: Weight loss if BMI ≥25 kg/m2. Use of hyaluronic acid injection. Low-impact aerobic exercise. Strengthening exercises.

Ans: use of hyaluronic acid injection

The APRN writes a prescription for trimethoprim-sulfamethoxazole for a patient who has a skin and soft tissue infection and known sulfa allergy. The patient takes two doses of the medication, has no reaction, then realizes this might be one of the problematic antibiotics. The patient notifies the practice and the APRN changes the medication. He makes a full recovery. Yes or No 1. Is there negligence? __ 2. Is there damage? __ 1-Yes 2-Yes 1-No 2-Yes 1-Yes 2-No 1-No 2-No

Ans:1-Yes 2-No

Assessment of the optic disc is a component of the evaluation of cranial nerve: I. II. III. .IV

Cranial II

You see Alexandra, born at 40.5 weeks' gestation, who is now 14 days old. According to her mother, she is a vigorous eater and is both breast- and formula-fed. On examination of the neonate, you note bilateral breast engorgement with physiologic galactorrhea on the left. You appreciate all of the following are correct concerning this condition except: Its onset is usually at approximately day 3-4 of life. That maternal hormonal influences are likely the cause. This breast engorgement will resolve without intervention within the first two months of life. Further evaluation is required to confirm this assessment.

Further evaluation is required to confirm this assessment.

When evaluating the Mantoux tuberculin skin test (TST), a positive or negative result is determined by: AThe area of erythema surrounding the injection site. The diameter of induration surrounding the injection site. Presence of generalized maculopapular rash. Presence of fever ≥99.8ᵒC.

The diameter of induration surrounding the injection site.

Match the following urinary incontinence types: A. Urge incontinence B. Stress incontinence C. Functional incontinence D. Transient incontinence 1. Associated with lifting __ 2. Occurs during an acute illness __ 3. Reports of strong sensation of needing to void __ 4. Often occurs in presence of mobility problems __ 1-D 2-A 3-B 4-C 1-B 2-A 3-D 4-C 1-C 2-D 3-B 4-A 1-B 2-D 3-A 4-C

ans: 1-B (stress incontinence/associated with lifting) 2-D (transient incontinence occurs during in an acute illness) 3-A(urge incontinence/reports strong sensation of needing to void) 4-C (functional incontinence/often occurs in presence of mobility problems)

Which of the following is a potential adverse effect of chronic use of higher-potency topical corticosteroids? Atrophy in intertriginous areas Epidermal sloughing Cutaneous fungal infection Acne rosacea

ans: Atrophy in intertriginous areas

Which of the following is not a component of the CRAFFT questionnaire when evaluating an adolescent for substance abuse?" Do you ever have trouble sleeping because you crave alcohol or drugs? Have you ever gotten in trouble while you were using alcohol or drugs? Do you ever use alcohol or drugs when you are alone? Do you ever feel the need to use alcohol or drugs to relax or feel better about yourself?

ans: Do you ever have trouble sleeping because you crave alcohol or drugs?

Mrs. Little is a 78-year-old woman with recently-diagnosed Alzheimer-type dementia (AD) who is here today for an office visit with her 55-year-old daughter. According to her daughter, Mrs. Little struggles with word-finding and has difficulty following directions. She appears "not to care about what is going on around her," while other times is engaged in family activities. According to her daughter, Mrs. Little will have an angry verbal outburst that is triggered by a minor problem. Her daughter states, "This is not like my mother. Usually she is very patient." When evaluating Mrs. Little, the NP considers that irritability in a person with early-stage dementia is often indicative of a: Mood disorder. Thought disorder. Normal pressure hydrocephalus. Hyperparathyroidism.

ans: Mood disorder.

The primary complaint of a patient with bacterial vaginosis is usually: Copious discharge. Painful intercourse. Intense pruritus. Profound vaginal odor.

ans: Profound vaginal odor.

In evaluating an 18-year-old man with testicular torsion, the NP considers the most appropriate course of action is: Watch and wait with re-evaluation in 2 days as it may resolve spontaneously Attempt manual detorsion Analgesics plus an anti-inflammatory agent Prompt referral to an urologist for surgical evaluation

ans: Prompt referral to an urologist for surgical evaluation

Richard is a 28-year-old man who presents with a chief complaint of left knee pain and swelling for the past month, as well as redness and tearing in the left eye for the past week. He also has had intermittent dysuria and 2-3 loose stools per day for the past 2 weeks. He denies weight loss, skin rash, or fever. Physical exam reveals a smooth, swollen, red, warm left knee with decreased range of motion, pupils equal and reactive to light with marked unilateral conjunctival redness. The urinary meatus is reddened. Richard's clinical presentation is most consistent with: Systemic lupus erythematosus. Polymyalgia rheumatica. Reactive arthritis (Reiter's syndrome). Psoriatic arthritis.

ans: Reactive arthritis (Reiter's syndrome).

You are counseling a 56-year-old woman who asks about increasing her dietary calcium intake with nondairy foods. You counsel about adding all of the following foods that are calcium-rich except for: Spinach. Tofu. Almonds. Red grapes.

ans: Red grapes.

Which of the following is not a potential acute gouty arthritis trigger? Use of a thiazide diuretic Consumption of organ meats Alcohol consumption Report of recently increased intake of acidic foods

ans: Report of recently increased intake of acidic foods

In keeping with HIPPA's requirements, all of the following are true except: The healthcare provider or designate cannot announce a patient's name in a waiting room. If interrupted during a patient visit, prior to leaving the examination room, the healthcare provider should lock or otherwise disable the electronic health record (EHR) to minimize the risk of unauthorized access to information. An employee of a healthcare facility is only able to access patient records for legitimate, job-related purposes. If a patient requests a paper copy of his/her record, the healthcare facility can charge a reasonable fee for this service.

ans: The healthcare provider or designate cannot announce a patient's name in a waiting room.

A 52-year-old woman who is Muslim arrives for an office visit. Her last primary healthcare visit was more than 10 years ago. She mentions that she does not want to disrobe or remove her head cover for a physical exam. You consider that: Her healthcare visit cannot proceed until the patient is able to disrobe for the physical exam. A mammogram should be ordered without prior breast examination. The option of having a modified physical examination with minimal disrobing should be discussed with the patient. The health history can be completed today and the physical examination deferred until a future office visit.

ans: The option of having a modified physical examination with minimal disrobing should be discussed with the patient.

A 38-year-old woman reports having 3-5 migraine episodes each month that can last 1-3 days. Her medical history is otherwise unremarkable other than bilateral tubal ligation (BTL) 6 years ago. She asks if there is any medication that can help to prevent these migraines. An appropriate prophylactic treatment options to offer this patient is: Ergotamine (Ergomar®). Sumatriptan (Imitrex®). Topiramate (Topamax®). Verapamil (Verelan).

ans: Topiramate (Topamax®

A 32-year-old woman presents with a recent onset of copious, green-yellow vaginal discharge. Physical examination reveals "strawberry spots" on the cervix. The NP anticipates finding the following on microscopic examination.

ans: Trichomoniasis (pics on the desk top) flagella tail bacteria

Which of the following is the best choice of antimicrobial therapy for a skin and soft tissue infection caused by community-acquired methicillin-resistant Staphylococcus aureus (MRSA)? Dicloxacillin Cefadroxil Trimethoprim-sulfamethoxazole Amoxicillin-clavulanate

ans: Trimethoprim-sulfamethoxazole

Since each state has distinct requirements for NP scope of practice, no questions specific to a certain state's practice act will appear on national certification exams. True False

ans: True

State level authorization dictates the ability of the NP to obtain a Federal DEA number. True False

ans: True

The majority of NPs are eligible to obtain a Federal DEA number. True False

ans: True

n evaluating Richard, the next most appropriate test to obtain is: Based on the below scenario: Richard is a 28-year-old man who presents with a chief complaint of left knee pain and swelling for the past month, as well as redness and tearing in the left eye for the past week. He also has had intermittent dysuria and 2-3 loose stools per day for the past 2 weeks. He denies weight loss, skin rash, or fever. Physical exam reveals a smooth, swollen, red, warm left knee with decreased range of motion, pupils equal and reactive to light with marked unilateral conjunctival redness. The urinary meatus is reddened. Serum antinuclear antibodies. Serum rheumatoid factor. Urinary PCR testing for N. gonorrhoeae and C. trachomatis. Stool for ova and parasites.

ans: Urinary PCR testing for N. gonorrhoeae and C. trachomatis.

You see a 28-year-old woman with HIV and being treated with antiretroviral therapy. At this visit, she mentions that she is considering pregnancy. In counseling the patient, you mention that antiretroviral therapy: Should be discontinued prior to attempting pregnancy Should be discontinued once pregnancy is confirmed Will likely continue throughout the pregnancy Should be discontinued only during the first trimester of pregnancy

ans: Will likely continue throughout the pregnancy

The most important option for the primary prevention of shingles is the use of: Varicella zoster immune globulin. Zoster vaccine. High-dose acyclovir at the onset of an outbreak. A tricyclic antidepressant post zoster episode.

ans: Zoster vaccine.

For a woman initiating bisphosphonate therapy for osteoporosis, treatment should continue: Until BMD improves by 25%. For 2 years. For 5 years. Indefinitely.

ans: for 5 years

You see a 24-year-old male with small flesh-colored verruca-form lesions on the shaft and tip of the penis. He reports some itching and discomfort in the genital area, but no voiding symptoms. The most likely diagnosis for this patient is: Syphilis Trichomoniasis Gonorrhea Genital warts

ans: genital warts


संबंधित स्टडी सेट्स

3421 Adults II Mod 7 - Shock, Sepsis, MODS, and Burns

View Set

464 prepu ch 13 - Key Pediatric Nursing Interventions

View Set

The divisions of the nervous system: central and peripheral (somatic and autonomic).

View Set

Chapters 9+10 Practice Quiz Questions

View Set

Institutes of the Christian Religion- John Calvin

View Set